Wikipedia:Auskunft/Archiv/2013/Woche 25

aus Wikipedia, der freien Enzyklopädie
Zur Navigation springen Zur Suche springen


Dies ist ein Archiv der Seite Wikipedia:Auskunft. Es enthält alle Abschnitte, die in der Kalender-Woche 25 im Jahr 2013 begonnen wurden.

Möchtest du in einer neuen Diskussion auf dieses Archiv verweisen? Nutze folgenden Link und ersetze ________ durch den Namen des Abschnittes. Die Nummer im Inhaltsverzeichnis gehört nicht dazu:

[[Wikipedia:Auskunft/Archiv/2013/Woche 25#________]]


← vorherige Woche Gesamtarchiv nächste Woche →

Schnittblumennahrung als Dünger?

Hallo, mir wurde gesagt, dass die "Schnittblumennahrung" die an Schnittblumen immer dabei ist einfach nur dafür sorgen soll, dass diese das Wasser besser aufnehmen können und es sich daher nicht so recht als Dünger eignet. Stimmt das? Ich würd die nämlich gern für meine Topfpflanzen verwenden. Inhaltsstoffe bei einer Packung die ich grad hab: "Pflanzenstärkungsmittel: Flower Food Clear 300", "Wirkstoffe: Dextrose 92%, Zitronensäure 5,3%, Salze 2,4%, Konservierungmittel 0,3%" ... keine wirklich erfolgversprechende Zusammenstellung :) -- 141.30.81.144 10:50, 17. Jun. 2013 (CEST)

Den Hauptbestandteil deines Pflanzenstärkungsmittel Dextrose braucht die Pflanze als Dünger nicht, den stellt sie selber her wenn sie nicht von ihren Photosyntheslingen (Blättern) abgeschnitten wurde. --Rubblesby (Diskussion) 11:22, 17. Jun. 2013 (CEST)
Für deine Topfpflanzen führt die Schnittblumennahrung nur zu Ansetzen von Schimmel, der die Dextrose mag. Das ist als Dünger völlig ungeeignet. --Sr. F (Diskussion) 12:13, 17. Jun. 2013 (CEST)

Arbeitsteilung

Das Buch Führer durch Tsingtau und Umgebung (1905) von Dr. Fr. Behme und Dr. M. Krieger enthält sehr schöne und detaillierte Karten. Hat jemand auf die schnelle die beiden "Eckdaten" der Autoren, damit ich mit dem Scannen beginnen kann? GEEZER... nil nisi bene 11:12, 17. Jun. 2013 (CEST)

Friedrich Behme, Maximilian Krieger. Krieger könnte ein Generalleutnant sein oder der Richter, den wir in Stephansort auf Neuguinea haben, GND 580459381, noch 1920 in der DNB nachgewiesen. Behme ist nicht im WBIS, Krieger nur als der Militär. --Aalfons (Diskussion) 11:30, 17. Jun. 2013 (CEST)

Tja, alles via GBS:

  • FRIEDRICH BEHME (1870-1958) Friedrich Behme fotografierte zwischen 1898 und 1904 in der damaligen deutschen Kolonie....(aus: Menschen, Orte, Zeiten: Fotografie am Deutschen Historischen Museum, 2009, Seite 351)
  • Dr. Friedrich Behme, ehem. Kaiserl. Richter in Tsingtau und Hilfsarbeiter im Reichsmarineamt und Auswärtigem Amt (Deutsch-englisches Schiedsgericht) Berlin, zuletzt Amtsgerichtsrat in Hannover. Neben seinem Hauptberuf als ... (Archiv für Landes- u. Volkskunde von Niedersachsen - Band 2 - Seite 118)
  • Tsingtau (...) Dolmetschereleve (...) Dr. Maximilian Krieger, Referendar, Dt. Kolonial-Handbuch, Erg.Band 1903, S. 196. Maximilian Krieger, Dr. jur., aus Ungarn, als höherer Verwaltungsbeamter bei dem Kaiserlichen Gouvernement in Kiautschou (Mitth. d. Seminars f Orientalische Sprachen 1902)
  • Dr. jur. Maximilian Krieger war früher im Kolonialdienst in ... Ein ominöser Paragraph des D.R.St.G. machte ihn den Dienst quittieren. Kr. ist Verfasser des Buches .Peking und Umgebung'58. Er war später Erzieher der Söhne des Gouverneurs von Kiautschou und ist jetzt Vertreter und Mitarbeiter des Ostasiatischen Lloyd, auch Korrespondent der Kölnischen Zeitung. Durch seine stets sehr jugendlichen... (Holzammer: Schedel, 2003)

Das war's. --Aalfons (Diskussion) 11:58, 17. Jun. 2013 (CEST)

Danke! Hmmm, Behme ist zu alt geworden, da muss man noch warten. GEEZER... nil nisi bene 12:01, 17. Jun. 2013 (CEST)
Dachte ich auch, aber sind denn die auch KArten von ihm? --Aalfons (Diskussion) 12:05, 17. Jun. 2013 (CEST)
Einige der farbigen (edlen!) Karten kleinerer Inseln da wurden in der geogr.-lith Anst. u. Steindr. von C.L. Keller, Berlin 5 erstellt. Ich habe jetzt mal den noch existierenden Verlag in Wolfenbüttel antelefoniert. Der Deal - falls möglich - wäre: Freigabe der Karten dieses Buches gegen einen WP-Artikel von Heckners Verlag ... :-)) à suivre ... GEEZER... nil nisi bene 12:36, 17. Jun. 2013 (CEST)

Self Editing

Was heisst: Self Editing.

--188.60.102.103 12:17, 17. Jun. 2013 (CEST)

In welchem Zusammenhang? --Dubaut (Diskussion) 12:21, 17. Jun. 2013 (CEST)
Wenn man danach googlet, findet man vor allem Ratgeber, die einem erklaeren, wie man als Autor seine eigenen Texte gewissermassen lektoriert und redigiert, also auf Fehler, Stil, Vollstaendigkeit, Geschwurbel usw. durchsieht. Alles Dinge, die eigentlich besser jemand anders, mit Aussensicht, macht. --Wrongfilter ... 12:26, 17. Jun. 2013 (CEST)

ich würde auch am ehesten vermuten, das "Self-Publishing" gemeint ist - siehe dazu etwa englische WP https://en.wikipedia.org/wiki/Self-publishing und Ratgeber im Netz --Cholo Aleman (Diskussion) 12:56, 17. Jun. 2013 (CEST)

Das sehe ich nicht so. Die technische und inhaltliche Arbeit an einer Publikation ist etwas anderes als der sogenannte Selbstverlag oder Eigenverlag, wo es im Schwerpunkt um das Verlegen und die Distribution geht. Was im Selbstverlag erscheint muß auch nicht zwingend per "self editing" hergestellt sein. --84.191.134.51 13:30, 17. Jun. 2013 (CEST)

Rosenberg-Tagebuch

Das Rosenberg-Tagebuch ist ja nun wieder aufgetaucht. Stand der Forschung in der Zwischenzeit auch kein Faksimile oder Nachdruck zur Verfügung? (Hat man sowas für die Nürnberger Prozesse nicht angefertigt, und immer nur das Original verwendet?) --Zulu55 (Diskussion) Unwissen 13:28, 17. Jun. 2013 (CEST)

Ich weiß es nicht definitiv, aber so weit ich sehe waren bisher nur die Auszüge bekannt, die im Nürnberger Prozeß benutzt wurden: "Um die Odyssee des Tagebuchs ranken sich viele Legenden, Gerüchte, Spekulationen. Die Alliierten hatten es am 10. August 1945 sichergestellt. Auszüge dienten im Nürnberger Hauptprozess gegen die NS-Täter als Beweismaterial. Dann, nach Rosenbergs Hinrichtung im Oktober 1946, verschwand es." Anders wären die geäußerten Erwartungen nach neuen Erkenntnissen und die Ankündigung einer mehrjährigen Auswertung ja auch nicht zu erklären. --84.191.134.51 13:45, 17. Jun. 2013 (CEST)
Zulu55, Dank für die Erinnerung, das wollte ich eh mal checken. Hier ist der Sachverhalt beschrieben. Es gab seriös bisher nur eine Auswahl-Dokumentation Kempners auf der Basis der im Prozess vorgebrachten Passagen. --Aalfons (Diskussion) 14:53, 17. Jun. 2013 (CEST)

Gedenktag des Jüdischen Namen Mirjam

Ich suche den Gedenktag 2013 für den jüdischen Namen Mirjam oder Mirijam.Bisher kein Erfolg über Suchmaschie oder Fragen im Bekannten-bzw. Freudeskreis. --79.247.134.34 13:42, 17. Jun. 2013 (CEST)--79.247.134.34 13:42, 17. Jun. 2013 (CEST)

--79.247.134.34 13:42, 17. Jun. 2013 (CEST)

Was für ein "Gedenktag" soll das sein? Etwa ein christlicher Namenstag? Dann siehe Miriam bzw. Maria. --FA2010 (Diskussion) 13:48, 17. Jun. 2013 (CEST)
Meinst du Claudia Mitscha-Eibl: Mirjam-Lied (Im Lande der Knechtschaft)?
Oder Mirjam (Prophetin)?
2. Mose (Exodus), 15,21 [1]: „Und Mirjam sang ihnen vor: Lasst uns dem HERRN singen, denn er hat eine herrliche Tat getan; Ross und Mann hat er ins Meer gestürzt.“ --Benutzer:Tous4821 Reply 15:00, 17. Jun. 2013 (CEST)
Das trifft alles nicht, aber ich kann auch nur darauf hinweisen, dass eine unserer jüngeren Pastorinnen/Pfarrerinnen mal einen besonderen Gottesdienst zum Mirjam-Tag abgehalten hat.--G-Michel-Hürth (Diskussion) 15:09, 17. Jun. 2013 (CEST)
Jetzt hab ich ihn: Der Mirjamsonntag fällt 2013 auf den 9. September Notiz der Ev. Kirche im Rheinland. --G-Michel-Hürth (Diskussion) 15:22, 17. Jun. 2013 (CEST) BITTE STREICHEN War archivierter Beitrag
Der Mirjamsonntag wird am 14. Sonntag nach Trinitatis gefeiert. Und der ist 2013 am 1.9. Kirchenkalender 2013--G-Michel-Hürth (Diskussion) 15:38, 17. Jun. 2013 (CEST)

Das noch so kleinste Detail

Gibt es das? Ein österreichischer Stahlkonzern macht derzeit Fernsehwerbung und behauptet, erkümmert sich bei allen Problem um jedes noch so kleinste Detail. Ich dachte bisher, dass man sich entweder um jedes kleine Detail oder auch um das kleinste Detail kümmert. --212.183.102.45 22:33, 17. Jun. 2013 (CEST)

Wahrscheinlich gehören die Angestellten des Konzerns auch zu den am besten angezogensten Personen. -- Geaster (Diskussion) 22:42, 17. Jun. 2013 (CEST)
Da sind sie aber auch die einzigsten. Chantk (Diskussion) 00:57, 18. Jun. 2013 (CEST)
Es gibt bis ins kleinste Detail und jedes noch so kleine Detail. Da ist den Werbefuzzis wohl eine Kontamination unterlaufen. --Jossi (Diskussion) 09:10, 18. Jun. 2013 (CEST)
Wohl eher sprachliche Schlampigstkeit. -- Geaster (Diskussion) 09:23, 18. Jun. 2013 (CEST)
Die aber zur vollsten Zufriedenheit des Auftraggebers. -- Ian Dury Hit me  11:09, 18. Jun. 2013 (CEST)
Vielleicht aber auch völligst beabsichtigst. --88.130.69.74 16:22, 18. Jun. 2013 (CEST)
Frage vollständigst beantwortet. --Optimum (Diskussion) 18:33, 18. Jun. 2013 (CEST)

Diese Abschnitt kann am archiviertesten werden. Geaster (Diskussion) 19:09, 18. Jun. 2013 (CEST)

Archivierung dieses Abschnittes wurde gewünscht von: erledigtst. --Rôtkæppchen68 20:46, 18. Jun. 2013 (CEST)

Wo doch Hyperlativ angesprochen wurde: eigentlich macht er durchaus Sinn, es gibt ja lokale und globale Maxima. Maximal für ein lokales, maximalst für ein globales :)--Alexmagnus Fragen? 10:17, 19. Jun. 2013 (CEST)

Globalstes?--Optimum (Diskussion) 10:44, 19. Jun. 2013 (CEST)
Das globalste Maximalstum. --Rôtkæppchen68 14:56, 19. Jun. 2013 (CEST)
Habt Ihr noch nicht genug? Echte Hyperlativisten scheinen die Hydrologen zu sein, wie man am Rheinfall sieht
Sprachlicher R(h)einfall
sieht... -- Geaster (Diskussion) 15:33, 19. Jun. 2013 (CEST)

Allergene

Welche Inhaltsstoffe einer Gesichtscreme können allergische Reaktionen verursachen?

--213.47.128.247 00:28, 17. Jun. 2013 (CEST)

Zunächst alle, die laut Inhaltsangabe drin sind, außer dem Wasser; beim Glyzerin ist es unwahrscheinlich, aber nicht auszuschließen.
Dann die Zutaten, die in Spuren vorhanden sind, ohne dass sie draufstehen (beispielsweise als Verunreinigungen anderer Zutaten oder der Herstellungs- und Abfüllanlage).
Schließlich auch, was beim Kauf der Packung noch nicht drin war, also Verunreinigungen, die durch die Verwendung hineingelangt sind, sowie Pilze o.ä., die daraus gewachsen sind. Chantk (Diskussion) 00:50, 17. Jun. 2013 (CEST)
Auch Wasser kann allergische Reaktionen hervorrufen. --87.156.58.149 18:09, 17. Jun. 2013 (CEST)
Unter Internationale Nomenklatur für kosmetische Inhaltsstoffe#Allergene Duftstoffe nach INCI findet sich eine Liste potentiell allergener Duftstoffe für Kosmetika. --Rôtkæppchen68 07:07, 17. Jun. 2013 (CEST)

Goldfische ohne Kopf im Gartenteich

Wir haben schon viele Jahre einen Gartenteich mit Golfischen, Moderlieschen, Stichlingen und 2 Stück Kois. In der letzten Zeit haben wir des öfteren mittelgroße Goldfische ohne Kopf gefunden. Das ist bisher noch nie vorgekommen. Wer kann der Übeltäter sein ? --217.234.133.195 14:36, 17. Jun. 2013 (CEST)

Siehe hier (und da). --Komischn (Diskussion) 14:47, 17. Jun. 2013 (CEST)
Ein Mem! --Aalfons (Diskussion) 14:59, 17. Jun. 2013 (CEST)
Kois also? Ich denke, dass eine Obduktion der Kois da Klarheit schaffen könnte. --Wicket (Diskussion) 15:54, 17. Jun. 2013 (CEST)

Ich würde ohne mit der Wimper zu zucken meine Katze dahinter vermuten. Allerdings bezweifle ich, das sie den Weg zu dir gemacht hat, es sei denn du wohnst in meiner Nachbarschaft. Ausserdem ist sie grad auf einem "Ich-Fress-Nur-Große-Nager"-Trip und bringt uns gern mal Zeisel an. Seit sie mitbekommen hat das sie die in der Wohnung nicht -ohne Ärger- gemütlich umbringen kann, kriegen wir nurmehr halbe Zeisel. Also mal Hinter - mal Vorderteil. Ansonsten wie Benutzer Wicket: Obduzieren lassen. Eine pauschale Mordanklage gegen das Katzenvolk kommt einer Kollektivschuld gleich. Das darf man nicht zu lassen. Marder schwimmen aber gerne. Allerdings fahren sie auh gern im Auto mit. Ach das ist alles so verworren. --Ironhoof (Diskussion) 21:56, 17. Jun. 2013 (CEST)

Zwischenfrage an Ironhoof: was ist ein Zeisel? --37.49.103.117 22:24, 17. Jun. 2013 (CEST)
Felin genuschelte Aussprache von Zeisig? Chantk (Diskussion) 22:28, 17. Jun. 2013 (CEST)
eher nicht, denn die gehören gewiß nicht zu den "Nagern" --37.49.103.117 22:31, 17. Jun. 2013 (CEST)
Ziesel mit Dreher. --Rôtkæppchen68 22:34, 17. Jun. 2013 (CEST)
Danke! --37.49.103.117 22:36, 17. Jun. 2013 (CEST)
Das ist ganz normal. Der Norden der Republik braucht Nachwuchs... Oder was meinst du, warum die Einwohner dort Fischköppe genannt werden? -- Ian Dury Hit me  11:12, 18. Jun. 2013 (CEST)

Webzugang über Mobiltelefon - welche IP?

Wenn ich beispielsweise aus der Turkei mit einem Deutsche Telekom Mobiltelefon jemanden in Italien anrufe, erscheint dem Empfänger der Anruf aus Deutschland zu stammen.

Wenn ich nun dieses Mobiltelefon mit meinem Laptop verbinde oder gar direkt im Telefon editiere, unter welcher IP wird das von den Webservern wahrgenommen? Erscheine ich dann hier als Kunde von Turkcell oder einem anderen Roamingpartner oder der Deutschen Telekom? --Eingangskontrolle (Diskussion) 15:55, 17. Jun. 2013 (CEST)

Laut diesem Dokument hängt es von den Roamingvereinbarungen der beteiligten Mobilfunkunternehmen ab. --Rôtkæppchen68 16:42, 17. Jun. 2013 (CEST)

Mein Fragen beziehen sich auf den Warp Antrieb Artikel.Warum wurde der Teil entfernt, der mit der Materieverteilung vor der Warp im Zusammenhang steht? Wurde das Problem bereits gelöst? Es geht auch leider nicht aus der Beschreibung hervor, wie genau sich dies Materieverteilung vor der Warp Blase darstellt. Ist hier eine Art "Schiene" wie bei einem Zug erforderlich ohne welche die Blase nicht beschleunigen kann? Wenn ja, ist diese Art "Schiene" über den ganzen Weg bis zum Ziel hindurch erforderlich oder reicht eine Art "Starthilfe" um auf die gewünschte Geschwindigkeit zu kommen? Das Problem wurde von Physikern wie Krasnikov oder Coule angesprochen, wo eine Art warp highway gefordert wird. In anderen Arbeiten (F. Loup) ist lediglich das Problem der erzeugung der Blase von außen, Steuerung und des Anhaltens genannt. In der englischen Fassung "Alcubierre drive" ist dieser Teil auch nach der Arbeit von Harold White von der NASA immer noch vorhanden.

Ich hoffe es sind nicht zu viele Fragen auf einmal. Danke im Voraus peter --82.113.106.190 18:15, 17. Jun. 2013 (CEST)

-> Status of "Warp Drive,Warp drive looks more promising than ever in recent NASA studies, Warp Field Mechanics 101, White–Juday warp-field interferometer.--Advanceddeepspacepropeller (Diskussion) 20:16, 17. Jun. 2013 (CEST)

Richtdiagramme von UKW-Sendern

Hallo an alle Radioexperten, im Falle einer Richtstrahlung bei UKW-Sendern sind in der Sendertabelle, in der die von dieser Sendeanlage ausgestrahlten UKW-Programme aufgeführt sind meist die Richtdiagramme angegeben, wie z.B. D (90–110°) Doch wo kommen diese Infos her? Gibt es eine Internetseite bzw. Verzeichnis, auf der die ganzen Richtstrahlungen angegeben sind? Vielen Dank im Voraus. --RadiohörerDiskBeiträge 19:13, 17. Jun. 2013 (CEST)

Üblicherweise findest Du diese Infos in den technischen Jahrbüchern der Senderbetreiber oder auf den Websites der zuständigen Landesmedienanstalt. --Rôtkæppchen68 20:13, 17. Jun. 2013 (CEST)
Die Webseite der schweizerischen Medienanstalt BAKOM beispielsweise gibt nicht viel her, auf der Infoseite des Senders Säntis z.B. werden keine Richtdiagramme angegeben, sondern nur die Programme und die Frequenzen. Trotzdem sind im Wikipediaartikel die Diagramme angegeben. --RadiohörerDiskBeiträge 20:38, 17. Jun. 2013 (CEST)
Die Webseite der Bundesnetzagentur hat ausführliche Daten zu allen Sendern, die im In- und grenznahen Ausland koordiniert worden sind, inklusive Strahlungsleistung in alle Richtungen im Abstand von zehn Grad. -- Liliana 21:08, 17. Jun. 2013 (CEST) addendum: viele Richtdiagramme sind auch schon bei FMscan eingetragen - da kann man die sich auch grafisch darstellen lassen, was für manchen einfacher zu lesen ist (besonders für die, die dBW nicht in dBµV umrechnen können). -- Liliana 21:10, 17. Jun. 2013 (CEST)
(BK)Im Falle des legendären Senders Säntis ist wohl Swisscom der Betreiber. Die BAKOM hat hier unter UKW-Koordinationsdaten entsprechende Tabellen. --Rôtkæppchen68 21:12, 17. Jun. 2013 (CEST)
Hallo Liliana, erstmal Vielen Dank für die schnelle Antwort. Wie kann man sich bei FMScan das fertige Richtdiagramm anzeigen lassen (so wie es in den Artikeln steht)? Und woran erkennt man in der Liste, ob rund oder gerichtet gesendet wird? (Ein ND oder D sehe ich in der Liste nicht). Muss man dafür in den Expert Mode? Welche Optionen muss man dort markieren? Habe bisher nur die Normale Version von FMScan genutzt und kenne mich daher nicht mit der Expertenversion aus. --RadiohörerDiskBeiträge 21:26, 17. Jun. 2013 (CEST)
In der Liste einfach die Sendeleistung anklicken, dann öffnet sich eine neue Seite. -- Liliana 21:38, 17. Jun. 2013 (CEST)
Hallo, eines habe ich zumindest verstanden: Wenn Rund gesendet wird, ist die Sendeleistung in allen Richtungen des Kreises gleich hoch. Die Richtstrahlung habe ich aber nicht ganz verstanden: Was bedeutet dieser rote Strich und die innere Markierung? Wenn wir jetzt bei dem Beispiel bleiben: Der Sender Säntis hat ein Richtdiagramm von 160-330 Grad, wie kommt man auf die Zahl bzw. wie liest man sie aus dem Kreis heraus? --RadiohörerDiskBeiträge 22:01, 17. Jun. 2013 (CEST)
Das ist ganz einfach: man schaut sich die Richtungen an, in die mehr als die Hälfte der Sendeleistung abgestrahlt wird. Dies ist deswegen so, weil Einzüge, die weniger als 3 dBW betragen (=die Hälfte der Sendeleistung), ignoriert werden können. Wenn man sich das Beispiel beim Säntis anschaut, sieht man, dass zwischen 160° und 330° stets mehr als 30 kW abgestrahlt werden. Das heißt, in diese Richtung wird mit voller Leistung gesendet, in alle anderen Richtungen bestehen Einzüge.
Ach so: der rote Strich ist irrelevant, der zeigt nur an, in welche Richtung du lägst. -- Liliana 06:53, 18. Jun. 2013 (CEST)
Vielen Dank! --RadiohörerDiskBeiträge 08:26, 18. Jun. 2013 (CEST)

Futuristischer Klang und dessen Kategorisierung gesucht

Hallo,

ich bin schon sehr lange auf der Suche nach der speziellen Kategorie in der man diese Art von Musik packen würde. Es geht um die ersten 28 Sekunden dieses Liedes. Ähnliche Klänge habe ich schon bei dem Film "Oblivion", "Tron Legacy ( Am I not Human), den kompletten Soundtrack von Deus Ex Human Revolution gehört, also gehe ich davon aus, dass diese Melodieform eine genauere Bezeichnung besitzt.

Weiß jemand zufällig mit welchen Musikprogrammen man diese Klänge erzeugen kann?

Danke im VOraus --37.80.97.234 20:14, 17. Jun. 2013 (CEST)

Synthesizer. Programme sind im Artikel verlinkt deren Qualität habe ich aber jetzt nicht getestet --78.34.19.11 20:34, 17. Jun. 2013 (CEST)
Das Musikprogramm DAW ist nicht so entscheidend. Wichtig ist, wie oben gesagt, der Syntesizer, egal ob als Instrument oder virtuell VST. Manche Musikprogramme bringen eigene VST mit. Dann ist es eine perkussive (Hüllkurve) Sägezahnwelle, die mit Filter (Cutoff) bearbeitet wird. Das ganze der Einfachheit halber in ein Arpeggio und fertig.--Wikiseidank (Diskussion) 08:22, 18. Jun. 2013 (CEST)

Gymnasium in Burundi

Ich versuche herauszufinden, wo in Burundi das Lycee of Rusengo ist. Ich kann weder den Ort noch die Schule finden. Das taucht auf im Lebenslauf des burundischen Botschafters in Deutschland, Edouard Bizimana, über den ich gerade einen Artikel schreibe. Lebenslauf. --Gereon K. (Diskussion) 22:02, 17. Jun. 2013 (CEST)

Openstreetmap hat hier eine École normale Rusengo, Burundi. --Rôtkæppchen68 22:13, 17. Jun. 2013 (CEST)
Prima. Vielen Dank. Kann ich dann mit Provinz Ruyigi näher beschreiben. --Gereon K. (Diskussion) 22:18, 17. Jun. 2013 (CEST)
Beide wohl in derselben Provinz. Eine école normale (Normalschule) ist eine Ausbildungsanstalt für Lehrer, während das Lycée, wo er wohl sein baccalaureat gemacht hat, eine Art Gymnasium ist. GEEZER... nil nisi bene 22:55, 17. Jun. 2013 (CEST)
Das Lycee hat eine Fcebookseite. Vielleicht könnte der Fragesteller – sofern er ebenfalls Facbookuser ist – mal dort anfragen. --Rôtkæppchen68 23:26, 17. Jun. 2013 (CEST)
Scheint aber nicht sehr beliebt zu sein... GEEZER... nil nisi bene 23:34, 17. Jun. 2013 (CEST)
Mit dem Satz "Edouard Bizimana absolvierte 1990 das Lyzeum in Rusengo in der burundischen Provinz Ruyigi." bin ich schon zufrieden, mehr muss ich für den Artikel nicht wissen. --Gereon K. (Diskussion) 23:42, 17. Jun. 2013 (CEST)

Notaufsichtsrat einer Genossenschaft

Wieviel Mitglieder muß ein solcher haben? 1, die gesetzliche Mindestanzahl von 3, oder soviel wie in der Satzung festgelegt ist?85.178.99.128 01:24, 18. Jun. 2013 (CEST)

Bestimmung des zuständigen Amtsgerichtes, Mietsache

Hallo!

Wie wird denn das zuständige AG in einem Mietstreitverfahren bestimmt, wenn der Kläger und der Beklagte aktuell unterschiedliche Meldeadressen haben? Nach einem Umzug aus dem ursprünglichen Bundesland weg sind noch Streitpunkte mit der alten Vermietungsgesellschaft offen. Kann man als Betroffener beantragen, dass das AG in der Nähe des eigenen Wohnortes zuständig ist? Grüße und danke für alle Informationen vorab :-) , 188.106.246.0 14:25, 18. Jun. 2013 (CEST)

Steht das nicht im (Miet)Vertrag ("Gerichtsort für PiPaPo ist XYZ.") ??? GEEZER... nil nisi bene 14:30, 18. Jun. 2013 (CEST)
disclaimer: hab nur ahnung vom österreichischen recht. das wird aber in DE nicht viel anders sein. gerichtsstand ist der ort, an dem die bestandssache gelegen ist. also an dem gericht, das für den ort, in dem die mietwohnung liegt, zuständig ist. im mietvertrag wird sowas wegen der zwingenden gesetzlichen vorschrift nicht explizit erwähnt. aber wie gesagt, am besten mit diesen infos googlen und für DE verifizieren. lg, --kulacFragen? 14:34, 18. Jun. 2013 (CEST)

http://dejure.org/gesetze/ZPO/29a.html --Vsop (Diskussion) 14:38, 18. Jun. 2013 (CEST)

Dieser Paragraph ist übrigens auch im Artikel Gerichtsstand verlinkt. --Rôtkæppchen68 15:11, 18. Jun. 2013 (CEST)

Smartphone legt WLAN lahm

Ich habe in meinem Haus-WLAN KabelBW über eine Fritzbox und versorge 1 PC, 1 Netbook, 1 iPad, 1 Samsung Galaxy Ace 2 und 1 Samsung Galaxy Ace. Jedes Gerät hat akzeptablen Speed und auch Online Spiele sind kein Problem. Sobald aber das letzte Gerät, das Samsung Galaxy Ace sich mit dem WLAN verbindet kann dieses zwar problemlos surfen, aber alle anderen Geräte sind schachmatt gesetzt und verlieren die Verbindung. Ich versteh einfach ums verrecken nicht warum. Das WLAN hat nen MAC-Adressen-Filter und ist durch ein etlich langes Passwort WAP2 verschlüsselt gesichert. Jmd. ne Idee? Vielen Dank im voraus! --109.193.59.194 18:48, 18. Jun. 2013 (CEST)

Hi, Energiespar-Modi sind gerne mal Auslöser für solche Effekte. Schalte mal versuchseweise sowohl den Access Point als auch Clients auf "volle Leistung" bzw. deaktiviere Energiespar- und Öko-Einstellungen. Viel Erfolg! —[ˈjøːˌmaˑ] 18:54, 18. Jun. 2013 (CEST)
Danke für den Tipp, werd das alles mal ausschalten. Hab zudem ne'n Firmware Update gemacht, das ging erst nicht, musste das iwi manuell machen. Mal schauen ob es jetzt noch zu Störungen kommt. lg--109.193.59.194 22:41, 18. Jun. 2013 (CEST)
Prima, halte uns gerne auf dem Laufenden! :) —[ˈjøːˌmaˑ] 22:47, 18. Jun. 2013 (CEST)
Okay, jetzt scheint es wieder gleichzeitig zu funktionieren. Da ich aber beides gemacht habe, weiß ich nicht an was es lag! ;) Danke für die Hilfe! Kann archiviert werden lg--109.193.59.194 00:26, 19. Jun. 2013 (CEST)
Archivierung dieses Abschnittes wurde gewünscht von: -- Ian Dury Hit me  13:00, 19. Jun. 2013 (CEST)

Fragen bezüglich Zombies

Ich habe mal zwei Fragen bezüglich Zombies:

1): wie reagieren Zombies auf elektrischen Strom? Würde ein direkt an die Steckdose angeschlossenes Gitter von Drähten sie aufhalten oder zumindest bremsen?

2): wie sicher/unsicher sind größere Glasfenster/Glastüren im zweiten Stockwerk im Falle einer Zombie-Epidemie?

--134.91.144.18 08:41, 19. Jun. 2013 (CEST)

Zombie hast du gelesen? "...wird die fiktive Figur eines zum Leben erweckten....". ad1;im The Zombie Survival Guide [2] wird sowas erwähnt; kann mich aber leider nicht mehr erinnern...sry:).--gp (Diskussion) 08:56, 19. Jun. 2013 (CEST)
Ja, das ist praktisch die "Zombie-Bibel" (Zombibel??). S. 56 : Hat stunning effect, also nur kurzfristig. Hat man aber einen Hochspannungslinie zur Hand (bitte metaphorisch verstehen...) und man kann das Gehirn des Untoten verkohlen, so ist der Effekt permanent. Be prepared!! GEEZER... nil nisi bene 09:20, 19. Jun. 2013 (CEST)
"Zombies" sind eine Erfindung der Unterhaltungsindustrie und genau wie Vampire keine real existierenden Lebensformen. Entsprechend werden diesen Fabelwesen immer wieder neue bzw. andere Eigenschaften angedichtet. Die Frage kann also höchstens im Zusammenhang mit einem Film, einer Serie oder einem Buch und auch nur wenn das dort vorkommt beantwortet werden. Beispielwseise bewegen sich "Zombies" in klassichen Werken sehr langsam, im neuen Blockbuster World War Z (Film) dagegen sehr schnell. --Kharon 08:59, 19. Jun. 2013 (CEST)
<quetsch>Eine Erfindung der Moderne? Zombies gibt es seit Menschengedenken und an manchen Orten so viele, dass man ihr hin und her durch Verkehrsschilder regeln muss (siehe: Bildbeweis)! Geoz (Diskussion) 21:21, 20. Jun. 2013 (CEST)
Du kannst Zombies nicht dauerhaft aufhalten. 1) Strom würde sie sicher durchschütteln (nimm besser den Starkstrom vom Herd) und kurz abhalten. Da ihr Mordtrieb aber größer ist als ihre Lernfähigkeit würden sie sich immer wieder in dein Gitter stürzen, bis ein Kurzschluss den Strom abstellt oder das Gitter aufgebogen/gerissen wurde. 2) Zweiter Stock ist schonmal gut. Die Schwachstelle wird dann auf Dauer aber wohl nicht das Fenster sein, sondern der Zugang durchs Haus, egal wie gut verbarrikadiert. Letztlich würden sich die Zombies aber auch unter dem Fenster türmen, und so an die lebenden Menschen im Obergeschoss zu gelangen. Auf Glas wird solange eingedroschen, bis es kaputt geht. Fazit: Bei Zombieinvasionen hilft Verbarrikadieren nur kurzfristig. Mittel- bis langfristig bist du auf einen Krisenplan der Zombieexperten der Regierungen und Geheimdienste angewiesen. Ob diese rechtzeitig eine wirksame Methode gegen die Invasion entwickeln oder ob die Zombies die ganze Menschheit ausrotten werden, ist freilich ungewiss. --Krächz (Diskussion) 09:20, 19. Jun. 2013 (CEST)
ja, aber ein paar Eigenschaften bleiben da doch gleich, sonst würde man das Wesen doch nicht wiedererkennen, oder? Den Unterschied "schnelle Zombies-langsame Zombies" würde ich jetzt nicht als so drastisch betrachten.
und falls es bezüglich meiner Fragen unterschiedliche Darstellungen gibt, bräuchte ich dannn die relevanten unterschiedlichen Fälle.

--134.91.144.18 09:13, 19. Jun. 2013 (CEST) es gab vor kurzem übrigens eine echte, wissenschaftliche studie, die die ausbreitungsgeschwindigkeit eines zombieproblems analysiert hat. ich glaube annahme war sowas wie eine viruserkrankung, die ja durchaus real sein könnte. ergebnis war, dass die menscheit sogar bei den langsamen zombies nur wenige tage überleben würde und bis dahin fast alle infiziert wären. tolle aussichten...übrigens lässt das z.b. die geschichte hinter den terminatorfilmen gleich in einem ganz anderen licht dastehen. man stelle sich vor, die "zombies" sind nicht nur schwer bewaffnet und gepanzert, sondern auch noch intelligent...lg, --kulacFragen? 11:51, 19. Jun. 2013 (CEST) PS: hier ist das paper. übrigens hier noch was zu meinem terminator-off topic.

Ad (2) Glastüren/fenster helfen nicht gegen schnelle Zombies, gegen langsame Zombies sind sie aber relativ effektiv. Persönlich bin ich durch Beobachtungen der Meinung, dass sie schon unter uns sind. GEEZER... nil nisi bene 12:09, 19. Jun. 2013 (CEST)

Das sind wohl eher Fragen aus dem "Big-Bang-Universum" bevorzugt von Raj gestellt. Vor allem, was passiert mit Zombies, wenn sie ihren "Blutdurst" nicht stillen können? Geht es ihnen dann schlechter, bekommen sie Hautunreinheiten oder Spliss in den Haaren?--Wikiseidank (Diskussion) 12:28, 19. Jun. 2013 (CEST)

Das mergelt sie natürlich aus. Ob der im Saft stehende oder der aus dem letzten Loch pfeifende Zombie die gepflegere Erscheinung ist, ist vermutlich Ansichtssache. --Krächz (Diskussion) 13:36, 19. Jun. 2013 (CEST)
Nein, Raj würde fragen: "Können Zombies tauchen?" -- Janka (Diskussion) 12:40, 19. Jun. 2013 (CEST)
Bist du im Obergeschoss, GEEZER? Yotwen (Diskussion) 13:07, 19. Jun. 2013 (CEST)
Solche Berechnungen hängen von viel zu vielen Variablen ab und mir ist keine Simulation bekannt, die auch nur annähernd genug Szenarien durchspielt um aussagekräftig zu sein. Variablen sind zum Beispiel:
  1. Inkubationszeit? Wie schnell stirbt man vom Biss, wie schnell steht man dann wieder auf?
  2. Wie viele werden tatsächlich zum Zombie bevor die anderen sie komplett gefressen haben und von welchen Faktoren hängt das ab?
  3. Wie gut können die Zombies Barrikaden überwinden? Lehnen sie sich so lange gegen ein Fenster bis es bricht oder schlagen sie aktiv dagegen? Können sie klettern?
  4. Wie gut sind die Sinne der Zombies? Agression erst durch Sichtkontakt oder Verfolgung des Geruchs über Kilometer?
Und so weiter... —PοωερZDiskussion 14:05, 19. Jun. 2013 (CEST)
"We model a zombie attack using biological assumptions based on popular zombie movies.We introduce a basic model for zombie infection and illustrate the outcome with numerical solutions." --Advanceddeepspacepropeller (Diskussion) 14:40, 19. Jun. 2013 (CEST)

Zu 1: Sie werden beim Kontakt mit Strom noch größer und stärker, LAUF LIEBER SCHNELL WEG! Zu 2: Wenn die Zombies davor den stromdurchflossenen Draht berührt haben, bist du nicht mehr sicher. Sie können dann 7 Meter hoch springen. LAAAAAUF!!! (nicht signierter Beitrag von AndoA (Diskussion | Beiträge) 13:55, 19. Jun. 2013 (CEST))

Letzteres Posting ist unseriös. Hier bittet jemand sachlich um Auskunft und du versuchst ihn kirre zu machen. Solche Schauermärchen! 7 Meter hoch springende Zombies - ich bitte dich! Je nach Zombie-Gattung würde ich mal von 0 bis maximal 1,5 Metern ausgehen. Mit vorherigem Stromkontakt hat das nichts zu tun. Die Qualität unserer Auskunft sinkt dramatisch. --Krächz (Diskussion) 14:16, 19. Jun. 2013 (CEST)

Ein Zombie konnte immerhin über Wasser gehen und in den Himmel fahren. Ich glaube kaum das man gegen den was mit elektrischem Strom oder sonstwas aufhalten kann. Das gute ist, dass er sich seit rund 2000 Jahren von der Erde fernhält. ;-) --Friechtle (Diskussion) 17:04, 19. Jun. 2013 (CEST)

Er will aber wiederkommen - und dann auch dich vorladen. --Eingangskontrolle (Diskussion) 23:19, 19. Jun. 2013 (CEST)
Die Titanic hatte auch mal vor anzukommen, dennoch wissen wir beide, dass das nie passieren wird. —PοωερZDiskussion 00:38, 20. Jun. 2013 (CEST)
Den Zombies bei Justin Cronin traue ich 7-Meter-Sprünge zu. --Gereon K. (Diskussion) 11:57, 20. Jun. 2013 (CEST)
Archivierung dieses Abschnittes wurde gewünscht von: Solltest du bereits volljährig sein, melde dich bitte umgehend bei deinem Arzt. :-) Cherryx sprich! 00:54, 20. Jun. 2013 (CEST)

Bei Pulp Fiction fehlt in der Angabe John Travolta

--176.199.158.90 18:47, 19. Jun. 2013 (CEST) Auf der Seite von Quentin Tarantino fehlt bei dem Pulp Fiction einer der Hauptdarsteller John Travolta. Ich bin zum ersten Mal hier. Gruß Werner

Auf der Seite Quentin Tarantino steht bei keinem Film Hauptdarsteller dabei. Deine Frage ist also nicht nur hier falsch (siehe Kasten oben), sondern auch noch unverständlich. --AndreasPraefcke (Diskussion) 18:58, 19. Jun. 2013 (CEST)
Archivierung dieses Abschnittes wurde gewünscht von: --AndreasPraefcke (Diskussion) 18:58, 19. Jun. 2013 (CEST)
Moeglicherweise bezieht sich die Frage auf die Tabelle der mehrfach eingesetzten Darsteller. Dann duerfte die Antwort auf die Frage in dem Wort "mehrfach" stecken. --Wrongfilter ... 19:46, 19. Jun. 2013 (CEST)

Gibt es eigentlich einen Ersatzkanzler?

Hallo, ich wollte mal fragen, ob es in Deutschland sowas wie einen Ersatzkanzler von Seiten der Senior-Regierungsfraktion gibt? Wer würde aus den Reihen der CDU/CSU das Kanzleramt übernehmen von Frau Merkel aus welchen Gründen auch immer etwas zustößt? Herr Rösler ist zwar Vizekanzler, aber ich glaube nicht, dass er diese Amtsperiode fertig regieren dürfte. --91.103.112.54 08:55, 17. Jun. 2013 (CEST)

„Die Vertretungsvollmacht des Vizekanzlers erstreckt sich grundsätzlich auf alle Aufgaben bzw. Rechte des Kanzlers.“ Doch klar: der Vizekanzler füllt die Rolle des Kanzlers komissarisch aus. Zwar anders also als z.B. beim US-Amerikanischen Vizepräsidenten, der im Falle des Ablebens des Päsidenten sofort zum neuen Präsidenten vereidigt wird, aber das ist nur ein Titelunterschied. —PοωερZDiskussion 09:03, 17. Jun. 2013 (CEST)
Wobei es ja auch deutlich weniger Aufwand ist, und schneller geht, wenn das Parlament einen neuen Kanzler wählen muss, als wenn dass das Volk machen muss. -- southpark 09:08, 17. Jun. 2013 (CEST)
Das wäre ja dann auch ein designierter Nachfolger für eine "reguläre" Nachfole. Den gibt es (zumindest öffentlich) derzeit wohl nicht. De Maizière galt mal als Kandidat, aber ich hab meine Zweifel, ob das noch gilt. --Eike (Diskussion) 09:12, 17. Jun. 2013 (CEST)
Bundeskanzler_(Deutschland)#Ende_der_Amtszeit ist sehr interessant: Anscheinend ist das alles weniger klar geregelt als man naiv annehmen möchte. --Wrongfilter ... 09:14, 17. Jun. 2013 (CEST)
Wenn das stimmt, was da steht, ist der Vizekanzler eben nicht der automatische Ersatzkanzler, weil er seine Funktion als Vizekanzler verliert, wenn der (‚echte‘) Kanzler ausfällt.--141.20.106.180 11:22, 17. Jun. 2013 (CEST)
Wenn das stimmt, was da steht, dann ist er es praktisch eben doch, weil das Auswahlrecht des Bundespräsidenten nach h.M. beschränkt ist. Ein Automatismus steckt freilich nicht dahinter, er muss schon noch verpflichtet werden. -- Ian Dury Hit me  11:44, 17. Jun. 2013 (CEST)
  • Die Frage war eher, ob im aktuellen Fall CDU/CSU einen Ersatzkanzler in der Tasche haben. Herr Rösler könnte zwar einige Tage Interim Regierungschef sein, aber das Anrecht auf den Kanzlersessel hat ja die Union und nicht die FDP. Das hieße in paar Tagen müsse der Bundestag einen neuen Kanzler wählen. Die Frage ist, ob der schon feststeht falls Frau Merkel ausfällt oder man diesen erst auserkoren müsste nachdem Frau Merkel ausgefallen ist. --91.103.112.54 12:54, 17. Jun. 2013 (CEST)
Kommunal habe ich im letzten Jahr folgende Regelung erlebt: Der bisherige Oberbürgermeister von Kiel, Torsten Albig, wurde zum Ministerpräsidenten gewählt. Eine Neuwahl für seinen bisherigen Posten fand im Herbst statt. Dazwischen übte Albigs Stellvertreter, Todeskino von den Grünen, alle Verpflichtungen aus. Ob dies allerdings auch auf Bundesebene übertragbar ist, weiß ich nicht. Euroklaus (Diskussion) 13:37, 17. Jun. 2013 (CEST)
Es gibt kein "Anrecht". --FA2010 (Diskussion) 13:39, 17. Jun. 2013 (CEST)
+1, Hier wird hier einiges unrichtig dargestellt. Der Vizekanzler bzw. korrekter: Stellvertreter würde in den gegebenen Fällen mitnichten automatisch Kanzler werden.
Zentral erscheint mir Art. 69 (1) und (3) GG:
"(1) Der Bundeskanzler ernennt einen Bundesminister zu seinem Stellvertreter.
[...]
(3) Auf Ersuchen des Bundespräsidenten ist der Bundeskanzler, auf Ersuchen des Bundeskanzlers oder des Bundespräsidenten ein Bundesminister verpflichtet, die Geschäfte bis zur Ernennung seines Nachfolgers weiterzuführen."
Demnach muß "die Geschäfte bis zur Ernennung seines Nachfolgers" nicht zwingend der Stellvertreter sondern "ein Bundesminister" weiterführen, der von dem Bundeskanzler oder dem Bundespräsidenten benannt wird. Wer das dann ist entscheiden Kanzler oder Präsident und aus der Funktion des "Stellvertreters" wächst juristisch kein Anrecht, allerhöchstens aus politischen Erwägungen heraus.
Außerdem greift möglicherweise auch Art. 115h (2) (ergänzt durch Siebzehntes Gesetz zur Ergänzung des Grundgesetzes vom 24. Juni 1968, "Notstandsgesetz"):
"(2) Wird eine Neuwahl des Bundeskanzlers durch den Gemeinsamen Ausschuß erforderlich, so wählt dieser einen neuen Bundeskanzler mit der Mehrheit seiner Mitglieder; der Bundespräsident macht dem Gemeinsamen Ausschuß einen Vorschlag. Der Gemeinsame Ausschuß kann dem Bundeskanzler das Mißtrauen nur dadurch aussprechen, daß er mit der Mehrheit von zwei Dritteln seiner Mitglieder einen Nachfolger wählt."
Auch hier spielt also die Funktion des Stellvertreters keine Rolle. --84.191.134.51 14:29, 17. Jun. 2013 (CEST)
Nun lest doch spaßeshalber mal die Frage...
Es geht nicht um die rechtliche Nachfolgereglung, sondern wen die Union aufstellen würde.
--Eike (Diskussion) 14:37, 17. Jun. 2013 (CEST)

Spekulation. Frau Merkel hat es ja gerade geschafft, alle in Frage kommenden Männer kaltzustellen.--Antemister (Diskussion) 18:39, 17. Jun. 2013 (CEST)

Nur die Maenner? Frau von der Leyen war auch mal als heisse Kandidatin auf die Nachfolge gehandelt worden. Wenn es jetzt auf einmal notwendig waere, wuesste ich wirklich nicht mehr, wer aus der CDU das Kaliber und die politische Unversehrtheit haette, zu uebernehmen. (Am Ende gar eine Chance fuer Horst aus Bayern, in die Luecke zu springen?) -- Arcimboldo (Diskussion) 03:28, 18. Jun. 2013 (CEST)

Zunächst: Das Grundgesetz kennt die Bezeichnung „Vizekanzler“ nicht. Das ist eine Erfindung von Medien auf der Suche nach plakativen Etiketten (vgl. das dümmliche „First Lady“). Der Vertreter des Bundeskanzlers übt im Wesentlichen ausschließlich formale Aspekte der Bundeskanzlerschaft aus (Leitung von Kabinettssitzung in Abwesenheit der Kanzlerin, vor allem). Da der Bundestag bei jeder Sitzung einen neuen Kanzler wählen kann, ist die Frage eines „Vizekanzlers“ im Allgemeinen ohne besondere Bedeutung: Scheidet ein Kanzler, warum auch immer, aus dem Amt, dann wählt der Bundestag bei der nächsten Sitzung einen neuen. Für Sonderfälle (Notstandsgesetze) wurde oben schon das Relevante geschrieben). --Freud DISK Konservativ 12:06, 19. Jun. 2013 (CEST)

schikane am arbeitsplatz

Also mein freund war ganz normal arbeiten bei 52grad innen temaratur wollte ich im nur etwas zu trinken und zu essen vorbei bringen was ne sekunde gedauert hätte. Er wollte für die sekunde an die tür kommen und es endgegen nehmen darauf hin hat sein meister direkt rausgeschmissen ... DARF MAN DAS????????? (nicht signierter Beitrag von 80.187.106.54 (Diskussion) 16:54, 17. Jun. 2013 (CEST))

Rechtshinweis bitte den hinweis zu rechtsthemen beachten!
wen hat der rausgeschmissen? die freundin oder den freund? 52°C hört sich für mich nach Hitzefrei an, außer am Hochofen... :-) --Heimschützenzentrum (?) 17:03, 17. Jun. 2013 (CEST)
Den Arbeitnehmer rausschmeißen darf der Meister nicht so ohne weiteres. Den Besucher mit Hausverbot belegen (=rausschmeißen) darf der Meister aber. Ich kenne so einen Fall, wo der Lebensgefährte dauernd im Büro aufgekreuzt ist, bis der Chef ihm Hausverbot erteilt hat. --Rôtkæppchen68 17:20, 17. Jun. 2013 (CEST)
Hier steht Einiges zu den Kündigungsgründen und vorher muss wohl eine Abmahnung ausgesprochen werden. --Merrie (Diskussion) 17:35, 17. Jun. 2013 (CEST)
Klingt irgendwie nicht nach einem regulären Arbeitsverhältnis. Hat dein Freund einen Arbeitsvertrag? Yotwen (Diskussion) 17:43, 17. Jun. 2013 (CEST)
Kündigungsbedingungen kommen auf den Arbeitsvertrag und das Arbeitsverhältniss an. Wichtig ist sowas sofort dem Arbeitsamt mitzuteilen und umgehend gegen die Kündigung zumindest versuchsweise beim Arbeitsgericht zu klagen. Arbeitgeber stehen in letzter Konsequenz zwar meißtens am längeren Hebel aber dann hat man zumindest nicht darüber hinaus auch noch Ärger mit dem Arbeitsamt. Davor sollte man natürlich aber auch versuchen mit dem Arbeitgeber diplomatisch mit Hinweiß auf die Extremsituation und Papierkrieg zu reden und den Streit so ohne letzteren zu bereinigen. --Kharon 17:55, 17. Jun. 2013 (CEST)

Äh, ich gehe davon aus die Besucherin wurde vom Meister rausgeschmissen. Das darf er, nach Hausrecht. In den meisten Betrieben sind Besucher nicht allzu willkommen (Sicherheit, Industriespionage...).--Antemister (Diskussion) 18:35, 17. Jun. 2013 (CEST)

Sie sage er wurde... Hmm, wenn da wirklich nichts anders war? Keine Probezeit? Keine Abmahnung? Sonst würde ich ich jemand Fragen und die gesamte Situation schildern. --Hans Haase (Diskussion) 00:31, 18. Jun. 2013 (CEST)
52 Grad Innentemperatur hört sich für mich jetzt auch nicht nach einem gemütlichen Büro an, wo man einfach mal so herein- oder herausspaziert - wie schaut es mit Sicherheitsrisiken für Besucher und auch Arbeiter aus? Und: Wenn ich von meinem Schreibtisch mal kurz aufstehe um was zu trinken zu holen dauert das auch sicher deutlich länger als eine Sekunde, ist aber kein Problem - wenn ich im Team irgendwas schweiße/montiere/ausmesse bedeutet das, das alle nicht weiterarbeiten können. Dafür gibt es Pausenzeiten. Ist aber alles müßige Spekulation, wenn wir nicht wissen, wer wen wo rausgeschmissen hat.feba disk 01:04, 18. Jun. 2013 (CEST)
"Pausenzeiten" ist wohl das gute Stichwort. Man wird nirgendwo dafür bezahlt, daß man Essen und Trinken vorbeigebracht bekommt. Dabei ist unerheblich, wie lange das dauert. Sicherlich gibt es Betriebe und Meister, die das großzügiger handhaben, aber das hat dann mit dem erwünschten Betriebsklima zu tun und nichts mit dem Arbeitsvertrag. In diesem stehen Pausenzeiten und in denen kann man sich Essen und Trinken an den Eingang des Betriebsgländes bringen lassen. Das ist erlaubt. Abmahnung ist das zweite gute Stichwort. Eine fristlose Kündigung ohne vorherige Abmahnung wäre nur bei Vorliegen eines wichtigen Grundes zulässig. Das ist hier offenbar nicht gegeben, es sei denn, der Freund wäre dem Meister ernsthaft an die Gurgel gegangen. Ein Verweis vom Arbeitsplatz ist zwar möglich (z.B. wenn der Arbeitnehmer betrunken ist und seinen arbeitsrechtlichen Verpflichtungen nicht nachkommen kann), hier scheinen mir trotz der Diffusität der Fallbeschreibung aber ebenfalls keine Voraussetzungen dafür gegeben. Daß betriebsfremde Personen vom Betriebsgelände verwiesen werden können, bedarf wohl keiner näheren Erklärung. Bei diesem Vorfall bereits von "Schikane" zu sprechen zeugt imho von einer sehr realitätsfernen Einschätzung der Machtverhältnisse und Interessensgegensätze im Arbeitsleben. Wenn man sie läßt, können die Chefs noch ganz anders. Die Begleitumstände beim Gebäudeeinsturz in Sabhar oder bei dem Brand bei Tazreen Fashion nahe Dhaka sind ja sicher noch in Erinnerung. Nicht? "Vier Tage nach dem Großbrand mit 112 Todesopfern in einer Textilfabrik in Bangladesch hat die Polizei drei Manager des Unternehmens festgenommen. Den Männern werde vorgeworfen, eine Evakuierung des Gebäudes verhindert zu haben, erklärte der Polizeichef von Dhaka, Habibur Rahman, der Nachrichtenagentur AFP. "Überlebende sagten uns, sie hätten Arbeitern nicht erlaubt, vor dem Feuer zu fliehen und hätten von einer Routinefeuerübung gesprochen", sagte Rahman. Laut Zeugen sollen die drei Führungskräfte Vorhängeschlösser an den Ausgängen angebracht haben, sodass kaum jemand den Flammen entkommen konnte." --84.191.136.149 18:09, 18. Jun. 2013 (CEST)
Na, wenn das mal nicht ein angemessenes Beispiel zur Frage ist - dann weiß ich auch nicht mehr... --Eike (Diskussion) 18:25, 18. Jun. 2013 (CEST)
:-) Ich schreibe ja nur: "Wenn man sie läßt, können die Chefs noch ganz anders." Das sind drei Botschaften... --84.191.136.149 18:29, 18. Jun. 2013 (CEST)
da bisher immer noch nicht einmal klar ist, ob nun eigentlich der Arbeitende oder aber nur der Besucher herusgeworfen wurde - wenn der Besuchende rausgeworfen wird, geht das auch ohne Abmahnung. Und beim Arbeitenden wäre auch noch die Frage nach dem Arbeitsverhältnis zu klären, da gäbe es noch das Stichwort der Probearbeitszeit, in der dann eine Abmahnung nicht nötig ist. Und bei den beschriebenen Temperaturen sollte man wohl meinen, das ein schon länger dort Tätiger sein Getränk schon selbst mitbringt.feba disk 02:51, 19. Jun. 2013 (CEST)

Durchsichtige Handtücher

Dyson Airblade

Gibt es sowas? Oder irgendwas anderes durchsichtiges, was man naß machen kann und das dann an der Luft gut trocknet. -- 141.30.146.65 17:10, 17. Jun. 2013 (CEST)

Durchsichtig?? Es gibt Mikrofaser-Handtuecher in allen Groessen bei Sport/Outdoorgeschaeften. Diese sind sehr leicht, extrem aufnahmefaehig und schnelltrocknend. --sitic (Diskussion) 17:35, 17. Jun. 2013 (CEST)
Tuchersatz, nämlich heiße Luft, Marktführer dürfte Dyson sein. --80.226.24.14 17:45, 17. Jun. 2013 (CEST)
Da kann man aber nicht alle Körperteile reinstecken. Schon bei den Füßen wird es arg artistisch. --Rôtkæppchen68 17:48, 17. Jun. 2013 (CEST)
Und zwischen den Füßen erst! ;-) omg --Giftzwerg 88 (Diskussion) 19:27, 17. Jun. 2013 (CEST)

Es geht um Kühlung mittels feuchtem Krempel, der aber nicht das Licht behindern soll... vielleicht könnte man einfach das Zeug nicht ans Fenster hängen... aber dann muss man den Luftaustausch anders verstärken... -- 141.30.146.65 18:04, 17. Jun. 2013 (CEST)

Mach es wie der Ulmer Spatz: Längs statt quer! -- 188.105.132.92 18:14, 17. Jun. 2013 (CEST)
@IP141: Du willst also etwas Nasses vor Fenster oder Tür hängen, damit die verdunstende Feuchtigkeit kühlt? Schreib das doch gleich! Meine Empfehlung: Alte, "abgewaschene", aber weiße (!) Bett-Tücher oder alte (aber weiße!) Gardinen, nicht zu groblöcherig. Die Betonung von "weiß", damit Sonneneinstrahlung zurückgehalten wird und es nicht zu dunkel wird. Abgewaschen, damit es verdunstet und nicht tropft. Handtücher sind zu dick und tropfen. --Bremond (Diskussion) 18:47, 17. Jun. 2013 (CEST)
Dünne Halstücher oder Seidentücher gehen auch gut. --Sr. F (Diskussion) 19:42, 17. Jun. 2013 (CEST)
Oder einfach ein kleiner Wasserfall. Kühlt und erfrischt beim Durchgehen. :-) --Friechtle (Diskussion) 19:39, 17. Jun. 2013 (CEST)
Das wäre eine sehr edle Büroeinrichtung :) -- 141.30.146.65 11:58, 19. Jun. 2013 (CEST)
Idealerweise nimmst Du ein Material, dessen Brechungsindex genau dem von Wasser enspricht. Nassgemacht erscheint das Material dann durchsichtig. Beispielsweise hat Benzin denselben Brechungsindex wie Papier, weswegen Paier durchsichtig wird, wenn Du es mit Benzin oder anderen Kohlenwasserstoffen benetzt. --Rôtkæppchen68 20:20, 17. Jun. 2013 (CEST)
idealerweise nimmt man gleich Wasser = eine Eisplatte, die kühlt gleich, oder man berieselt eine Glasplatte mit einem Wasserfilm. - andy_king50 (Diskussion) 22:05, 18. Jun. 2013 (CEST)
Benzingetränktes Papier vor die Fenster zu hängen empfiehlt sich aber nur in Nichtraucherhaushalten! --Optimum (Diskussion) 18:38, 18. Jun. 2013 (CEST)
…und für Krösusse, wo das Zeugs doch so teuer ist. Wasser hat eine höhere Verdampfungswärme und ist nicht so teuer. Fehlt nur noch ein Vorhangmaterial mit dem richtigen Brechungsindex. --Rôtkæppchen68 20:50, 18. Jun. 2013 (CEST)
Unabhängig von der Praktikabilität ist das wirklich ne spannende Frage :) -- 141.30.146.65 11:58, 19. Jun. 2013 (CEST)

Was ist beim rechtsgelenkten Auto gespiegelt und was gleich?

Sind beim Rechtlenker die Armaturen, Scheibenwischer und/oder die Schaltung gespiegelt oder sind sie gleich, nur dass das Lenkrad auf der anderen Seite ist? 46.115.105.192 19:49, 17. Jun. 2013 (CEST)

Da musst du teuflisch aufpassen. Als ich das erste Mal auf Malta mit einem Leihwagen bergauf, ca 15 % Steigung, im 1. Gang mit Vollgas anfahren wollte, war das Geräusch unerträglich. Es war der dritte. Zum lück: Leihwagen. -jkb- 19:52, 17. Jun. 2013 (CEST)
Blinker und Scheibenwischer sind auch vertauscht. Das ist lustig. Wenn es regnet, da blinkst du :-)--Phlixx (Diskussion) 20:02, 17. Jun. 2013 (CEST)
Es ist klassischerweise praktisch alles vertauscht bis auf den Blinker. --Mamicale (Diskussion) 20:03, 17. Jun. 2013 (CEST)
Eben NICHT! Die Gänge sind gleich. Die niedrigen (hier: am Körper) links, die hohen (hier: weg vom Körper) rechts, und da schaltet man falsch, wenn man eben so assoziiert (am Körper - weg vom Körper). -jkb- 20:05, 17. Jun. 2013 (CEST)
Und die Pedale? Links Gas geben, rechts bremsen? --212.183.102.45 20:12, 17. Jun. 2013 (CEST)
Gleich, wobei beim Schaltwagen auch ganz links die Kupplung ist. Nur Achtung: rechts vor links führt zu Problemen! -jkb- 20:23, 17. Jun. 2013 (CEST)
Wieso führt das bei rechts-vor-links zu Problemen, wenn man rechts sitzt und Rechtsfahren gewöhnt ist? Es müsste doch nur beim Überholen stören oder wenn man Rechtsverkehr nicht gewöhnt ist.`46.115.106.231 21:18, 17. Jun. 2013 (CEST)
Eine berühmte Ausnahme ist der Austin Seven (original ein Rechtslenker), dessen Linkslenkerversion Dixi (Automarke) komplett bis zur letzten Schraube gespiegelt war. --Rôtkæppchen68 20:29, 17. Jun. 2013 (CEST)
Ursprünglich als Rechtslenker konzipierte Autos haben meist den Tank links und den Auspuff rechts. Das wird dann auch in der Linkslenker-Variante üblicherweise so beibehalten. -- Aspiriniks (Diskussion) 20:38, 17. Jun. 2013 (CEST)

Sind die beiden Autotypen also Enantiomere oder doch Diastereomere, und wo ist das Chiralitätszentrum --Eu-151 (Diskussion) 21:13, 17. Jun. 2013 (CEST)

So einfach ist das gar nicht. Bei LKWs unterscheiden sich sogar die Trittstufen des Führerhauses bei Rechts- und Linkslenker. --Rôtkæppchen68 21:21, 17. Jun. 2013 (CEST)
Touristen aus linkslenkenden Ländern, die mit rechtsgesteuerten Autos unterwegs sind, erkennt man zuverlässig daran, dass sie an Engstellen den Scheibenwischer betätigen (Sie wollen eigentlich per Lichthupe den Gegenverkehr zum Durchfahren auffordern...). -- Geaster (Diskussion) 21:26, 17. Jun. 2013 (CEST)
@Rôtkæppchen68: bis zur letzten Schraube oder einschließlich der letzten Schraube? (nicht signierter Beitrag von 212.183.102.45 (Diskussion) 22:30, 17. Jun. 2013 (CEST))
Die Gewinde würde ich dann doch rechts lassen, denn Links- und Rechtsschrauben passen beide in dieselbe Öffnung. --Rôtkæppchen68 22:42, 17. Jun. 2013 (CEST)
Rechtsgelenkte Wagen sind auf dem Gebrauchtwagenmarkt günstiger zu bekommen bzw. für den selben Preis jüngeren Erstzulassungsdatums. Bei den recht billigen Importen aus UK kann man allerdings mit einer höheren Durchrostungswahrscheinlichkeit bei gleichem Alter rechnen. 46.115.106.231 22:51, 17. Jun. 2013 (CEST)
Samoa hat wegen der billigen australischen gebrauchten Rechtslenker am 7. September 2009 6 Uhr auf Linksverkehr umgestellt. --Rôtkæppchen68 00:42, 18. Jun. 2013 (CEST)
Beim Scheibenwischer/Blinker kommts auf die Marke an. In Singapur standen mir 2 Firmenwagen zur Verfügung. Der eine ware ein Japaner (Toyota?), der andere ein britischer Rover. Beim Japaner waren Scheibenwischer/Blinker gespiegelt, also Blinker mit der rechten Hand zu bedienen. Beim Briten wars, als ob man das Lenkrad links rausgeschnitten hätte und rechts eingesetzt. Also der Blinker links am Lenkrad. Tja, ich musste auch erst immer gucken in welchem Wagen ich saß, um den beim Abbiegen nicht die Scheibe zu wischen :-( Bonusfrage, wie ist das mit dem Zündschloss? Ich weiß nicht mehr, ob die auch an unterschiedlichen Stellen (links/rechts vom Lenkrad) waren. -194.138.39.54 08:45, 18. Jun. 2013 (CEST)
Könnte eventuell auch Links- und Rechtsverkehr#Fahrzeugbau hilfreich sein? Dort steht z. B. "Die Anordnung der Hebel zur Bedienung von Blinker und Scheibenwischer wird unterschiedlich umgesetzt. In Fahrzeugen europäischer Hersteller ist der Hebel für Blinker und Fernlicht immer auf der linken Seite des Lenkrades angebracht, in japanischen Modellen hingegen stets auf der fahrzeugäußeren Seite."--IP-Los (Diskussion) 11:37, 18. Jun. 2013 (CEST)
Das ist es. Danke. Sehr informativ und gut geschrieben. Leider teilweise im falschen Artikel. Vielleicht kann hier ein Link helfen. 46.115.82.246 19:39, 18. Jun. 2013 (CEST)

15 Volt aus einem PC Netzteil

Ist es möglich, die 12 und die 3 Volt Schinen eines PC Netzteiles in Reihe schalten, um 15V zu erhalten? Oder mögen das Netzteile gar nicht? --95.112.236.247 23:04, 17. Jun. 2013 (CEST)

Übertrager
Spannungssummierschaltung zur Beschaltung des Schaltspannungsreglers – hier die Widerstände anpassen (probieren)
was erhoffst du dir denn davon? Möchtest du deinen PC schneller aufladen  :) --Macuser10 (Diskussion) 23:07, 17. Jun. 2013 (CEST)
Du kannst zwischen den −12 Volt und den +3,3 Volt problemlos 15,3 Volt abgreifen, solange die versogte Schaltung nicht geerdet ist. Ist sie irgendwo geerdet, schließt Du eine der beiden Versorgungsspannungen kurz. --Rôtkæppchen68 23:20, 17. Jun. 2013 (CEST)
laut diesem Artikel: http://www.rc-raceboats.de/forum/showthread.php?16339-2-Netzteile-in-Reihe-schalten muss man besser sehr vorsichtig sein, manche sagen es geht überhaupt nicht. --Macuser10 (Diskussion) 23:24, 17. Jun. 2013 (CEST)
Zwei Netzteile in Reihe schalten ist etwas anderes. PC-Netzteile sind normalerweise sekundärseitig geerdet. Das geht also nur, wenn eines der Netzteile über einen Trenntrafo oder mit abgeklemmtem Schtzleiter (Vorsicht Unfallgefahr) betrieben wird. Ich würde mir für 15 Volt einen passenden DC/DC-Wandler oder gleich ein 15-Volt-Netzteil holen. --Rôtkæppchen68 23:30, 17. Jun. 2013 (CEST)
Richtig. Ein 15V-Netzteil ist wirklich keine große Investition. Bei den Preisen, zu denen die erhältlich sind, würde ich wirklich nicht erst anfangen zu basteln. --Wicket (Diskussion) 23:38, 17. Jun. 2013 (CEST)
Kommt auf den Strom an! Es mein Gebiet: Schaue mal unter PC-Netzteil, Abschnitte Funktion, Sicherheit und Umbau für... Da ist was Du suchst. Beachte die Sicherheit! Nach dem Ausstecken kann immer noch 400V Spannung in großen Kondensatoren vorhanden sein. Sollte die Info nicht ausreichen, schreib hier noch einmal oder auf meiner Diskussionsseite oder benutze das Artikelfeedback: „Hast Du gefunden was Du gesucht hattest?“. --Hans Haase (Diskussion) 23:39, 17. Jun. 2013 (CEST)
Die -12V sind nur minimal belastbar. PC-Netzteile haben sekundär Zweiwegegleichrichter. In Reihe geht nicht. Die 3,3 werden aus den 5V über einen Phasenverschub erzeugt. Da wollte ich noch ein Bild nachliefern. Die 3,3 V haben einen separaten Gleichrichter, der über induktivitäten auf die Wicklungen der 5V Ausgänge des Hauptübertragers geht. Die Beschaltung weicht in der Praxis von Modell zu Modell ab. Der Eingriff in die Referenzspannung, um aus den 12V 15V zu machen, dürfte hier Zielführend sein, dabei steigen alle Spannung mit an. --Hans Haase (Diskussion) 23:49, 17. Jun. 2013 (CEST)
Vorsicht: 0V (Masse, PE) sind auf dem Gehäuse, Serienschaltung bedeutet Schutzleitertrennung (Lebensgefährlich!) oder sehr Aufwendiger Umbau. 0V=GND=PE --Hans Haase (Diskussion) 23:55, 17. Jun. 2013 (CEST)
die hier fehlenden L5 und L6 (Bildmitte) holen aus den 5V-Wicklungen die 3,3V, sie führen auf einen separaten Gleichrichter (hier D23), klicke auf das Bild, es ist mit Notizen versehen Anmerkung: Es gibt bei diesem Exemplar keinen 7912 sondern dafür die LA7 (am blauen Kabel).
(BK)Die −12 Volt des PC-Netzteils sind mit 0,5 bis 1 Ampere belastbar. Für manche Zwecke reicht das. Ich hab jahrelang eine Siemens Logo 24R zwischen der mit 1 A belastbaren +12-Volt- und der mit 0,2 A belastbaren −12-Volt-Schiene eines kleinen Einbauschaltnetzteils betrieben, bis ich mir letzte Woche endlich für schlappe 6 Euro ein 24-Volt-Steckernetzteil geholt habe. --Rôtkæppchen68 23:58, 17. Jun. 2013 (CEST)
Wieviel A? --Hans Haase (Diskussion) 00:41, 18. Jun. 2013 (CEST)
0,15 A. --Rôtkæppchen68 00:46, 18. Jun. 2013 (CEST)
Ok daher der Preis. --Hans Haase (Diskussion) 00:49, 18. Jun. 2013 (CEST)
Bei 1,8 Watt für die Logo und 0,016 Watt für die angeschlossene Peripherie reicht das auch. --Rôtkæppchen68 01:04, 18. Jun. 2013 (CEST)
Mit einer zweiten Doppeldiode könnten imo die -12V frisiert werden. Linearregler oder Phasenschieber raus, Mindestlast dran, und wenn's nur der Lüfter ist, größere Elkos ca. 2200…4700µF und gut ist es. Wenn Du die 12 V von GND entfernen wolltest, kannste die 6 Litzen vom Übertrager aufdröseln. In den Exemplar sind es 2x parallel 5V und einmal 12V je nochmal gespiegelt für den Zweiwegegleichrichter. Pins: 5A 5A 5B 5B 12A 12B. Die Regelung muss dann neu konstruiert werden, oder die 12V machen was sie wollen, schwanken aber nicht wie bei einem konventionellen Trafo-NT. --Hans Haase (Diskussion) 01:00, 18. Jun. 2013 (CEST)

Was darf ich aus der interessanten Diskussion schließen? Kann ich mein Verbraucher an +12V und -3V verbinden um 15V zu erhalten? --95.112.236.247 02:45, 18. Jun. 2013 (CEST)

Dürfen tust Du das schon, nur können tust Du das nicht. Ein PC-Netzteil hat nur −12 Volt und keine −3,3 Volt. Die bei AT und ATX Version 1.0 vorhandenen −5 Volt wurden in der ATX-Version 2.0 gekappt. Bleiben nur −12 Volt und +3,3 Volt. --Rôtkæppchen68 07:13, 18. Jun. 2013 (CEST)
Die Auskunft ist stehts geöffnet, hier herrscht nur Trollausschluss :-) Die 3,3V werden überweigend aus den Wicklungen der 5 V uber eine/zwei stehende Rinkern-Spule(n) gemacht. -3,3V ist gibt's nicht. Hier ist nur die Frage ob dazu ein zweiter Zweiwegegleichrichter ausreichen würde um die 3,3 + und - noch 3,3V sein zulassen? Hier könntest Du Dir die Pins 1 und 3 der D23 holen und jeweils (2 dioden!) negativ gleichrichten und gegen Masse sieben. Achtung es kann mehr werden als 5V! Kurzschlüsse bauen die Schaltung um, dass Du 7V erhalten könntest, wohl nicht belastbar und baldigen Tot des Gleichrichters. --Hans Haase (Diskussion) 13:32, 18. Jun. 2013 (CEST)
7824 macht 24V und LM317 ist einstellbar (7815 macht 15V)

Schon mal an ein Notebook Netzteil gedacht? Google findet für 15V 3A welche für Toshiba Geräte z.B. dieses. Wer eins im Elektronikschrott findet hat Glück. Die üblichen 18-20V Netzteile müsste man extern runter bringen, was für hohe Stromentnahme aufwändiger ist als wenn man nur 500 mA haben möchte. Für kleine Ströme genügt ein LM7815. --Netpilots -Φ- 13:51, 18. Jun. 2013 (CEST)

Auch ne Idee! Macht ca. 4A, kannst hier einen 7815 mit einem PNP-Transistor strecken, da das NT selbst Kurzschlussfest ist: E auf +20V, B auf 7815 Eingang, C auf 7815 Ausgang und +15V Ausgang. Am Ausgang den C und den RL geschätzt: 10kOhm und 100µF nicht vergessen. --Hans Haase (Diskussion) 14:28, 18. Jun. 2013 (CEST)
Nochmal zur Spannungsreferenz, da da immer außenherum gefragt wird: Hold Dir ein Datenblatt zu LM317, einem programmierbaren Linearspannungsregler. Programmiert wird er durch den Spannungsteiler am Ausgang. Beim PC-Netzteil ist das genau das gleiche nur mit einer anderen Referenzspannung, siehe Datenblatt oder probiere es aus. Diese Referenz ist vorgegeben. Die Widerstände werden von der gewünschten Ausgangsspannung auf die Referenz berechnet und ausgewählt.
Es gibt auch programmierbare Notebook-Ersatznetztreile. Bei denen steckst den passenden Ausgangsstecker unter Beachtung der Polarität (!) auf den Adapter und am Netzteil einen Stecker mit einem Widerstand drin an das Netzteil, der das Netzteil auf die Ausgangsspannung programmiert. Das ist der untere Widerstand im Ausgangsspannungsteiler oder ein parallel geschalteter davon, hier könntest Du mit anderen Widerständen experimentieren, falls die 15V oder was Du brauchst nicht dabei sein sollten. --Hans Haase (Diskussion) 14:28, 18. Jun. 2013 (CEST)
Bevor der Fragesteller da an seinem PC-Netzteil rumlötet und dabei eine mit maximal 0,5 Ampere belastbare Lösung herausbekommt, kauft er besser ein 15-Volt-Steckernetzteil für 8,98 Euro (0,8 Ampere). --Rôtkæppchen68 14:52, 18. Jun. 2013 (CEST)
Wenn löten, denke ich ab 15A und mehr, sonst überlegt man sich das Löten. --Hans Haase (Diskussion) 16:06, 18. Jun. 2013 (CEST) Passender Gleich-riecht-er zur Hand?
Ich seh grad, die 15-Volt-Frage stammt von derselben IP wie die Peltier-Frage oben. Ich nehme an, der Fragesteller will seine Peltierkühlung aus dem PC-Netzteil betreiben. Da reichen die paar Milliamperes aus den −3,3 Volts nicht und der Fragesteller besorgt sich besser ein kräftiges Netzteil mit 15 bis 18 Volt und 130 bis 150 Watt. --Rôtkæppchen68 22:17, 18. Jun. 2013 (CEST) PS: Ich hab oben meine unpassenden Antworten gestrichen. --Rôtkæppchen68 23:46, 18. Jun. 2013 (CEST)
Ich würde da eher die Sinnfrage stellen. Falls das ein Prozessorkühler werden soll, um das Ding zu übertakten würd ich's lassen und lieber ein neues Board+Prozessor+Speicher kaufen, das auch ohne Übertakten schneller ist. Billiger ist das allemal, vor allem wenn man die Stromkosten mit einrechnet. (jedes Watt kostet bei einem Jahr Dauerbetrieb mindestens 2€ !) -- Janka (Diskussion) 23:29, 18. Jun. 2013 (CEST)
Ja, Peltier-Elemente brauchen recht viel Strom, wenn die Leistung nach dem Übertakten ausreicht würde ich aber eher eine leistungsfähigere Luftkühlung kaufen, kostet nicht viel und verbaucht nicht viel Strom (wahrscheinlich etwa gleich viel wie die alte, jedenfalls wahrscheinlich eine einstellige Wattzahl) und neuer Prozesor + Mainboard + Speicher kann leicht ein paar hundert € kosten und je nachdem, wie schnell der alte PC war, kann es auch sein, dass man mit eienr Billig-Kombination um z.B. 100€ nicht viel rausholen kann. Wenn man z.B. 200€ oder 300€ investiert muss man ziemlich viel Watt ziemlich lang einsparen, selbst bei Dauerbetrieb, weil vor allem bei Leuten, die ihren PC im Dauerbetrieb rennen lassen, ist der meist idle, da ist der Unterschied, wenn das System nicht uralt ist (Netburst oder eine noch ältere Architektur) der Verbrauch auch bei älteren Komponenten recht gering. --MrBurns (Diskussion) 23:40, 18. Jun. 2013 (CEST)
Nach Archivierung:

Eine weiter Antwort auf die Frage befindet sich Wikipedia:Auskunft/Archiv/2015/Woche 13#Welche Spannungen kann ich an einem PC Netzteil abgreifen? --Hans Haase (有问题吗) 17:10, 31. Mär. 2015 (CEST)

Sowjetisches Kernkraftwerk

Ich bin auf der Suche nach dem Namen eines sowjetischen Kernkraftwerkes, das keine getrennten Kreisläufe benutzte, sondern Flusswasser direkt in den Reaktor geleitet hat. --188.23.225.155 07:48, 18. Jun. 2013 (CEST)

Dieses hier?
"Die Atomanlage Majak (übersetzt: Leuchtturm) liegt im Bezirk Tscheljabinsk, etwa 2000 Kilometer östlich von Moskau. Am 29. September 1957 explodierte ... Trotzdem fischen und jagen dort Leute. Kinder baden in den Gewässern. Der Gebrauch des Flußwassers ist verboten aber da es keine Alternative gibt wird das Wasser auch weiterhin aus dem Fluß geholt." <= aus dem Web. GEEZER... nil nisi bene 08:43, 18. Jun. 2013 (CEST)
Majak war nicht für die Stromerzeugung gedacht, also kein Kraftwerk, sondern eine rein militärische Anlage für den Bau von Atombomben. Es war das sowjetische Los Alamos. --El bes (Diskussion) 13:45, 18. Jun. 2013 (CEST)
Dann könnte es hier dabei sein. <= Das ist lesbar an einem verregneten Nachmittag. GEEZER... nil nisi bene 14:15, 18. Jun. 2013 (CEST)
da, S.98 findet sich: "Radioactivity in river water has arisen through the direct-flow method of cooling at the plutonium factory at Krasnoyarsk...".--Advanceddeepspacepropeller (Diskussion) 16:47, 18. Jun. 2013 (CEST)
Die Anlage, die in dem von Advanceddeepspacepropeller verlinkten Buch gemeint ist, ist die Kerntechnische Anlage Schelesnogorsk. Aber auch von Majak ist derartiges durchaus bekannt. -- Felix König 19:00, 18. Jun. 2013 (CEST)

Titel eines Hörbuchs

Guten Tag,

ich habe eine Frage an alle Wikipedia-Benutzer:

Vor geraumer Zeit gab auf dem Sender WDR 5 ein Sprecher eine Geschichte zum Besten.

Der Ich-Erzähler betritt ein Haus, das auf sehr eigentümliche und unheimliche Weise von ihm beschrieben wird und dessen Besitzer ihm gegenüber noch nicht in Erscheinung getreten ist.

Leider konnte ich den Beitrag nicht zu Ende hören und nun quält mich meine Unwissenheit.

Kann mir eventuell jemand weiterhelfen?

Vielen Dank im Voraus!

--93.184.128.34 11:05, 18. Jun. 2013 (CEST)

Du brauchst mehr Elemente: Etwa um welche Tageszeit hast du das gehört? An welchem Wochentag? War es ein Hörspiel oder "Karl-Heinz erzählt einen Schwank aus dem Leben"? Wie lange dauerte das, was du gehört hast. etc. etc. Du verdichtest die dir vorliegenden Informationen, bis ein konkretes Ziel entsteht. :-) GEEZER... nil nisi bene 11:21, 18. Jun. 2013 (CEST)
... wie lang war die "geraume Zeit" ungefähr (Monate? Jahrzehnte?), was war so eigentümlich? --Eike (Diskussion) 11:30, 18. Jun. 2013 (CEST)

War das vielleicht das preisgekrönte WDR-Hörspiel "Das Haus" nach dem Roman House of Leaves? --FA2010 (Diskussion) 12:19, 18. Jun. 2013 (CEST)

Zunächst einmal herzlichen Dank an alle für die schnelle Bearbeitung meiner Anfrage.

Woran ich mich noch teilweise erinnern kann:

Die Sendung wurde zur Mittagszeit auf besagtem Sender ausgestrahlt. Seitdem dürften mittlerweile einige Monate ins Land gegangen sein. In dem Zeitraum, den ich mit verfolgte, handelte es sich ausschließlich um einen jungen, männlichen Erzähler. Andere Sprecher kamen nicht zum Einsatz. Daher vermute ich, dass es sich um ein Hörbuch handelte; kein Hörspiel. Was nun die eigentümliche Beschreibung des Hauses betrifft, kann ich leider nicht mit weiteren Informationen dienen. Die Art und Weise wie das Haus samt Inventar und seinem noch unbekannten Bewohner beschrieben wurde, hatte einfach etwas Unheimliches an sich.

Mehr Informationen habe ich leider nicht.

--93.184.128.34 13:03, 18. Jun. 2013 (CEST)

In ihren Podcasts (ich höre nur die politischen, kann zu Deiner Beschreibung daher nix sagen) werben die WDR5er regelmäßig für ihr Hörertelefon, auf deren Website versteckt sich vermutlich auch irgenwo eine Emailkontaktadresse. Vielleicht kann man Dir dort weiterhelfen. feba disk 02:56, 19. Jun. 2013 (CEST)

Die Literatursendungen bei WDR 5 sind eher abends (Ohrclip). Es ist lange her ("einige Monate"). Und die Beschreibung ist doch recht unspezifisch, falls nicht Der Untergang des Hauses Usher gemeint ist. --Bremond (Diskussion) 14:39, 19. Jun. 2013 (CEST)

Pseudoprimzahl - welche enthalten alle normalen Primzahlen?

Auf die Frage gibt der Artikel keine Antwort. Mein Ziel ist einen Algorithmus zu haben, der wie "immer +2 ab der Zahl 3" alle Primzahlen enthält, aber deutlich weniger falsche. -- 141.30.146.65 14:26, 18. Jun. 2013 (CEST)

Du könntest Dein Inkrement so wählen, dass Du vorab die Vielfachen kleiner Primzahlen aussortierst, z.B. anfangen mit 2, 3, 5 und dann immer abwechseln 2 und 4 addieren: Damit sind Deine Kandidaten 2, 3, 5, 7, 11, 13, 17, 19, 23, 25, 29, ... und Du hast das Kandidatenfeld schon einmal um ein Drittel verkleinert. In weiteren Schritten filterst Du vorab die Vielfachen von 5, 7, 11, usw, musst die gefilterten Zahlen dann aber am Anfang gesondert berücksichtigen, wie in Deinem Fall die 2. --Rôtkæppchen68 15:59, 18. Jun. 2013 (CEST)
Schon gefunden? Sieb des Eratosthenes. --Optimum (Diskussion) 18:30, 18. Jun. 2013 (CEST)
SdE erfordert einen Speicher für das Sieb. Wer ohne den Speicher auskommen will (z.B. für eine Reihe sehr großer Zahlen), kann z.B. für jedes x ausrechnen ob die Zahl 2x–1 – 1 ein Vielfaches von x ist. Das gilt bei jeder Primzahl und bei relativ wenigen Nichtprimzahlen. Das geht schneller, wenn man in den Zwischenergebnissen jeweils die Vielfachen von x aussortiert.
Ein Beispiel: x = 17. Statt 216 – 1 direkt auszurechnen, kann man 28 = 256 ausrechnen und den Rest 1 behalten (256 = 15*17 + 1). Dann gilt 216 = 12 = 1 (bis auf Vielfache von 17), und 217-1 – 1 = 1 – 1 + (irgendein Vielfaches von 17). Das ist langsamer als Rotkäppchens Methode, aber bei großen Zahlen recht zuverlässig (wenige Irrtümer). (nicht signierter Beitrag von One.Ouch.Zero (Diskussion | Beiträge) 08:46, 19. Jun. 2013 (CEST))
Mist, wieder 'ne Signatur vergeigt....
Aber Achtung, nicht alle Pseudoprimeln sind auch prim... (siehe auch hier, http://thedailywtf.com/Articles/The-Prime-Candidate.aspx) One.Ouch.Zero (Diskussion) 12:30, 19. Jun. 2013 (CEST)

Falscher Eintrag bei Wikipedia

Nikolaus Schneider - gesund und munter

Sehr geehrte Redaktion von Wikipedia ! Unter den Verstorbenen, die auf dem Waldfriedhof München beigesetzt wurden, ist " Nikolaus Schneider " verzeichnet. Dabei handelt es sich um den ehemaligen Präses der Ev. Kirche im Rheinland und ehemaligen Ratsvorsitzenden der Ev. Kirche in Deutschland. Dieser Nikolaus Schneider erfreut sich bester Gesundheit und ist noch nicht verstorben !!!!! Bitte ändern Sie den Eintrag unter Waldfriedhof München. Mit freundlichen Grüßen Heinz-Herbert Grün (nicht signierter Beitrag von 188.97.125.167 (Diskussion) 15:55, 18. Jun. 2013 (CEST))

Es scheint sich um Liste von Begräbnisstätten bekannter Persönlichkeiten zu handeln. Bei den Artikeln zum Waldfriedhof und zu den dort bestatteten Personen wird kein Schneider erwähnt. Bei einem derart häufigen Namen könnte natürlich auch eine andere Person gemeint sein, aber es wird nicht klar, welche, also nehm ich das einfach raus. Das hättest Du übrigens auch machen können, hier gibt es keine Redaktion, einfach "Seite bearbeiten" klicken und verbessern. --FA2010 (Diskussion) 16:01, 18. Jun. 2013 (CEST)
Der falsche Name wurde in der Zwischenzeit entfernt.
Nikolaus_Schneider_(Begriffsklärung) listet auch keinen bereits verstorbenen Nikolaus Schneider auf, der hier gemeint gewesen sein könte. Gibt es vll. trotzdem einen Namensvetter, der auf dem Waldfriedhof München beigesetzt ist? --88.130.69.74 16:32, 18. Jun. 2013 (CEST)
Der Eintrag war hier dazugekommen. --Eike (Diskussion) 16:27, 18. Jun. 2013 (CEST)
Interessant. Das sind ja allesamt keine "bekannten Persönlichkeiten". Wenigstens die roten Links gehören m. E. alle raus in dieser Liste. Ist inzwischen wohl auch geschehen. Aber was da noch so an Fehlern (wg. Namensgleichheit) drinsteckt, ist schwer zu sagen. --FA2010 (Diskussion) 16:53, 18. Jun. 2013 (CEST)

Militärischer Rang und akademischer Titel

Hallo, wenn ich an jemanden einen förmlichen Brief schreibe, der Offizier und Akademiker ist, schreibe ich dann korrekterweise an Herrn Dr. Oberst XY oder an Herrn Oberst Dr. XY? Danke! --178.82.46.72 16:58, 18. Jun. 2013 (CEST)

Schau mal dort Der akademische ist immer das Letzte vor dem Namen --RobTorgel (Diskussion) 17:08, 18. Jun. 2013 (CEST)
Zu dem Ergebnis hätte auch ein simpler Vergleich der Google-Ergebnisse zu den Abfragen "Herr(n) Dr. Oberst" und "Herr(n) Oberst Dr." geführt - Letztere sind klar in der Überzahl, obwohl Erstere auch etliche Doktoren namens Oberst mitnehmen (und dafür anders als die zweite Gruppe in aller Regel auch nicht aus Bundeswehr- oder sonstigen "offiziellen" Kreisen stammen). --YMS (Diskussion) 20:51, 18. Jun. 2013 (CEST)
Kommt auf den Zusammenhanng an weil die formale Anrede sich im Militärischen natürlich nur auf den entsprechenden militärischen Rang reduziert - das Dr. also weggelassen wird. Im zivilen Zusammenhang wird dagegen üblicherweise der Rang Oberst weggelassen weil es kein Titel sondern eben ein militärischer Rang ist. --Kharon 08:44, 19. Jun. 2013 (CEST)

CO2 Steuer? Abgabe?

gemäß diesem Bericht: http://www.auto-motor-und-sport.de/news/co2-steuer-grosse-autos-im-ausland-extrem-teuer-701341.html

erhebt Frankreich unter anderen Ländern Strafaufschläge für Autos, die viel co2 ausstoßen. Ich konnte per google nichts darüber finden was Zahlen und FAkten anbelangt. Mich interessiert insbesondere die Situation in Frankreich und in GB. Hat mir jemand einen Link, in dem man nachlesen kann, was für Aufschläge fällig werden? --77.3.174.85 20:02, 18. Jun. 2013 (CEST)

In Österreich gibt es etwas ähnliches (wird im Artikel auch erwähnt), das nennt sich Normverbrauchsabgabe. So teuer wie in Norwegen ist es aber nicht. --El bes (Diskussion) 21:47, 18. Jun. 2013 (CEST)
Bis 1985 war die KFZ-Steuer für alle Fahrzeuge gleich 14,40 DM (= 7.36 €) pro angefangene 100ccm Hubraum. Heute kostet ein "Euro 3" genormter Benziner etwa gleich viel (6,75 € + Zuschläge). Die KFZ-Steuer für ein nicht schadstoffarmes Diesel-Fahrzeug liegt nun bei 37,58. Das sind über 500 % Erhöhung. CO2 wird schon lange besteuert, nur nicht exakt auf die tatsächlichen Emmissionen abgestimmt. --Kharon 00:03, 19. Jun. 2013 (CEST)
Sind die Zahlen pro Monat oder pro Jahr. Falls ersteres stimmt, kannst du in Österreich einen Nuller hinter dran hängen. --El bes (Diskussion) 00:40, 19. Jun. 2013 (CEST)
In Deutschland gelten die pro Jahr und angefangene 100 ccm Hubraum Kraftfahrzeugsteuer_(Deutschland). In Österreich nach Kraftfahrzeugsteuer (Österreich) pro Monat und angefangene Tonne höchstes zulässiges Gesamtgewicht. Bei einem Mittelklassefahrzeug mit z.B. 1500 ccm Hubraum und 1,5 Tonnen zulässigem Gesamtgewicht kommt das dann wohl auf was ähnliches raus. --Kharon 08:37, 19. Jun. 2013 (CEST)
Mit dem neueren BJ ((also auf Grundlage CO2 Besteuerung) kommt so ein gewisser Japaner mit verbrennungselektischem Antrieb und ebenfalls 1,5 l Motor in D auf ca. 60,-- KFZ-Steuer/Jahr. --Hans Haase (Diskussion) 15:17, 19. Jun. 2013 (CEST)

Tabelle über Widerstandsmomente bei Holzbalken

--88.117.19.170 20:19, 18. Jun. 2013 (CEST) --88.117.19.170 20:19, 18. Jun. 2013 (CEST)Wikipedia kann eine solche Tabelle nicht finden. ich benötige lediglich die Widerstandsmomente (cm³) von einigen Holzbalken zur statischen Berechnung.--88.117.19.170 20:19, 18. Jun. 2013 (CEST)

In den Artikeln Flächenträgheitsmoment und Widerstandsmoment findest Du Formeln zur Berechnung. --Rôtkæppchen68 20:39, 18. Jun. 2013 (CEST)
Hast Du mal bei informationsdienst-holz.de geschaut? Die haben da zum Beispiel solche Veröffentlichungen mit reichlich Daten zu Konstruktionsholz (die ich, zugegeben, allesamt nicht verstehe). --Rudolph Buch (Diskussion) 10:18, 19. Jun. 2013 (CEST)

gebrochene Unifrakturschrift

--146.52.198.167 21:35, 18. Jun. 2013 (CEST) Ich bin 80 Jahre und habe mich erst seit einem Jahr mit dem Internet befasst. Insofern bin ich ein absoluter Laie. Alles, was ich inzwischen gelernt habe, habe ich mir durch stetes Üben selber beigebracht. Mit den Fachausdrücken kann ich nicht´s mehr anfangen und die technischen Begriffe sind für mich auch nicht mehr erlernbar. So begnüge ich mich nur mit dem Wesentlichen,das ich auch verwenden kann. Da meine Kinder und Enkel allesamt ca. 500km von mir entfernt wohnen,bin ich auf das Schreiben von Karten (zu unterschiedlichen Jubiläen) angewiesen und fertige mir meine Glückwunschkarten selber an. Da ich eine ausgesprochene Liebhaberin der alten deutschen Zierschrift bin und diese auch zu gerne für meine Karten verwende, würde ich zu gerne die gebrochene Unifrakturschrift von Rudolf Koch verwenden. So gefällt mir auch die Schrift "der Frühling". Ich würde sie gerne auf meinen Laptop downloaden, weß aber leider nicht wie ich da vorgehen kann. Deshalb meine Frage an Sie:"Können Sie mir diesbezüglich behillich sein?" Ich würde mich riesig feuen. Mit freundlichen Grüßen--146.52.198.167 21:35, 18. Jun. 2013 (CEST)--146.52.198.167 21:35, 18. Jun. 2013 (CEST) Gerda Krumpel

Haben Sie eine Webseite gefunden, bei der man diese Schriften downloaden kann (und darf)? Dann würde uns die Webadresse beim Helfen helfen...
Außerdem müssten wir wissen, welche Windows-Version Sie verwenden. Die steht glaube ich (ich verwende gerade kein Windows) meist senkrecht am linken Rand, wenn man auf den Start-Knopf drückt.
--Eike (Diskussion) 21:50, 18. Jun. 2013 (CEST)
(BK)Unter http://unifraktur.sourceforge.net/ gibt es diese Schrift zum runterladen. Dazu auf UnifrakturMaguntia.2012-10-19.ttfautohint.zip klicken. Das Ziparchiv runterladen und im Windows-Explorer öffnen. Darin den Ordner UnifrakturMaguntia.2012-10-19.ttfautohint doppelklicken. Darin findet sich die Datei UnifrakturMaguntia.ttf. Diese doppelklicken. Freuen. --Rôtkæppchen68 21:54, 18. Jun. 2013 (CEST)
UnifrakturCook kann hier runtergeladen werden. Installation sinngemäß. --Rôtkæppchen68 22:02, 18. Jun. 2013 (CEST)


Ich werfe noch [3] in die Runde, wo solche Fragen gut aufgehoben sind. -- Janka (Diskussion) 22:03, 18. Jun. 2013 (CEST)
Die gesuchte Schrift Frühling von Koch gibt es hier für etwas Geld; die Koch-Fraktur (wie von Rotkaeppchen68 schon angemerkt) als „UnifrakturCook“ für umsonst hier. PDD 22:10, 18. Jun. 2013 (CEST)

Frageportale

Es gibt zwei große Frageportale in deutsch. Gutefrage.net und Yahoo Answers. Gibt es eigentlich noch andere mit einer großen Anzahl an Usern? --85.181.220.119 22:43, 18. Jun. 2013 (CEST)

Siehe auch: Kategorie:Digitaler_Auskunftsdienst und Digitale Auskunft#Digitale_Auskunftsdienste. --88.152.193.150 08:24, 19. Jun. 2013 (CEST)

LVDS und eDP bei notebook monitoren, was ist der unterschied?

Hallo erst mal. Ich hab da mal eine frage, was ist der unterschied zwischen LVDS und eDP bei Notebook monitorem? LVDS gibt es in der wiki, eDP (gibt es nicht in der wiki als monitor anschluss) Gruss-- Conan (Nachricht an mich? Bitte hier lang.) 23:11, 18. Jun. 2013 (CEST)

Siehe en:DisplayPort#eDP. --MrBurns (Diskussion) 23:29, 18. Jun. 2013 (CEST)
LVDS ist der Anschluss für den internen Monitor, hat jeder Laptop, ist nicht rausgeführt, brauchst du dir aleso keine Gedanken zu zu machen. eDP ist eine Displayport-Schnittstelle für einen zweiten Monitor. -- Janka (Diskussion) 23:36, 18. Jun. 2013 (CEST)
mein 3d monitor ist per eDP am mainboard verbunden, mein altes notebook hatte die display verbindung LVDS.-- Conan (Nachricht an mich? Bitte hier lang.) 00:11, 19. Jun. 2013 (CEST)
Falls du die Erklärung unter en:DisplayPort#eDP nicht verstehst: eDP ist praktisch die Implementierung von DisplayPort zum Anschluss von internen Monitoren, z.B. in Laptops und ersetzt LVDS, was früher der Standard war zum Anschluzss von internen Monitoren. Alle Unterschiede aufzuzählen übersteigt wohl den Rahmen der Auskunft, einige Unterschiede werden unter en:DisplayPort#eDP erklärt, um alle Unterschiede zu wissen hilft es wohl nur, die beiden Spezifikationen zu lesen, falls die überhaupt allgemein zugänglich sind. --MrBurns (Diskussion) 00:24, 19. Jun. 2013 (CEST)

Welcher ist der höchste Berg im Umkreis von 50 Km um Tübingen?

frage sihe oben. --95.112.185.147 00:42, 19. Jun. 2013 (CEST)

um Tübingen gibt es keine Berge, nur Hügel. Baden-Württemberg hat überhaupt gar keinen Anteil von den Alpen, sondern nur relativ flache Hügel die zum Mittelgebirge gehören. Der höchste Hügel in Baden-Württemberg ist der Feldhügel (Hügel im Schwarzwald) mit 1493 m. Wie weit der von Tübingen weg ist, kannst du selber messen, mit Google Earth. --El bes (Diskussion) 00:53, 19. Jun. 2013 (CEST)
Es ist der Lemberg (Schwäbische Alb), der 46 Kilometer von Tübingen weg ist. --Rôtkæppchen68 01:00, 19. Jun. 2013 (CEST)
Berg ist ein mehrdeutiger Begriff. Im Elbe-Weser-Dreieck ist jeder Huckel höher als 3 m ein Berg. —PοωερZDiskussion 01:01, 19. Jun. 2013 (CEST)
Und genau einer ist der höchste. Und solange die Stadt Tübingen gemeint ist und nicht verlegt wird, steht er dem Elbe-Weser-Dreieck fern. Chantk (Diskussion) 01:08, 19. Jun. 2013 (CEST)
Das war nur ein Beispiel für eine flache Region. —PοωερZDiskussion 01:09, 19. Jun. 2013 (CEST)
Ja, dann hatte ich die Frage wohl falsch verstanden, sorry. Chantk (Diskussion) 02:29, 19. Jun. 2013 (CEST)
BW hat sehr wohl Anteil an den Alpen, siehe Adelegg als nördlicher Alpenausläufer. Und was genau (in Metern) ein Hügel ist und was ein Berg, darüber schweigen sich z. B. die Wikipedia-Artikel jeweils sehr bewusst aus, weil es eben nicht ganz klar ist und Definitionssache. --FA2010 (Diskussion) 09:13, 19. Jun. 2013 (CEST)
In Indien gelten 3000 Meter noch als Hügel. -- southpark 12:51, 19. Jun. 2013 (CEST)
Der liegt aber in Myanmar. ----Mauerquadrant (Diskussion) 13:44, 19. Jun. 2013 (CEST)
Tatsächlich. Hätte ich sogar lesen können. Wobei es in Indien meiner Erinnerung nach noch mehr solche Hügel gibt, und ich sie nur gerade nicht finde. -- southpark 15:25, 19. Jun. 2013 (CEST)

Knickenkuhle, die zweite

Nachdem ich hier den entscheidenden Hinweis zum Auffinden dieses Loches gefunden habe, eine weitere Frage: Wie genau sind die offiziellen Karten der Ämter im Internet?

Wem kann man noch trauen? Gruss --Nightflyer (Diskussion) 00:46, 19. Jun. 2013 (CEST)

Meiner Ansicht nach ist das Loch des Bundesamts für Naturschutz auch in der Digitaleratlasnord-Karte, an derselben Stelle. Das tiefe Loch auf der Digitaleratlasnord-Karte, das ich für die Knickenkuhle halte, fehlt auf der Bundesamt-für-Naturschutz-Karte, der augenscheinlich eine topographische Karte größeren Maßstabs zugrundeliegt. --Rôtkæppchen68 01:27, 19. Jun. 2013 (CEST)
Ja, das ist wohl so. Die Karte des BFN ist auch im Digitalen Atlas Nord enthalten. Einfach links unter "Karteninhalt" die topographischen Karten auswählen den Maßstab 1:10001 eintippen. Geht man dann auf 1:10000, kommt die andere Karte und man kann gut vergleichen. --тнояsтеn 11:05, 19. Jun. 2013 (CEST)

Herkunft Flagge/Wappen gesucht

Ich suche die Herkunft (also welche es überhaupt ist - habe hier in der Wikipedia nichts gefunden): (hochkant gesehen): Rot links und weiß rechts und in der Mitte zwei Löwen, die sich gegenüber stehen... Welche Flagge/Wappen kann das sein?! --93.131.78.242 14:19, 19. Jun. 2013 (CEST)

In welchem Zusammenhang ist Dir das begegnet? Region? Zeit? Und ob es ein Wappen oder eine Flagge ist, das wenigstens solltest Du ja schon sagen können. --FA2010 (Diskussion) 14:30, 19. Jun. 2013 (CEST)

Deine Beschreibung ist auch nicht sehr anschaulich. Was heißt "in der Mitte"? Eher so wie File:Wappen Unterroth.svg, nur dass die Löwen sich anschauen? Oder Rot und weiß gespalten und in der Mitte dann nochmal ein weiterer andersfarbiger Bereich ("Herzschild"), in dem die Löwen dargestellt sind? Welche Farbe haben die Löwen? Am einfachsten wäre es, Du lädst das, was Du hast, irgendwo hoch (bei www.bilder-hochladen.tv oder ähnlichem) und verlinkst es mal hier. --14:35, 19. Jun. 2013 (CEST)

Am besten einfach mal die Commons-Kategorie "Flags" durchsuchen. --LimboDancer (Diskussion) 14:37, 19. Jun. 2013 (CEST)
"Einfach mal" ist gut. Das sind tausende. In commons:Category:Flags with lions ist das gesuchte jedenfalls schon mal nicht drin, falls es denn eine Flagge ist und nicht nur irgendein Logo. Daher: Kontext. --FA2010 (Diskussion) 14:50, 19. Jun. 2013 (CEST)
Am ehesten sonst noch File:Banner Witten.svg --FA2010 (Diskussion) 14:53, 19. Jun. 2013 (CEST)

Macht es Sinn mehrere Peltier Elemente übereinander zu legen, um damit eine tiefere Temperatur zu erreichen?

Oder verbietet sich das aufgrund der geringen Effizienz der Elemente? --95.112.236.247 22:24, 17. Jun. 2013 (CEST)

Du musst dabei die äußeren Peltierelemente entsprechend kräftiger dimensionieren, da sie ja auch die Stromwärme der inneren Peltierelemente mit abtransportieren müssen. --Rôtkæppchen68 22:39, 17. Jun. 2013 (CEST)
So was ähnliches dachte ich mir. Das heißt, habe ich lauter gleichartige Elemente hier, ist es kontraproduktiv die zu stapeln. Ich müsste also ausrechnen was die jeweilige Verlustleistung des Elementes ist und das nächste Element dann derart dimensionieren, dass es zusätzlich zur relativ geringen efffektiven Wärmemenge die ganze Verlustwärme des darüberliegenden Elementes abpumpen kann? --95.112.236.247 22:52, 17. Jun. 2013 (CEST)
Ungefähr so. Ich würde mir auf jeden Fall das Datenblatt der Peltierelemente anschauen. Evtl bringt es eher etwas, mehrere Peltierelemente parallel anstatt in Reihe zu verwenden. --Rôtkæppchen68 23:22, 17. Jun. 2013 (CEST)
Kannst du aus diesem[4] Datasheet eine Verlustleistung ablesen? (nicht signierter Beitrag von 95.112.236.247 (Diskussion) 23:32, 17. Jun. 2013 (CEST))
Aus den Daten geht hervor, dass bei außen 25 °C 71 Watt Kühlleistung bei 130,9 Watt elektrischer Leistung übertragen werden. Das Peltierelement heizt die Umgebung also mit 201,9 Watt. Um diese Wärme über gleiche Peltierelemente abzuführen, brauchst Du schon drei Stück davon. Bei 50°C Außentemperatur sieht es ähnlich aus: Aus 147 Watt elektrischer Leistung und 79 Watt Kühlleistung werden 226 Watt Heizleistung. --Rôtkæppchen68 15:21, 18. Jun. 2013 (CEST)
Wenn Du zwei gleiche Peltierelemente stapelst, muss das Innere auf ca. 40 Prozent seiner Leistung gedrosselt werden, um das äußere nicht thermisch zu überlasten. Damit hast Du ungefähr die doppelte Temperaturdifferenz bei 40 Prozent der Kühlleistung eines einzelnen Peltierelements erreicht. --Rôtkæppchen68 15:43, 18. Jun. 2013 (CEST)
Das gilt aber nur, wenn die "heiße" Seite auch 25°C (bzw. 50°C) hat! Die übertragbare Wärme sinkt linear mit der Temperaturdifferenz, bei den von dir angenommenen Werten ist sie bei 70°C Temperaturdifferenz Null. Höher kann dieses Element die Wärme nicht "pumpen". -- Janka (Diskussion) 23:14, 18. Jun. 2013 (CEST)
Bevor man sowas andenkt, muss man zuerst die Wärmeübertragung auf das Element und weg davon durchoptimiert haben. Das ist nämlich der Faktor, der die erreichbare Temperaturdifferenz am ehesten begrenzt. Also nicht nur den Rückkühler anblasen, sondern auch auf der kalten Seite für eine große Oberfläche des Kühlers sorgen und - am wichtigsten - das zu kühlende Material umwälzen. -- Janka (Diskussion) 00:45, 18. Jun. 2013 (CEST)
Was heißt umwälzen? --95.112.236.247 02:46, 18. Jun. 2013 (CEST)
wikt:Umwälzung[2]. --Rôtkæppchen68 09:32, 18. Jun. 2013 (CEST)
Kaskadieren von Peltier-Elementen ist durchaus üblich. Ich kann mich an ein paar Peltier-Kühler mit Kälteklasse N erinnern. Das funktioniert nur mit kaskadierten Elementen. Yotwen (Diskussion) 21:06, 18. Jun. 2013 (CEST)
Ja, wenn man an die Temperaturgrenze eines einzelnen Elementes kommt geht es halt nicht anders. Wegen des extrem geringen Wirkungsgrades kaskadierter Elemente sollte man aber schon vorher alles versuchen, um die Wärme möglichst effektiv aus den Kühlgut rauszubekommen (also z.B. die Kalte Seite besser isolieren oder die Masse des Kühlgutes verkleinern). Mit kaskadierten Elementen muss man das nämlich dann erst recht tun. -- Janka (Diskussion) 23:22, 18. Jun. 2013 (CEST)
Ich nehme an, der Fragesteller will seinen Rechner mit den Peltierelementen kühlen, da die Frage nach den #15 Volt aus einem PC Netzteil von derselben IP-Adresse stammt. In so einem Fall würde ich echt empfehlen, mehrere Peltierelemente parallelzuschalten, um die Verlustwärme der CPU möglichst optimal abzuführen. Ordentliche Desktop-CPUs haben mindestens 70 Watt Verlustleistung, womit das vom Fragesteller gewählte Peltierelement an seine Leistungsgrenze kommt. Zwei Stück kaskadieren mit 30 Watt Kühlleistung funktioniert dann nur bei runtergetakteten Notebook-CPUs. Und ich denke, die Intention des Fragestellers ist, möglichst viel aus der CPU rauszuholen und nicht, die CPU-Kühlung mutwillig zum Flaschenhals der Systemperformance zu machen. --Rôtkæppchen68 23:36, 18. Jun. 2013 (CEST)
Technisch löst man das besser, wenn zwischen die Elemente ein Medium gelegt wird. Im Idealfall würde ein Peltieelement ein Kühlmittel kühlen, welches dann die Wärme des Sekundärelements abführt. Dazu benötigt man aber recht komplizierte Schläuche, exotische Flüssigkeiten (sterile Kochsalzlösungen) usw. Daher begnügten sich industrielle Hersteller solcher Lösungen mit einem soliden Block extrudierten Aluminiums zwischen den Elementen und grossen Aluplatten mit oberflächen-vergrössernden Finnen auf der Kalt- und der Warmseite. Yotwen (Diskussion) 13:03, 19. Jun. 2013 (CEST)
Wasserkühlung für PCs gibt es mittlerweile fertig zu kaufen. Wenn das Teil sowieso an Schläuchen hängt, kann man den heißen Wärmetauscher auch in den Kühlschrank tun. Ein Kondensationsproblem hat man aber in jedem Fall, egal ob mit Kühlschrank oder Peltier. Die Kühlung muss geregelt sein, damit sich kein Kondensat bildet, das beim Herabtropfen ganz schnell den Rechner ruinieren kann. Oder man muss den Rechner von der potentiell feuchten Umgebungsluft abkapseln und sämtliche Abwärme über ein Kühlsystem nach außen bringen. --Rôtkæppchen68 15:04, 19. Jun. 2013 (CEST)
Von den Wasserkühlungen, dies wirklich schon fertig zu kaufen gibt (also die man auch ncuht zusammenbauen muss) würde ich eher abraten. Die sind nicht nur teurer als gute Luftkühlungen, sondern auch deutlich schlechter. Anders ists bei den Sets, die zwar alles enthalten, die man aber noch sleber zusammenbauen muss. Die sind teilweise besser in der Kühlleistung als Luftkühlungen, aber wenn man wirjklich eine bessere Küphlleistung als bei guten Tower-Luftkühlern will, noch viel teurer (ich bin nicht ganz auf dem laufenden was die Preise betrifft, aber ich denke für unter 300€ bringt man kaum eine Wasserkühlung, die im Vergelich zur Luftkühlung viel bringt) und rentireen sich daher mMn nur bei High-End-Systemen, deren Preis weit im vierstelligen €-Bereich liegt. Bei einem alten Rechner bekommt man um den geluichen Preis eine viel größere Performancesteigerung hin, indem man einfach CPU + Mainboard + RAM tauscht, oder man bekommt eine etwas kleienre Performancesteigerung mit einem deutlich besseren Preis/Leistungsverhältnis, indem man für ca. 30-80€ die Luftkühlung aufrüstet (wobei normalerweise alles ab 50€ High-End ist,ab ca. 70-80€ bekommt man schon den Phanteks PH-TC14PE, der mMn sowohl von der Kühlleistung her las auch von Kühlleistung/Lautstärke der beste Luftkühler für CPUs ist, wobei man für die maximale Kühlleistung andere Lüfter braucht, weil die mitgelieferten relativ langsam drehen, siehe z.B. hier und hier, wobei beim 2. Test zu beachten ist, dass nur 1 Lüfter verwendet wurde, womit der Vergleich mMn nur zu anderen 2-Lüfter-Kühlern fair ist, nicht zu 1-Lüfter-Kühlern wie dem Thermalright HR-02 Macho, weil ein 2. Lüfter bringt zwar 3d B(A) mehr, aber eine sehr deutliche Steigerung der Kühlleistung). --MrBurns (Diskussion) 15:32, 20. Jun. 2013 (CEST)

Beerenagriffe

Wissen Sie, wie häufig (oder selten) tödliche Beerenangriffe sind?


--82.164.205.69 15:30, 18. Jun. 2013 (CEST)

Beeren oder Bären? --Gereon K. (Diskussion) 15:34, 18. Jun. 2013 (CEST)
Weltweite Zahlen habe ich noch nicht gefunden, aber in Rumänien gibt es durchschnittlich zwei Todesopfer pro Jahr. [5] --Eike (Diskussion) 15:35, 18. Jun. 2013 (CEST)
(BK) Oft werden bei Angriffen auf Beeren die ganzen Beerensträucher getötet. Besonders Meister Petz kann sich da nicht zurückhalten. In USA kommt es häufig vor, dass so ein Grizzly nur die männlichen Beerensträucher angreift (dort Himbeeren genannt), manchmal auch die weiblichen (Herbeeren). Gibt er dabei Geräusche des Wohlbefindens ab, sind es übrigens Brummbeeren. --Aalfons (Diskussion) 15:40, 18. Jun. 2013 (CEST)
That was grizzly hard to bear ... GEEZER... nil nisi bene 16:11, 18. Jun. 2013 (CEST)
(BK) Also Beerenangriffe sind sehr selten, nur vor den tollen Kirschen sollte man sich in Acht nehmen (und auch anderen), die können tödlich sein. --BeverlyHillsCop (Diskussion) 15:44, 18. Jun. 2013 (CEST)
Unter Liste tödlicher Beeren sind noch mehr Angriffsbeeren aufgelistet. --Rôtkæppchen68 15:47, 18. Jun. 2013 (CEST)
Es müssen keine Giftbeeren sein. Auch wenn dich eine Stachelbeere anspringt, kann das schmerzhaft werden. --YMS (Diskussion) 15:50, 18. Jun. 2013 (CEST)
Auch ein Hubschrau-ber ist nicht zu unterschätzen. --RobTorgel (Diskussion) 16:15, 18. Jun. 2013 (CEST)
Wenn dir eine Melone auf den Kopf fällt, kann das evtl. schon weh tun, aber bei Nüssen (z.B. Kokosnuss) gibt es sogar eine Statistik der Todesfälle. --Sr. F (Diskussion) 16:21, 18. Jun. 2013 (CEST)
Stimmt. Bei Beeren müssen natürlich auch Kürbisse, Melonen, Zucchini, Gurken und Vanille mit berücksichtigt werden. --Rôtkæppchen68 16:29, 18. Jun. 2013 (CEST)
Wahrscheinlich werde ich nur ein "Ein bisschen Humor muss sein!" ernten, aber: Muss es sein, dass auf eine ernstgemeinte Frage mit Rechtschreibfehler fünf Leute nur antworten, um auf dem Rechtschreibfehler rumzureiten? Gereon K. hat es zumindest höflich formuliert und sich nicht drüber lustig gemacht. Ein im Sinne der Wikipedia:Auskunft ausgesprochenes "Danke" an Eike für die ernsthafte Antwort.
Die englische Wikipedia hat einen Artikel dazu: en:Bear attack, der ein Paar Zahlen nennt. In Kurzform: etwa zwei Verletzte pro Jahr in den USA bzw. drei Tote pro Jahr in Kanada und USA zusammen. Schwer zu beurteilen, wie solide die Datenbasis dafür ist. Die Quelle hier spricht auch von ca. 2 Toten pro Jahr in Nordamerika. Erscheint im Vergleich mit den Zahlen zu Rumänien erstmal wenig. Aber die ungefähre Größenordnung dürfte damit zu erkennen sein. --::Slomox:: >< 16:27, 18. Jun. 2013 (CEST)
Ein bisschen Humor muss sein! —[ˈjøːˌmaˑ] 16:30, 18. Jun. 2013 (CEST)
Ein bisschen Humor muss sein!
Size matters aber auch:
Hier gab es nämlich einen Angriff der Franzosen, die es aber nicht schafften, damals schon Besatzungsmacht zu werden. Man hat sie nicht durchgelassen bis nach Westberlin, aber die Abwehr kostete über tausend Menschenleben; damit hat es sich wohl um den größten Beerenangriff der Geschichte gehandelt, weshalb man ja bis heute von Großbeeren spricht. Chantk (Diskussion) 17:22, 18. Jun. 2013 (CEST)
Versteh nicht, was hier spaßig sein soll. Was ist, wenn die Frage völlig Ernst gemeint ist? --Wicket (Diskussion) 17:49, 18. Jun. 2013 (CEST)
Diese Schlacht wurde unter dem Titel Angriff der Killertomaten sogar verfilmt. Tomaten sind ja bekanntlich große Beeren. --Rôtkæppchen68 17:52, 18. Jun. 2013 (CEST)
Und ganz schlimm kann der Angriff von Beer-bel enden ... wechduck --Gwexter (Diskussion) 20:12, 18. Jun. 2013 (CEST)
Zur Großthematik der Klassiker ... --Zerolevel (Diskussion) 22:58, 18. Jun. 2013 (CEST)

Wenn die Leute nicht richtig lesen, geben sie falsche oder lustig gemeinte (und ebenso falsche) Antworten. Schade eigentlich, denn ee unterscheidet sich ja deutlich von ä. Wie auch immer das Missverständnis der Antwortenden aufkam, hier gibt es dioe Antwort auf Deine Frage: [6]. Danach sind Beerenangriffe nicht tödlich, nehmen aber an Häufigkeit zu. 217.9.49.1 11:33, 20. Jun. 2013 (CEST)

Frage an (Berufs-?)Chemiker

Zitat von DIskussion:Trivialname_(Chemie) (gekürzt): Trivialnamen sind essentiell für jeden Chemiker - Wer in meinen Vorlesungen von "Methylbenzol" spricht wird schief angeschaut und sofort gefragt, we den der "richtige" Name (Toluol) lautet. Ausserdem braucht man gar nicht zu probieren 2-Propanon an der Chemikalienausgabe zu kaufen, da wird man nur schief angeschaut - Aceton bekommt man allerdings sofort abgefüllt! Ich wünsche mir hier Hinweise auf die Ursprünge/Geschichte der Namen!

Stimmt das? Wird ein Chemiker tatsächlich "schief angeschaut" oder gar dumm gefragt, wenn er beispielsweise versucht, beim Chemikalienhändler Hydroxydiphenylethansäure oder 2-Hydroxybutan-1,4-disäure unter diesem Namen zu kaufen? Benutzen auch Chemiker im "normalen" Verkehr unter sich Trivialnamen? Wie ist es beisppielsweise an Universitäten?

--Eu-151 (Diskussion) 23:43, 18. Jun. 2013 (CEST)

Ich versuche, im täglichen Leben nach Möglichkeit die tradierten Trivialnamen zu nutzen. Die systematischen Namen sollte man aber dennoch kennen. Gegen Urinstein hilft Salzsäure und nicht etwa Chlorwasserstoffsäure. In manchen Fällen sind die Trivialnamen aber ungebräuchlich geworden. Dort verzichte ich dann eher darauf: Holzgeist sagt doch keiner mehr, das heißt jetzt Methanol. --Rôtkæppchen68 00:00, 19. Jun. 2013 (CEST)
+1 Die systematischen Namen dienen der zweifelsfreien Benennung (und die Systematik sollte jeder Chemiker kennen). Ich habe sie mit Wonne und flinker Zunge benutzt, als mein Haar noch voll und meine Geldbörse noch leer war. Aber bereits vor dem Vordiplom lernte man, dass z.B. Benennungen wie zyz-Xylol und Berliner Blau / Preußisch Blau / Turnbulls Blau / blausaures Eisen / Ferri-Kaliumcyanid etc. mehr Information enthalten als der rein systematische Name (man lernt den Krempel von verschiedenen Seiten (Anwendung, Geschichte, Struktur, ...) anzusehen... The Magic begins...).
Dann erfolgte der Übergang in die Biochemie - und da sind die Moleküle so gross und komplex, dass z.B. das Aufsagen des systematischen Namens für BSA (bovines Serumalbumin) etwa 10-20 Min. dauern würde (damit wäre der Vortrag halb rum und die Zuhörerschaft eingeschlafen). Bei FVIII wäre man eine gute Stunde beschäftigt - und nochmal 2 Stunden, um die Glykosilierungen und Sulfatierungen richtig hinzubekommen (wäre natürlich ein fantastisches systematisches Gedächtnistraining - wobei dann aber sicherlich ein paar Mal der Hochzeitstag auf der Strecke bleiben würde. Man müsste Abstriche machen.) Unterhalten sich aber zwei ProteinsequenziererInnen über die Validität des bestimmten C-Terminus, benennen sie jede Aminosäure einzeln mit der entsprechenden Sequenznummer (die Mediziner stehen dann immer ganz nervös dabei und schauen intensiv auf die Schuhspitzen... :-) So wie es die Sequenzierer tun, wenn die Mediziner weit ausholend die gerade diagnostizierte Krankheit systematisch benennen, statt "Tja Alter, Lungenkrebs!" zu sagen.)
Ergo: Man muss das System kennen, aber man sollte auch soviel Seiten- und Abkürzungswissen haben, um schnell und effektiv kommunizieren zu können. GEEZER... nil nisi bene 09:09, 19. Jun. 2013 (CEST)
Das "schief anschauen" könnte einen anderen Grund haben: Gruppen (Arbeitsgruppen, Studenten, Teenager, Senioren,...) können ein ganz bestimmtes Vokabular haben (Systematik hin oder her). Kommt dann ein Neuling, der (noch) die "falschen" Begriffe benutzt, guckt man sich den, der noch nicht völlig dazu gehört, erstmal von der Seite an. Ein Bekannter, der in derselben Branche den Job gewechselt hat, hat mir das beschrieben: In der neuen Firma benutzten sie für die ihm bekannten Systeme andere Kürzel, wodurch er anfangs als "Neuer" auszumachen war, weil er (a) seine alten Begriffe benutzte und (b) die firmeneigenen noch nicht verstand. GEEZER... nil nisi bene 10:30, 19. Jun. 2013 (CEST)
Ab morgen kommt bei mir nur noch β-(2S,3S,4S,5R)-Fructofuranosyl-α-(1R,2R,3S,4S,5R)-glucopyranosid in den Kaffee. --Rôtkæppchen68 14:21, 19. Jun. 2013 (CEST)
Also bei 2-Propanon werden ganz sicher Irritationen hervorrufen und ein dummer Spruch fallen, schon weil das mit der 2 uebersystematisiert ist. Ich würde aber sagen das sind wirklich nur eine Handvoll bei den Restlichen wird der systematische Name durchaus verstanden. Bei deinem Chemikalienhaendler Beispiel kann es aber durch etwas dauern bis man dann an der richtigen stelle im Katalog gesucht hat. Dafuer gibt es ja dann CAS-Nummern.--Livermorium (Diskussion) 23:03, 19. Jun. 2013 (CEST)
zum Thema: SMBC (nicht signierter Beitrag von 141.20.106.66 (Diskussion) 12:15, 20. Jun. 2013 (CEST))

Woher kommt der Sauerstoff in unserer Atmosphäre

Die gängige These ist, wenn ich nicht was verpaßt habe, folgende: Zu Beginn war die Atmosphäre ohne Sauerstoff. Dann haben Algen oder Bakterien die Luft langsam durch ihren Energieumsatz den Sauerstoff von anderen Elementen abgespalten und damit die Atmosphäre angereichert. Wenn Darwin recht hat, und er hat recht, dann kann diese These doch so nicht stimmen, denn sein Kernsatz ist doch: Die Evolution kam dadurch zu Stande daß sich alle Lebewesen an immer neuen Umweltverhältnissen anpassen mußten. Nach der obigen These hätten aber die Lebewesen ihre Umwelt geschaffen und nicht umgekehrt. Also bleibt die Frage: Wo kommt der Sauerstoff her. Wenn man Wasser (H2O) durch elektrischen Strom aufspaltet dann erhält man doch genau das: 2 Gase, Wasserstoff und Sauerstoff. Wasser hat es in der Erdkruste genug, und elektrische Ströme auch. Nach dem ein Wissenschaftler nachgewiesen hat, daß die Erde großflächig diese beiden Gase „ausatmet“ wäre es doch nicht abwegig wenn man die Herkunft des Sauerstoffes unter diesem Aspekt neu überdenken würde.

Wenn hier ein grober Denkfehler vorliegt, dann werdet ihr mir das schon schonend beibringen,

bin gespannt darauf. (Avis an Eike, laß deine Troll Flinte vorläufig im Schrank, ich mein es ernst).


--85.1.106.78 23:52, 18. Jun. 2013 (CEST)

Denkfehler von vorne bis hinten.
  1. Evolution kam nicht zustande sondern ist ein fortlaufender Prozess.
  2. Lebensformen passen sich an ihre Umwelt an. Wie man daraus zu dem Schluss kommen kann, dass sie sie deshalb nicht verändern können, ist mir schleierhaft.
  3. Die Lebensformen, die Sauerstoff abgeben, spalten diesen von CO2 ab um an Kohlenstoff zu kommen, sie haben kein Interesse an einer sauerstoffreichen Atmosphäre, das ist nur Nebenprodukt.
  4. Die Lebensformen, die Sauerstoff atmen, tun dies um an Energie zu kommen, sie brauchen eine sauerstoffreiche Atmosphäre und haben sich erst entwickelt, als es diese bereits gab.

PοωερZDiskussion 00:11, 19. Jun. 2013 (CEST)

Siehe Große Sauerstoffkatastrophe. --Pp.paul.4 (Diskussion) 00:15, 19. Jun. 2013 (CEST)
Der Sauerstoff (O2) auf unserem Planeten ist biogen. Gewisse Mikroorganismen (Algen) haben durch Zufall die Fähigkeit erlernt, Sonnenstrahlen durch Photosynthese als Energiequelle zu nutzen, als chemisches Abfallprodukt haben sie Sauerstoff in die Atmosphäre abgegeben. Später haben sich andere Organismen entwickelt, die keine Photosynthese betreiben, sondern in ihren Stoffwechsel genau dieses Abfallprodukt nutzen. Dazwischen sind natürlich ein paar Millionen Jahre. --El bes (Diskussion) 00:45, 19. Jun. 2013 (CEST)
Natürlich hat Darwin recht. Aber die Evolution ist nicht nur fortlaufend, sondern dynamisch und komplex. Die Lebewesen, die sich verändern, sind gleichzeitig einer der Faktoren, die Veränderungen (Anpassungen) von Lebewesen verursachen. Beispiel Verfügbarkeit von Futter (das sind Lebewesen, nämlich meist Tiere oder Pflanzen), Beispiel große Weidetiere (Rinder, Elefanten...), die bestimmte Pflanzen nicht mehr hochwachsen lassen (sondern vorher platttrampeln) und dadurch anderen Pflanzen günstigere Bedingungen schaffen.
Dass Lebewesen auch ihre Umwelt mit "schaffen", widerspricht nicht der darwinschen (und inzwischen verfeinerten) Darstellung der Evolution, sondern beeinflusst und verändert fortlaufend die Bedingungen, unter denen und durch die Evolution stattfindet. Heute ist vermutlich der Mensch diejenige Spezies, die Bedingungen am stärksten und am schnellsten verändert. Die Evolution setzt auch die so entstehenden Zustände mit den ihr eigenen Prinzipien um - diesen Prinzipien ist es egal, wer die jeweiligen Zustände geschaffen hat. Es kann passieren, dass die Ergebnisse dann der Spezies Mensch nicht gefallen oder nicht zuträglich sind (Verkarstung von Böden nach Abholzung der Wälder, nur mal als Beispiel, Zunahme von "Schädlingen" durch anthropogene Veränderungen) - aber auch das ist der Evolution / Natur egal... Chantk (Diskussion) 01:02, 19. Jun. 2013 (CEST)
In Cyanobakterien schreiben wir zur Zeit: "Vor etwa 2,5 Milliarden Jahren veränderten die sich massenhaft im Wasser verbreitenden Vorläufer der heutigen Cyanobakterien die Lebensbedingungen auf der Erde entscheidend. Sie nutzten das Sonnenlicht zur Photosynthese und setzten als Abfallprodukt Sauerstoff (O2) frei. Diese massenhafte Produktion von Sauerstoff führte schließlich zu einer entscheidenden Veränderung der sauerstofflosen Atmosphäre in eine sauerstoffhaltige Atmosphäre." Als Referenz ist angegeben: Heinrich D. Holland: The oxygenation of the atmosphere and oceans. In: Phil. Trans. R. Soc. B, Bd. 361, S. 903-915. doi:10.1098/rstb.2006.1838. Wenn ich den Fragesteller richtig verstehe, geht es (wenn die Annahme stimmt) um die Frage: Warum haben diese Vorläufer der heutigen Cyanobakterien sich verändert und Sauerstoff produziert? Wesentlich scheint mir der Hinweis in Große Sauerstoffkatastrophe, daß es oxygene Photosynthese auch bereits vor dem GOE gab.
Es gelang, 27 Seemeilen südlich von Island ein sehr altes, übriggebliebenes (besser: von der Evolution übersehenes) Cyanobakterium zu einem Interview zu überreden:
Frage: Ähm ...
Altes Cyanobakterium: Machen Sie's kurz. Und keine Fragen zur Systematik. Ich bin was ich bin.
Frage: Es geht um den Sauerstoff. Warum produzieren Sie O2 ?
Altes Cyanobakterium (lacht): Das fragt man einen Hund auch bei einem bestimmten Verhalten und meine Antwort ist die gleiche: Weil ich es kann.
Frage: Aber der Sauerstoff...
Altes Cyanobakterium: Ach Quatsch, der Sauerstoff war mir doch immer wurscht. Wir hatten in der Zeit der zweiten Atmosphäre jede Menge Kohlendioxid um uns rum...
Frage: ...durch die hohe UV-Einstrahlung...
Altes Cyanobakterium: Genau, dafür konnten wir ja garnix. Das war reine Photochemie. Wasser-, Methan- und Ammoniakmoleküle wurden zu Kohlendioxid und Stickstoff...
Frage: Und die damaligen Cyanobakterien ihrer Art?
Altes Cyanobakterium: Na, wir hatten Hunger! Leben bedeutet Stoffwechsel. Schreiben Sie sich das hinter die Ohren, Sie junger Spunt: Leben bedeutet Stoffwechsel! Und wir wollten leben... Natürlich...
Frage: Aber warum ausgerechnet vom Kohlenstoff?
Altes Cyanobakterium: Der war lecker. Wahrscheinlich haben andere auch was anderes ausprobiert. Aber wenn was funktioniert bleibt man ganz gerne dabei und gewöhnt sich dran. Mir hats jedenfalls immer geschmeckt und chemisch war's ja auch ziemlich einfach und praktisch...
Frage: Der Sauerstoff, den Sie damit freigesetzt haben, hat die Welt dann völlig verändert.
Altes Cyanobakterium: Ja, warum auch nicht? Alles ändert sich. Und für uns konkret hat sich ja auch garnicht so viel verändert. Außerdem haben wir auch schon vor dem GOE zu einer Menge Oxidationen beigetragen. Das war ja auch ein hartes Stück Arbeit, bis da sehr viel gesättigt war. Und irgendwann war dann einfach Kohlendioxid auch total in und mehr als genug davon auf dem Markt, die Temperatur war total angenehm, unsere Farben waren cool und wir haben eben zugelangt und es uns wohl sein lassen. Und wir haben, um ein Bild zu benutzen, nicht nur gefressen sondern sozusagen auch gepupst. Ihr Menschen setzt eure Haufen ja auch überall hin und denkt nicht über die Folgen nach...
Frage: Tja... Dann jedenfalls herzlichen Dank für Ihre Antworten und noch alles Gute.
Altes Cyanobakterium (läßt sich davontreiben): Wurde auch Zeit.
--84.191.136.149 02:15, 19. Jun. 2013 (CEST)
<Wütender Zwischenruf aus dem Publikum> Kommen Sie zurück! Wir sind noch nicht fertig! Warum haben Sie Ihre Delegation auf dem letzten Klimagipfel angewiesen, einer Reduktion der Sauerstoffproduktion nicht zuzustimmen? Wessen blaugrüne Interessen haben Sie da vertreten?? Sie produzieren tonnenweise ein korrosives Gas, ohne sich um die Folgen zu kümmern! Das ist doch unverantwortlich! Warum steigen sie nicht auf alternative Energien um und betreiben eine anoxygene Photosynthese mit harmloser Ablagerung von umweltfreundlichen Schwefelkügelchen? In Äonen wird man auf Sie zurückblicken und in Ihrem Verhalten - ich wiederhole! - in Ihrem verantwortungslosen Verhalten die Ursache der Probleme unserer Nachkommen .... <Sicherheitskräfte entfernen den zeternden Störer> GEEZER... nil nisi bene 07:25, 20. Jun. 2013 (CEST)
Nachtrag zu der Elektrolyse-Hypothese: Welcher Wissenschaftler ist das und um wieviel Wasserstoff und Sauerstoff handelt es sich denn angeblich? (Eine "ausatmende" Erde scheint mir da ein deutlich überstrapaziertes Bild zu sein.) Bedenke einfach mal, wo vermutlich mehr Energie herkommt: Vom Sonnenlicht für Photosynthese oder durch "elektrische Ströme" in der "Erdkruste". Sauerstoff ist in der Luft zu etwa 20% vorhanden. Wieviel Energie braucht es wohl, diesen Sauerstoff durch Elektrolyse zu erzeugen... ? Wo ist denn dann eigentlich der viele Wasserstoff abgeblieben? Schade, das alte Cyanobakterium ist schon weg... --84.191.136.149 02:46, 19. Jun. 2013 (CEST)
Im Karbon waren's übrigens mehr als 30 %. Anschlussfrage: Gab's da tatsächlich viel mehr gasförmigen Sauerstoff, oder nur weniger sonstige Atmosphäre? --Zerolevel (Diskussion) 14:52, 19. Jun. 2013 (CEST)
Entwicklung des O2-Gehaltes in der Atmosphäre während der letzten Jahrmilliarde
Ok, dann aber auch noch der Hinweis auf Entwicklung der Erdatmosphäre, woraus die nebenstehende schöne Grafik entnommen ist... --84.191.134.86 15:32, 19. Jun. 2013 (CEST)

SÄUREN die KALIUMKANAL öffnen

Der en:WP-Artikel zu den Kaliumkanälen erwähnt aktivierende Säuren. Oder geht es um die Aminosäuren im Kanal-Komplex selber? GG

Hallo zusammen,

ich habe gleich zwei Fragen:

1. Ich habe folgenden Artikel gelesen:

Säuren die Kaliumkanal öffnen: R/KXX 4-7

Was bedeutet das genau? Welche Säuren sind hier gemeint?


2. Gibt es einen Zusammenhang in den Aminosäureketten:

- GHB - Asparagin - GABA

Hängen diese Säuren in irgend einer Weise zusammen?

Vielen Dank schonmal und liebe Grüße Vera (nicht signierter Beitrag von Frau Holle1969 (Diskussion | Beiträge) 09:10, 19. Jun. 2013 (CEST))

Hallo Vera! Kannst du bitte noch die Quelle und den Zusammenhang angeben? GEEZER... nil nisi bene 09:23, 19. Jun. 2013 (CEST)
Vielleicht geht es darum: [7], Seite 7. Es handelt sich um basische, positiv geladene Aminosäuren in der Peptidkette des Proteins. R steh für Arginin, K für Lysin. X steht für eine beliebige Aminosäure. Diese Abfolge der Aminosäuren wiederholt sich mehrmals wodurch dem Kaliumkanal eine spannungsinduzierte Kanalöffung erlaubt wird.
GHB steht für 4-Hydroxybutansäure und GABA für γ-Aminobuttersäure, beides keine Aminosäuren, im Gegensatz zu Asparagin. Diese Substanzen binden nicht am Kaliumkanal, sondern an Rezeptoren, wie zum Beispiel GABA-Rezeptoren. --78.34.19.11 18:34, 19. Jun. 2013 (CEST)

Probleme mit dem Papierkorb (OS X Mountain Lion)

Ich bin auch nach knapp einem Jahr nach meinem Wechsel von PC zu Mac ein Mac-Analphabet. Folgendes ist passiert:
Ich habe eine externe Festplatte für Time Machine bereitgestellt.
Diese war nun schon recht voll, was zu recht langsamen Backups geführt hat. Der Automatismus, der alte Backup-Dateien löscht, wenn die Festplatte langsam voll wird, greift wohl erst dann, wenn die Platte zu 99,99 % voll ist…
Also löschte ich einen Großteil der alten Backup-Dateien (die schon deswegen entbehrlich sind, weil ich ein zweites Backup-System habe).
OS X verschob nun die Dateien in den Papierkorb. Der läßt sich nicht mehr entleeren. Wird der Entleer-Vorgang gestartet, erscheint eine Anzeige, die die Zahl der zu löschenden Dateien anzeigt. Er zählt und zählt, irgendwann landet er bei -269000 (das Minus ist nicht mein Irrtum; es steht wirklich da; die Zahl ist irgendeine sechsstellige Zahl).
Ein Versuch mit „sudo rm -rf ~/.Trash/*“ brachte gar nichts. Was tun? --Freud DISK Konservativ 11:47, 19. Jun. 2013 (CEST)

(nur weil sich noch keiner hier geregt hat) Hallo Freud, ich hätte ja zuerst versucht Time Maschine selbst zum Löschen der Daten zu bringen, keine Ahnung ob das per Hand so vorgesehen ist. Aber wie wäre es wenn man den Papierkorb einfach (vorübergehend) zeitweise wieder ausräumt ? D.h. die Hälfte der Daten wieder zurückbewegen und dann nochmal einen Löschversuch probieren, zur Not auch mehrfach reduzieren. Alternativ kannst Du den Löschbefehl ja auch im Dateisystem anwenden ?!? Grüße --RalfDA (Diskussion) 14:47, 19. Jun. 2013 (CEST)
Hm. Zunächst mal: Danke für Deine Antwort. Ich war schon gestern drauf und dran, das zu machen - aber mei, es sind halt zusammen 1,8 TB. Das dauert und dauert und dauert - und ob’s gelingt, weiß man nicht. Notfalls also mache ich es so. --Freud DISK Konservativ 17:40, 19. Jun. 2013 (CEST)
"Ein Versuch mit „sudo rm -rf ~/.Trash/*“ brachte gar nichts." Wenn ich mich nicht irre, sprichst Du mit diesem Befehl den Papierkorb Deines Nutzerkontos auf der internen Festplatte an. Du müsstest vielleicht mit cd auf die externe Festplatte (namentlich ansprechen) wechseln und dann „sudo rm -rf .Trash/*“ oder so verwenden. --° (Diskussion) 18:01, 19. Jun. 2013 (CEST)
Aber isch abe doch nur eine Papierkorb... Dachte ich zumindest. --Freud DISK Konservativ 19:37, 19. Jun. 2013 (CEST)
Im GUI hast Du nur einen Papierkorb, aber hinter den Kulissen (Shell/Terminal) hat natürlich jeder einzelne Benutzer auf jeder einzelnen Festplatte seinen eigenen "Trash"-Ordner. Würde auch garnicht anders gehen, oder willst Du das Dein Papierkorb geleert wird, wenn ein anderer Benutzeraccount seinen Papierkorb leert? --° (Diskussion) 19:48, 19. Jun. 2013 (CEST)
Du hast natürlich recht. Aber da ich niemanden, und das ist buchstabengetreu gemeint, an meinen Mac lasse, ist die Unterscheidung aus meiner Warte theoretischer Natur. Aber, so oder so: Was tun? --Freud DISK Konservativ 20:06, 19. Jun. 2013 (CEST)
Versuch mal folgendes: Gib in der Shell "cd /Volumes" ein. dann müsstest Du mit "ls" eine Auflistung Deiner Festplatten bekommen. Dann mit "cd <Name der Festplatte>" zur externen Festplatte wechseln (wenn der Plattenname Leerzeichen enthält mit Anführungszeichen eingeben) und dann mit cd und ls suchen, bis Du das Trash-Verzeichnis gefunden hast. Dann wie oben geschrieben löschen. --° (Diskussion) 20:27, 19. Jun. 2013 (CEST)

St. Martini-Kirche in der Martinistraße, Bremen

Ein Hinweis an die Verantwortlichen ---> Rubrik Kirchen: die hier abgebildete Kirche ist die St. Martini-Kirche in der Martinistraße, Bremen. Sie ist hier fälschlicherweise als "Liebfrauenkirche" bezeichnet worden. Bitte ändern!

--91.96.151.167 13:13, 19. Jun. 2013 (CEST)

Nun suchen wir mal nach dem falschen Bild... --Eingangskontrolle (Diskussion) 15:10, 19. Jun. 2013 (CEST)

Ist doch ganz einfach. In der Rubrik Kirchen :-) Aber wir können es ja die Verwantwortlichen machen lassen. --AndreasPraefcke (Diskussion) 15:16, 19. Jun. 2013 (CEST)
Liste_der_Kirchen_in_Bremen könnte gemeint sein, die sind sich ähnlich, aber es gibt doch Unterschiede (mit/ohne Uhr) und sicher noch mehr. --Eingangskontrolle (Diskussion) 15:18, 19. Jun. 2013 (CEST)
Bremen#Die_Kirchen_in_der_Altstadt? --Arcimboldo (Diskussion) 16:52, 19. Jun. 2013 (CEST)

Ladegeräte für Samsung Galaxy S2 und S4

Hallo Zusammen,

ich brauche 2 neue Ladegeräte für die Galaxy S2´s hier im Haus. Die letzten die ich gekauft habe für 4 Euro sind natürlich Mist, aber laut Amazon alles "ORIGINAL". Jetzt meine Frage. Wenn ich ein ORIGINAL Ladegerät vom S4 kaufe, dass 2000 mAh "Ladekraft" hat und damit die S2 Geräte dauerhaft lade, die normalerweise mit einem 1000 mAh Gerät geliefert werden, kann ich damit die Handys beschädigen??? --95.208.189.95 13:17, 19. Jun. 2013 (CEST)

Nein, die Angaben am Ladegerät besagen nur, was es maximal liefern kann, wieviel Strom wirklich fließt bestimmt das Handy selbst. Nur umgekehrt kanns Probleme geben: man sollte nicht ein Handy, das z.B. 2000mA braucht mit einem 1000mA-Ladegerät laden. --MrBurns (Diskussion) 13:23, 19. Jun. 2013 (CEST)

Vielen Dank. Dann lasse ich jetzt die Finger von den 4 € "Original"-Teilen, bei denen jedes 2. schon nach nem Monat defekt ist. Anbei noch ne Frage: Laut Amazon sind die Teile ALLE Original, ich habe aber noch nie eins erhalten, dass so gut ist wie das Gerät ist, das mitgeliefert wurde. Weiß jemand, wo ich das wirklich ORIGINALE Ladegerät bekomme? --95.208.189.95 13:30, 19. Jun. 2013 (CEST)

Ob die Ladegeräte original sind, kannst Du leicht anhand der Artikelnummer herausfinden. Für das S2 gibt es zwei Ladegeräte, eines mit abtrennbarem USB-Kabel (ETA0U80E) und eines mit festem Kabel (ETA0U10E). Das ETA0U10E wird auch beim Galaxy Ace mitgeliefert. Es hat 5 Volt 0,7 Ampere. --Rôtkæppchen68 14:43, 19. Jun. 2013 (CEST)
Drauf achten welche Spannung angegeben ist. Es fließt max. soviel Strom wie das Mobiltelefon braucht. der Laderegler ist im Mobiltelefon selbst eingebaut. Bei USB sind es 5V Es wird nichts kaputt gehen, wenn das Ladegerät Netzteil mehr Strom liefern kann und keine zu hohe Spannung ausgibt. Mit dem USB-Adapterkabel hast Du freie Auswahl. Da geht selbst der PC. Bei zugekauften USB-Adapterkabeln drauf achten, ob das Kabel zum Laden gemacht ist. Wenn am USB des PC geladen wird, drauf achten ob es ein USB 1.0 , 2.0 oder 3.0 ist. Manche unterstützen nur ½A (=500mA) Strom, da spinnt der Laderegler. Das kann mit einem USB-Doppelstecker gelöst werden. Man benötigt 2 USB-Anschlüsse am PC, nur einer verbindet dabe die Datenleitungen, der zwete verbinden nur +5V und Masse (= Minus/0V). --Hans Haase (Diskussion) 22:55, 19. Jun. 2013 (CEST)
Sicherungen und Induktivitäten auf der 5V-Leitung für USB- und PS/2-Anschlüsse
Wo gibt es heute noch USB 1.0? Der hat doch nie wirklich funktioniert, weswegen sich erst USB 1.1 ubiquitär verbreiten konnte. --Rôtkæppchen68 23:28, 19. Jun. 2013 (CEST)
Wenn das 5-Volt-Netzteil oder der USB-Anschluss zu schwach ist, merkt man das ganz schnell daran, dass der Akku nicht vollständig auflädt oder die Ladekontrollanzeige ständig an ist. Das Netzteil meines Fotoapparats (4,2 Volt, 400 mA) taugt trotz USB-A-Buchse nicht als USB-Ladegerät. Mein Sony Walkman lädt daran eeeewig und der Akku wird nicht voll. Am PC mit 5 Volt 500 mA ist er ruckzuck geladen. Bei manchen PCs können auf den USB-Ports durch geeignete Software bis zu 1,5 Ampere freigeschaltet werden, nicht nur bei USB 3.0, wo standardmäßig 0,9 Ampere bereitgestellt werden. Auf Arbeit hab ich noch zwei gebrauchte Netzteile mit 5 Volt 300 Ampere. Wie schnell mein Smartphone wohl damit geladen wird? --Rôtkæppchen68 23:37, 19. Jun. 2013 (CEST)

Da gibt es eine Reihe von Ursachen:

  • PC: USB bricht bei höheren Strömen ein: Grund: mehrfach auslösende/PTC-Sicherungen machen zu, und wegen der Störabstrahlung sind die 5V gefiltert (ist bei modernen PCs anders!)
  • Steckerladegerät gealterte oder überlastete Bauteile
  • Zigarettenanzünderadapter in Schrottversion nur für max. 500mA (statt 1A – billigerer IC aus selber Serie)
--Hans Haase (Diskussion) 10:26, 20. Jun. 2013 (CEST)

Frage zur redensartlichen Gürtellinie

Mir ist bewusst, dass das Folgende wie eine Trollfrage klingt. Aber ich versichere, sie hat für einige Menschen recht ernste Bedeutung, und auch reine Jas und Neins helfen als Tipps weiter.

Situation: In einem Verein tut sich ein weibliches Mitglied sehr als Kritikerin der FunktionärInnen hervor. Sie war früher selber eine und ist aus Protest gegen Pflichtversäumnisse von anderen zurückgetreten. Sie postet reichlich (und häufig nachts) schriftliche Zitate zur Untermauerung ihrer Ansichten. Man munkelt auch über sie, dass sie ihre Nächte primär mit der Sortierung und Aufbereitung solcher Zitate verbringt, um jedes im optimalen Moment gegen jemanden einsetzen zu können. Eine Funktionärin (mit einem männlichen Vereinskollegen liiert) erklärt die Kritik an ihr als für sich irrelevant. Sie werde ihre Arbeit machen statt darauf zu reagieren, und schließt mit:

"Abschließend bemerke ich, dass ich mich nicht nachts in Erinnerungen schwelgend und frustriert vor den Computer setze, um mich und andere dort mit meinen unterdrückten Hormonen zu quälen. Ich verbringe die Nächte genussvoller und kann das weiterempfehlen."

Frage: Ist diese Aussage als "unter die Gürtellinie gehen" zu bewerten oder eher nicht? --Anonyme Frage (Diskussion) 13:38, 19. Jun. 2013 (CEST)

Das ist doch gar nicht die Frage. Bei einem solchen Verein hilft nur: schnellstens austreten. Sollen sie sich doch mobben wie sie lustig sind. --FA2010 (Diskussion) 13:45, 19. Jun. 2013 (CEST)
Ja. Die Aussage soll andeuten, dass diejenige sexuell frustriert sei. Inwieweit da auch Strafgesetze tangiert sein könnten, wage ich nicht zu beurteilen. --Eike (Diskussion) 13:51, 19. Jun. 2013 (CEST)

Aussage is völlig in Ordnung. Niemand wurde beleidigt. --95.208.189.95 13:47, 19. Jun. 2013 (CEST)

Sicherlich ist das unter der Gürtellinie. Es wird ein psychologisierendes negatives Bild einer Persönlichkeit gezeichnet, das auf nichts anderem als bösartigen Unterstellungen fußt. Mit dem sachlichen Konflikt hat das nichts zu tun. Noch nicht einmal mit der Metaebene einer Kritik der Verkehrsformen. Zusammengefaßt und zugespitzt ist der Anwurf: Du bist krank und brauchst mal guten Sex und mir, auf die ich dich jetzt neidisch mache, geht es viel besser ätschebätsch. Interessant, daß diese Giftspritzerei von einer Frau kommt. Ursprünglich ist es ja ein typisches Machoverhalten, Kritik an Frauen mit ihrem Sexualleben zu verbinden und dann sich selbst ins Spiel zu bringen, wenn es darum geht, wer es wem "mal richtig besorgen" wird.
Die Formulierung "Unter der Gürtellinie" bezieht sich aber imho nicht auf den Bereich der Sexualität sondern allgemein auf einen tabuisierten Bereich. Dazu gehört auch, die Kontrahentin subtil zu psychiatrisieren ("in Erinnerungen schwelgend und frustriert") oder Privatangelegenheiten mit Sachfragen zu vermischen. Ich würde die Funktionärin, die einerseits angeblich nicht auf die Kritik reagieren will, andererseits doch in dieser ekelhaften Form reagiert, statt zur Sache Stellung zu nehmen und zu der für eine Meinungsbildung nötigen Transparenz beizutragen, bei geeigneter Gelegenheit abwählen. Wer sowas macht, macht sicher keinen guten Job. --84.191.134.86 14:06, 19. Jun. 2013 (CEST)
(quetsch) Doch, macht sie (meiner Meinung nach), und die Kritikerin war ihrerseits äußerst unfair und destruktiv. Die Kritik zu ignorieren fand ich schon richtig. Das Zitat war dann aber (meiner Meinung nach) ein unnötiges Folgeproblem. --Anonyme Frage (Diskussion) 16:26, 19. Jun. 2013 (CEST)
"Unter der Gürtellinie" sehe ich in zwei Bedeutungen: einerseits als sexuelle Anspielung, und andererseits - vielleicht aus dem Boxen - als unerlaubte und unfaire Handlung/Aussage (hier nicht unbedingt im tabuisierten Bereich, außer, dass Unfairness grundsätzlich tabu ist). --Eike (Diskussion) 14:34, 19. Jun. 2013 (CEST)
Yepp, die Ableitung aus dem Boxsport scheint plausibel. Und die online-Redewendungsseiten scheinen es zu bestätigen. Also auch: Eine unfaire Regelverletzung (was ja auch zu den Regeln, die einen sachlichen, zivilisierten Diskurs gewährleisten sollen, paßt).
Ich finde hier in einem Forum einen Hinweis, den man mit seriösen Quellen noch verifizieren müßte: "1719 wurde der Preiskämpfer James Figg so bekannt, daß er zum englischen Meister erklärt wurde. Diesen Titel hielt er dann 15 Jahre. Einer seiner Schüler, Jack Broughton, unternahm die ersten Schritte Boxen zu einer akzeptierten Sportart zu machen. Broughton entwickelte 1743 das erste Regelwerk, das, mit minimalen Einschränkungen, bis zur Einführung der London Prize Ring Rules 1838 gültig war. Er lehnte die Kneipenschlägertricks seiner Vorgänger ab, und verließ sich hauptsächlich auf seine Fäuste. Obwohl Ringergriffe immer noch erlaubt waren, durfte nicht mehr unter der Gürtellinie gegriffen werden." Demnach schützte die Gürtellinie (nach dem Artikel Boxen imaginär in Höhe des Buchnabels und durch den oberen Hosenrand erkennbar) nicht nur vor dem "Schlag unter die Gürtellinie" sondern in der Frühzeit des Sports auch vor Griffen. --84.191.134.86 15:03, 19. Jun. 2013 (CEST)

Klingt wie ein Edit auf WP:VM oder WP:SP. Natürlich ist es ein persönlicher (und nicht sachlicher), ein herabwürdigender (und nicht argumentativer) Edit. --Freud DISK Konservativ 14:18, 19. Jun. 2013 (CEST)

Siehe auch: Liste deutscher Redewendungen#G. --Merrie (Diskussion) 19:18, 19. Jun. 2013 (CEST)

Marsallis

Aus welchem Land stammt der Familienname Marsallis (Marsalis) ursprünglich? Tippe auf Italien. --80.226.24.6 17:38, 19. Jun. 2013 (CEST)

Da "Marsalis" wohl eher nur in Amerika vorkommt, könnte er auch eine Verballhornung von Marcellus o. ä. sein. Ich würde da eher nicht "tippen" wollen. --FA2010 (Diskussion) 18:19, 19. Jun. 2013 (CEST)
Erste Näherung: Von 5 in die USA eingewanderten "Marsalis" haben 2 Ursprungsorte in Italien (bei den anderen sind keine Ursprungsländer angegeben.
Zweite Näherung: Hinknien und Familienforschung machen... :-) GEEZER... nil nisi bene 20:25, 19. Jun. 2013 (CEST)

Interwikiverlinkungen ?

Warum fehlt in der englischen Wikipedia und auf Wikidata die deutsche Verlinkungsseite und wird gleichwohl vollständig auf der deutschen Wikipedia alle Interwikis angezeigt ? Beisspielsweise bei dem Artikel Snowboard

178.11.189.123 18:54, 19. Jun. 2013 (CEST)

Ich kann in beiden Versionen jeweils den Link zur anderen sehen. --Eingangskontrolle (Diskussion) 18:59, 19. Jun. 2013 (CEST)

http://www.wikidata.org/wiki/Q2000617 ist das Sportgerät Snowboard, das haben wir nicht extra, sondern unser Artikel Snowboard gehört zur Familie "Snowboarding", das ist http://www.wikidata.org/wiki/Q178131 --AndreasPraefcke (Diskussion) 19:03, 19. Jun. 2013 (CEST)

Wo ist dieses Bild wohl aufgenommen? Auf der Festung Königstein oder früher. Und vielleicht erkennt jemand die Person neben ihm. --Eingangskontrolle (Diskussion) 18:56, 19. Jun. 2013 (CEST)

Man könnte hier (die Gefangenen) nachsehen und die Namen der anderen Franzosen - denn es ist einer - einer Bildersuche unterwerfen. GEEZER... nil nisi bene 07:04, 20. Jun. 2013 (CEST)

Was ist ein Geusenwort oder was will es denn mal werden?

Was willst du denn mal werden? frug man früher Kinder. In dem Sinne Was ist ein Geusenwort oder was will es denn mal werden? Der (deutschsprachige) Begriff ist noch dabei, etabliert zu werden. Wikipedia leistet dabei die Hauptarbeit; vielleicht geht ihr noch irgendein Sozpädseminar oder Germanistikleerstuhl zur Hand, aber vor allem ist hier die Wikipedia tätig. Sie sollte also auch wissen, was es denn mal werden soll. Laut Artikel kann es nur Personengruppen bezeichnen, also die Piraten oder die Nerds oder so. Nun aber soll auch çapuling ein Geusenwort sein (auf der Hauptseite). Übrigens ganz großen Respekt vor den Türken, die hier großartig ironisch ein ganz tolles Wortspiel erfunden haben. Nur heißt doch die Personengruppe çapulcu - wenn auf dem Plakat Everyday I'm çapuling steht, dann soll eindeutig çapuling die Tätigkeit sein (bzw. ironischerweise eben gerade die nicht ausgeführte Tätigkeit), keinesfalls aber die Personengruppe.

Man müsste es eigentlich auch an anderen Stellen fragen, aber lasst es mal hier stehen. Also, soll der Begriff, wenn er denn mal etabliert ist, auch Tätigkeiten bezeichnen? --Chantk (Diskussion) 19:20, 19. Jun. 2013 (CEST)

Ich kann auch nicht ganz nachvollziehen warum çapuling ein Geusenwort sein soll, wo doch das "[Wort das] ursprünglich eine Personengruppe beschimpfen soll" çapulcu ist, während çapuling ein davon abgeleitetes Kofferwort ist und somit ein anderes Wort das eine positive Bedeutung bekommen hat. --Mps、かみまみたDisk. 21:25, 19. Jun. 2013 (CEST)
Dass die an sich negative (oder kritische oder beschimpfende) Bedeutung in eben ihr Gegenteil umschlägt, soll ja der Trick sein beim "Geusenwort".
Aber Du hast recht, dass çapuling aus noch einem zweiten Grund nicht der zunächst zu etablierenden Definition eines Geusenwortes entspricht, weil es ja dieses çapuling vorher gar nicht gab und es erst von der umgedrehten Bedeutung des eigentlich negativen çapulcu abgeleitet ist und zwar als künstliche Zusammensetzung mit einer englischen Endung (ob das ein Kofferwort ist, weiß ich nicht; da denkt man ja eher an zwei ganze Wörter, die zusammengesteckt werden).
Die Frage ist aber, wie ich schon andeutete, nicht, ob çapuling ein Geusenwort ist. Sondern wohin (auf welche Bedeutung) dieser Begriff Geusenwort etabliert werden soll (dass das im wesentlichen durch die Wikipedia geschieht, erfährt man durch intelligentes Googeln). Vielleicht liest hier der Etablierer (Etableur?) mit. Je nach der beabsichtigen Definition von "Geusenwort" ist çapuling eins oder nicht. Chantk (Diskussion) 21:35, 19. Jun. 2013 (CEST)
Ich bin schuld, weil ich das Wort so auf die hauptseite gebracht habe. Es handelt es sich eigentlich um einen Geusenwortstamm, weil ja verschiedene Flektionen als Schimpfwort und Selbstbezeichnung kursieren. Serten (Diskussion) 15:40, 20. Jun. 2013 (CEST).

Lebensmittel werden sauer durch Gärung, aber was ist daran schädlich?

So, ich hab jetzt Gärung und alle Artikel zu den verschiedenen Typen durchgelesen um diese Frage zu beantworten: kann ich sauer riechende Lebensmittel (die im frischen Zustand nicht sauer riechen), durch Kochen genießbar machen oder bleiben da schädliche Dinge übrig? Meine Antwort fällt mit "Ja" aus. Die Gärungen scheinen kaum schädliche Nebenprodukte abzuwerfen (im Gegensatz zu vielen Schimmelpilzen z.B.) und die gebildeten Säuren und Alkohole verdampfen alle bei Temperaturen niedriger als 200°C, was auf dem Herd auf höchster Stufe mit etwas Zeit locker überschritten wird. Einziger Nachteil ist also eine etwas sauer riechende Küche (oder eine explodierende, wenn zuviel Essigsäure dabei war :). Seh ich das richtig? -- 217.68.187.90 01:18, 20. Jun. 2013 (CEST)

Du kannst Sauerkraut roh oder gekocht essen. Essigsäure explodiert nicht (falls hier Anschlagsplaner mitlesen). GEEZER... nil nisi bene 07:31, 20. Jun. 2013 (CEST)
Es kommt auch darauf an, welches Lebensmittel du vor dir hast. Saure Milch ist nicht direkt schädlich, aber ungenießbar. Du kannst auch nicht garantieren, dass das Lebensmittel nicht auch weitere Mikroorganismen enthält. Wenn es lange genug herumstand um zu gären, können auch Schimmelpilze und/oder E.coli und/oder Botulinusbakterien enthalten sein. Ob du auf dem Herd 200°C in jedem Bereich des Essens erreichst hängt stark vom Wassergehalt ab. Wasser verdampft bei 100°C, da wirst du niemals 200°C erreichen. Bei kontrolliert vergorenen Lebensmitteln wie Joghurt, Sauerkraut und Kefir sieht das natürlich anders aus. --Sr. F (Diskussion) 07:55, 20. Jun. 2013 (CEST)
Einspruch! ;-) Ich weiss nicht, wieviel Liter Schlickermilch ich in meiner Jugend zum Verzehr vorgesetzt bekommen habe, aber mit einem Löffel Zimtzucker und dem Lächeln einer Mutter ist das durchaus geniessbar - und ich kann mich noch an den Geschmack erinnern. Oink! Oink! GEEZER... nil nisi bene 08:20, 20. Jun. 2013 (CEST)
Einspruch Logik! Ist Schlickermilch nur sauer, oder nur vergoren, oder sauer und vergoren, oder keines von beiden? --Pp.paul.4 (Diskussion) 09:23, 20. Jun. 2013 (CEST)
„Rose is a rose is a rose is a rose
Loveliness extreme.
Extra gaiters,
Loveliness extreme.
Sweetest ice-cream.
Pages ages page ages page ages.“
Habe erweitert. Jetzt fehlt nur noch die Ferkelmast was einigen meiner Kinderfotos einen wissenschaftlichen Hintergrund verleiht... GEEZER... nil nisi bene 10:28, 20. Jun. 2013 (CEST)
Die Schlickermilch: ...welche zu Quarkkäsen gebraucht, oder dem Gesinde mit eingebrocktem Brot zum Verspeisen gegeben wird. Im Präsens forumliert. Dann können wir ja mal Gesinde suchen und befragen, um mehrere Meinungen darüber zu hören. --80.140.145.40 10:59, 20. Jun. 2013 (CEST)
Signed, sealed, delivered - it's yours..! :-) GEEZER... nil nisi bene 11:52, 20. Jun. 2013 (CEST)

Die Antwort ist NEIN. Wie eigentlich schon in der Einleitung des Artikels Gärung beschrieben wird, hat der Begriff mehrere Bedeutungen. Verallgemeinert sind die gesteuerte Milchsäure- und alkoholische Gärung für den Menschen ungefährlich, egal ob kalt oder erhitzt. Ein erhöhter Stoffwechsel durch schnellere Verdauung nicht ausgeschlossen, da eine Frage des Gewöhnungsniveaus. Bei einer ungesteuerten derartigen Gärung sind weitere chemische und biologische Prozesse mit entsprechenden Produkten (zB. Methanol, Bakterien, Schimmel) geschehen, welche einen "säuerlichen" Geruch verursachen können. Das kann man nicht pauschal durch Erhitzen beheben.Oliver S.Y. (Diskussion) 10:40, 20. Jun. 2013 (CEST)

Verdorbene Lebensmittel enthalten einen ganzen Zoo an Mikroben, nicht nur ein paar Milchsäurebakterien. Jedes dieser Mikroben produziert andere Stoffwechselprodukte, die Du gar nicht alle kennen willst. Du kannst also nicht von vorneherein davon ausgehen, dass da nur gesunde Stoffe dabei sind. Botulismus wurde schon genannt. Auch wenn noch kein „Schimmel“ sichtbar ist, heißt das noch lang nicht, dass da kein Aspergillus drin ist. Schimmelbefall fängt unsichtbar an, kann aber dennoch toxisch sein (Aflatoxine).--Rôtkæppchen68 13:26, 20. Jun. 2013 (CEST)

Welcher Luftentfeuchter?

Welche Art von Luftentfeuchter soll ich für einen Weinkeller ohne Heizung mit Temperaturen zwischen 7 und 19 Grad Celsius über das ganze Jahr verwenden: Kondensations- oder Absorptionstrockner.

(nicht signierter Beitrag von Dahlailama (Diskussion | Beiträge) 11:01, 20. Juni 2013 (CEST))

Wie gross ist der Raum und welche Luftfeuchtigkeit herrscht da normalerweise? GEEZER... nil nisi bene 11:55, 20. Jun. 2013 (CEST)
Wenn der Raum tatsächlich um 12°C schwankt ist er zur Weinlagerung ohnehin ungeeignet. Gut ist eine jahreszeitliche Schwankung von höchstens 3°C, 7°C sind noch akzeptabel. Zu große Temperaturschwankungen bewirken sehr hohe Schwankungen der relativen Luftfeuchte und denen wirst du auch mit einem einfachen Trocknungsgerät nicht Herr. Das kann nämlich nicht die Luft in den Raumecken trocknen. Professionelle Geräte zur Gebäudetrocknung pusten gezielt warme Luft in Ecken und Ritzen, das ist für Wein gänzlich ungeignet. -- Janka (Diskussion) 12:47, 20. Jun. 2013 (CEST)
Die Raumtemperatur liegt ausserhalb des "normalen" Betriebsbereichs von Haushaltskompressoren. Das kann im Anlauf Probleme verursachen, weil die Viskosität des Öls im Kompressor in diesen Bereichen möglicherweise so hoch werden kann, dass der Kompressor im Anlaufbetrieb beschädigt werden kann. Handelsübliche Luftentfeuchter verwenden aber keine Kompressoren mit Ölheizung. Also würde ich den Wein umlagern, den Keller heizen und für ausreichend Luftaustausch sorgen. Das wird naturgemäss im Winter erfolgreicher sein als im Sommer. Yotwen (Diskussion) 13:10, 20. Jun. 2013 (CEST)

Name eines US-Kriegsschiffes gesucht

Hallo, Nach meiner Kenntnis wurde das US-Kriegsschiff, welches die Atombombe nach Hiroshima brachte auf der Rückfahrt von einem japanischen u-boot torpediert und versenkt. Könnten Sie mir bitte den Namen dieses Schiffes mitteilen? Vielen Dank im voraus und freundliche Grüsse, Roman Zigerli (nicht signierter Beitrag von 80.218.33.67 (Diskussion) 22:14, 13. Jun. 2013‎ (CEST))

USS Indianapolis (CA-35) (nicht signierter Beitrag von 92.225.68.152 (Diskussion) 22:17, 13. Jun. 2013‎ (CEST))
Archivierung dieses Abschnittes wurde gewünscht von: gp (Diskussion) 08:08, 21. Jun. 2013 (CEST)

Speedport hinueber?

Modem nach Gewitter

Nach Stromausfall fuhr ich gerade meinen Rechner mit Speedport und Peripherie hoch, Sicherung knallte durch. Sicherung rein, alles ausser dem Router Speedport W 503V funkioniert. Strom kommt an, drei der Laempchen glimmen schwach. Neustart ergebnislos, Reset auch. Technisch bin ich Oma. Neuen Speedport kaufen, alle Stecker genau so reinstecken, fertig? Oder gibt's eine etwas elegantere Methode? Der anfaengliche Kurzschluss macht mich etwas misstrauisch. Ich schalte bei längeren Reisen oefters alles aus (genauer: ziehe den Netzstecker für alles), da gab's bisher nie ein Problem. --Aalfons (Diskussion) 09:58, 21. Jun. 2013 (CEST)

Hattet ihr eventuel Gewitter, als es zum Stromausfall kam?--79.238.25.154 10:13, 21. Jun. 2013 (CEST)
Hört sich nicht gut an. Falls du ersetzen musst: Es gibt solche Geräte für wenig Geld. (Ich glaube in der Größenordnung 20 Euro, ohne Gewähr.) Du brauchst zusätzlich zur Verkabelung vermutlich noch einen Usernamen und ein Passwort zur Konfiguration, das sollte in irgendeinem Brief stehen. Es gibt allerdings auch Provider, bei denen man nur deren Modem verwenden darf und kann, dann müsstest du wohl von denen Ersatz bekommen. --Eike (Diskussion) 10:17, 21. Jun. 2013 (CEST)
Speedport hört sich nach Telekom an! Kauf oder Miete? -- Johnny Controletti (Diskussion) 10:23, 21. Jun. 2013 (CEST)
@IP: Ja, Gewitter. Hatte schon vor dem Stromausfall alles vom Netz. @Eike: Kann ich denn bei diesem Schadensbild sicher sein, dass der Router das Problem ist? @JC: telekom-kauf. --Aalfons (Diskussion) 10:33, 21. Jun. 2013 (CEST) sorry für die kurzen antworten, smartphone-edits. --Aalfons (Diskussion) 10:38, 21. Jun. 2013 (CEST)
Kann ich nicht sagen, aber wenn die Lämpchen jetzt schwächer leuchten als vorher, hört sich's für mich stark danach an. Im Normalfall solltest du den Router per Webbrowser (IP im Handbuch nachschlagen) erreichen können, das kannst du mal probieren. --Eike (Diskussion) 15:01, 21. Jun. 2013 (CEST)
Unter Umständen ist nur das Netzteil hinüber, die heutigen Billugnetzteile halten Überspannungspulsen nicht immer stand. Probier es mit einem Universalnetzteil aus, bei dem du die Spannung passend einstellst und bei dem die Stromstärke ausreicht. Polung beachten! -- Janka (Diskussion) 10:56, 21. Jun. 2013 (CEST)
So sieht ein Modem aus, wenn der Blitz in die Anschlussleitung schlägt. Gruss --Nightflyer (Diskussion) 11:05, 21. Jun. 2013 (CEST)
@janka: Habe kein Universalnetzteil mit Spannungswaehler. in den foren ist viel von kaputten netzteilen die rede, aber wuerden dann noch die dioden glimmen? @nightflyer: hübsch. als hier im haus vor drei jahren der blitz einschlug und in allen zehn wohnungen alle e-Geräte zerstörte, auch die Ausgeschalten, sah manches auch so aus. seither hatte ich auch diesen neuen router... --Aalfons (Diskussion) 11:17, 21. Jun. 2013 (CEST)
Wenn der W503V im Gewitter von Netz und Telefonleitung getrennt war, ist der Blitzschlag nicht die Ursache. Hatte es auch einmal ohne Gewitter bei einem Gerät diesen Typs. Es ist die interne Stromversorgung/Spannungsaufbereitung des Routers und ist reparabel durch Leute, die es können. --Hans Haase (Diskussion) 14:23, 21. Jun. 2013 (CEST)
Update: Neukauf erledigt. Man kann diese Dinger heutzutage tatsächlich selbst installieren, in omischer Geduld. --Aalfons (Diskussion) 16:52, 21. Jun. 2013 (CEST)
Archivierung dieses Abschnittes wurde gewünscht von: --Aalfons (Diskussion) 16:52, 21. Jun. 2013 (CEST)

Windows Update - wirklich "wichtig"?

Ich hab grad das Office Live add-in 1.5 vor Augen, ständig blinkt es, dies sei 'wichtig'. Ich seh dessen Nutzen aber nicht. Kann man wichtige oder auch optionale Updates aktiv ignorieren? Oder werden die einem immer wieder per Meldung aufgezwungen. Optional wäre z.B. der 'Bind Desktop v1.3'. Wenn ich diesen grundsätzlich nicht haben will, wie bekomm ich das aus meiner Liste möglicher Updates raus (außer es eben zu installieren)? Gruß, --WissensDürster (Diskussion) 15:27, 21. Jun. 2013 (CEST)

Rechtsklick auf das Update und dann "ausblenden" oder soetwas in der Art. Kann man auch wieder rückgängig machen bei Bedarf. --тнояsтеn 15:36, 21. Jun. 2013 (CEST)
Wollt ich auch schreiben. Ist vermutlich auch abhängig von der Windows-Version. --Eike (Diskussion) 15:37, 21. Jun. 2013 (CEST)
Bei XP Hakerl entfernen und im weiteren Verlauf "Keine weite… für dieses Update" anhaken. Ich fand es durchaus sehr wichtig. --Hans Haase (Diskussion) 15:47, 21. Jun. 2013 (CEST)
Ob es für jemanden wichtig ist hängt wohl davon ab, ob er Office Live nutzt, wenn das nicht der Fall ist, braucht man das Update nicht, daher wäre es unter "Optional" besser aufgehoben als unter "Wichtig". --MrBurns (Diskussion) 15:56, 21. Jun. 2013 (CEST)
Vielen Dank, auf den Rechtsklick hätte ich wohl auch selbst kommen müssen :) Nun muss ich nur hoffen, dass ich kein wirklich wichtiges Update so "entferne". --WissensDürster (Diskussion) 16:15, 21. Jun. 2013 (CEST)
Archivierung dieses Abschnittes wurde gewünscht von: WissensDürster (Diskussion) 16:15, 21. Jun. 2013 (CEST)

Biotische Umweltfaktoren

Sind Temperatur, Luftfeuchtigkeit, Sonneneinstrahlung und Konkurrenzkampf biotische Umweltfaktoren?--Joël57 (Diskussion) 18:11, 21. Jun. 2013 (CEST)

Diese Frage beantworten Dir die Artikel mit den weltfremden Lemmata Abiotischer Umweltfaktor und Biotischer Umweltfaktor. --Rôtkæppchen68 18:15, 21. Jun. 2013 (CEST)
(BK) Biotisch bezieht sich auf Lebewesen, die können natürlich Temperatur, Luftfeuchtigkeit und Sonneneinstarhlung beeinflussen (z.B. Beschattung durch einen Baum) aber an sich sind das abiotische Umweltfaktoren. Beim Konkurrenz ist die Wirkung natürlich direkter, wobei der umgekehrt natürlich auch wieder von abiotischen Faktoren beeinflusst wird (bei Staunässe gibt es keine Konkurrenz um Wasser, bei Trockenheit schon). -- Cymothoa 18:16, 21. Jun. 2013 (CEST):Archivierung dieses Abschnittes wurde gewünscht von: Einleitung vergessen zu lesen. --Rôtkæppchen68 18:15, 21. Jun. 2013 (CEST)

Pflanze kommt zu Blüte = Ende der Pflanze?

Hi, ich wollte mal fragen, wenn es heißt "Die Pflanze kommt zur Blüte", ob das dann bedeutet die Pflanze lebt nachdem sie zur Blüte kam ab bzw. es zeichnet sich ein nicht mehr allzu langes leben danach ab? oder kann auch eine Pflanze die "zur Blüte kommt" noch lange leben?

Mein Ritterstern hat schon drei mal geblüht und ist immer noch nicht am Ende. Mache ich etwas falsch? --Rôtkæppchen68 22:32, 19. Jun. 2013 (CEST)
Das kommt ganz darauf an, ob es sich um Einjährige Pflanzen, Zweijährige Pflanzen oder Mehrjährige Pflanzen handelt. --78.34.19.11 22:37, 19. Jun. 2013 (CEST)
Und so etwas blüht ca. 5 bis 7 mal im Jahr. --Hans Haase (Diskussion) 22:42, 19. Jun. 2013 (CEST)
Während es mich bei meinem Bambus ziemlich ärgern würde. --Rudolph Buch (Diskussion) 22:44, 19. Jun. 2013 (CEST)
Pflanzen, die nur einmal blühen, werden als Hapaxanthe Pflanze bezeichnet; diejenigen, die mehrmals blühen, als Pollakanthe Pflanze. Der Ausdruck "Die Pflanze kommt zur Blüte" heißt aber schlicht, dass die Pflanzen Blüten ansetzt und zu blühen anfängt. Unabhängig davon, ob dies das einzige mal in ihrem Leben sein wird oder eines von vielen Malen. Griensteidl (Diskussion) 22:55, 19. Jun. 2013 (CEST)
Agaven--Giftzwerg 88 (Diskussion) 22:58, 19. Jun. 2013 (CEST)
Hallo Griensteidl, vielen Dank für die beiden Links - wegen Antworten wie Deiner hänge ich so gern in der Auskunft rum: Man lernt echt interessante neue Dinge und Begriffe :-) --Rudolph Buch (Diskussion) 23:16, 19. Jun. 2013 (CEST)

super vielen Dank (nicht signierter Beitrag von 134.93.54.72 (Diskussion) 23:20, 19. Jun. 2013 (CEST))

Wie heißt die Industrie/Technik, die Telefonanlagen so programmiert, dass man sich durch Nummerndrücken zu einem Bearbeiter durchmanövrieren muss?

...und gibt es dazu einen Artikel? Und warum erfolgt dies so selten mit Sprachunterstützung (Barrierefreiheit bei der Verwendung von Smartphonen "drücken sie die 1")?--Wikiseidank (Diskussion) 12:32, 19. Jun. 2013 (CEST)

Wenn es bei deinem Telefon kompliziert ist, eine Ziffer zu drücken, hast du ein schlechtes Telefon gekauft. Die machen das so, weil das Erreichen einer Person per Telefon durch Drücken von Ziffern seit Jahrzehnten bewährt ist. --Eike (Diskussion) 12:36, 19. Jun. 2013 (CEST)
Sprachdialogsystem ist gemeint. --Schaffnerlos (Diskussion) 13:02, 19. Jun. 2013 (CEST)
Ich weiß. Stattdessen kommen Telefonnummern (Durchwahlen) zum Einsatz. Find ich besser. Geht schneller, ist einfacher und zuverlässiger.
A propos: "Industrie" ist das falsche Wort, aber die "Technik" könnte man durchaus so nennen: Durchwahl.
--Eike (Diskussion) 13:13, 19. Jun. 2013 (CEST)
(BK) Ich glaube, ich hatte die Frage missverstanden. Es geht wohl um ein Dialogsystem (eben keine Durchwahl) mit Zifferneingabe. Die Antwort verändert sich dadurch nicht stark: Ziffern eingeben muss jedes Telefon leicht können (sonst ist es schlecht), und die Erkennung dieser Pieptöne auf der Gegenseite ist einfacher und zuverlässiger. --Eike (Diskussion) 13:23, 19. Jun. 2013 (CEST)
(BK) Auch bei nur-Touchscreen-Handys gehts üblicherweise leicht. Z.B. bei Android muss man zumindest bei meinem Samsung Galaxy S3 nur unten auf ein Symbol klicken, das ausschaut wie ein Tastenfeld, dann klappt das numerische Tastenfeld aus und man kann z.B. die 1 drücken. Und was die Durchwahlen angeht: das bedeutet aber für die Firma, die die Gespräche entgegennimmt mehr Arbeit, da viele die Durchwahlen nicht kennen udn daher bei der Klappe 0 (Vermittlung) anrufen. Außerdem kann es für die Kunden dadurch zu doppelten Wartezeiten kommen, daher zeurst wartet man bei der Vermittlung und dann beim Mitarbeiter, ist dem man verbunden wird. Meine Meinung: wenn es aus der Sicht des Anrufers relativ wenige Abteilungen (maximal 10) gibt ist das Sprachdialogsystem besser (und mMn ist es im Prinzip idiotensicher), wobei es gut ist, wenn man auch die entsprechende Nummer als Klappe wählen kann, bei mehr Abteilungen ist es besser, eine Vermittlung und die Klappen zu haben.--MrBurns (Diskussion) 13:21, 19. Jun. 2013 (CEST)
Nervigerweise passen die Fragen oft nicht wirklich zum eigenen Anliegen, wenn es da kein "sonstiges" gibt, ist man aufgeschmissen. Nervig ist auch, das ich das Telefon zum Nummerntippen von seinem sinnvollen Platz am Ohr wegnehmen muß und beim Zurückführen möglicherweise den Anfang der nächsten Ansage verpasse - es soll ja nun auch Leute geben, die etwas länger dafür brauchen, gerade ein Telefon kann man auch im hohen Alter noch nutzen. ABER: Eine Spracherkennung ist um so vieles schlimmer und unzuverlässiger; schon mit Erkältung faktisch nicht nutzbar. Barrierefreiheit ist keine Einbahnstraße, irgenwie bezweifle ich, das ein Blinder sinnvoll einen reinen Touchscreen nutzen kann - dazu braucht es dann ein Eingabegerät, in dem der Nutzer die gewünschte Taste finden kann.feba disk 02:40, 20. Jun. 2013 (CEST)
Sprachdialogsystem als Überbegriff für jede Art der Telefonnavigation trifft es, obwohl ich gerade nicht Sprachsteuerung meinte. Das von Feba beschriebene Problem meine ich. Auf dem Weg "weg vom Ohr - hin zum Ohr" werden Infos verpasst (noch mehr, wenn das Smartphone immer gleich in den Tastensperrenmodus wechselt). Und: vor dem Anruf weiß man nicht immer, ob man gleich mit einem Sprachdialogsystem verbunden wird.--Wikiseidank (Diskussion) 07:35, 20. Jun. 2013 (CEST)
@feba: üblicherweise gibt es heute eine kurze Pause, bevor die nächste Ansage kommt, wei man sich eben schon darauf eingestellt hat, dass die meisten mt dem handy anrufen und nicht einem altmodischen Festnetztelefon, wo der Hörer vom Ret getrennt ist. Da muss man mE schon extrem langsam sein, um etwas von der nächsten Durchsage zu verpassen, aber meistens ist das auch nicht schlimm, weil die Durchsage meist widerholt wird, wenn man nichts drückt. Und auch wenn amn seine Endstelle erreicht hat, gehts meist nicht zu schnell, weil die Person dort ja auch erst abheben muss. Was die Sprachsteuerung angeht: damit habe ich bisher immer sehr schlechte Erfahrungen gemacht, selbst mit mE guter Verbindung + klarer und lauter Aussprache wurden meine Antworten von diesen Systemen oft nicht erkannt, ich wurde dann erst nach einigen fehlgeschlagenen Versuchen mit der Vermittlung verbunden, die mich dann mit der richtigen Stelle verbunden hat, daher das ganze Spracherkennungssystem war nur Zeitverschwendung. Und es soll ja auch Systeme geben, die ganze Sätze erkennen wollen und nicht wie bei den Systemen, mit denen ich bisher Erfahrungen gemacht habe nur einzelne Wörter. Die funktionieren dann wahsrcheinlich noch schlechter. Und was die barrierefreiheit angeht: wenn jemand einen ausländischen Akzent hat oder nuschelt funktionieren diese Systeme wahrscheinlich überhaupt nicht (wobei de rAkzent auch bei mir das Problem sein könnte: es waren zwar österreichische Nummern, aber für Österrreich werden eventuell keine eigenen Systeme entwickelt, eventuell wurde meine Stimme also nicht erkannt, weil ich einen Wiener Akkzent habe und keinen norddeutschen. Solche Prfobleme gibts aber wohl in vielen Sprachen: bei englischen Muttersprachlern gibt es z.B. sehr viele unterschiedliche Akzente). --MrBurns (Diskussion) 11:13, 20. Jun. 2013 (CEST)
Versuchs mal mit Automatischer Vermittlung oder Vermittlungssystem --Hans Haase (Diskussion) 14:34, 21. Jun. 2013 (CEST)

Stimmung in Berlin heute

Ich weiß nicht ob es nur mir so ging als ich die Fernsehbilder gesehen habe, aber der Auftritt Obamas in Berlin war nun wirklich erbärmlich. Da gings ja zu wie auf einem Dorffest bei dem der MdL spricht. Weiß einer wie da die Akkreditierungsbestimmungen für Besucher waren? Waren das denn nur Claqueure?--Antemister (Diskussion) 22:14, 19. Jun. 2013 (CEST)

Ich habe keinerlei Video gesehen, sondern nur ein paar Fotos.
Aber ich würde mal vorschlagen:
- paar tausend Gäste einladen;
- diese zu halbwegs formaler Kleidung zwingen;
- sie bei schweißtreibenden Temperaturen unter die Sonne setzen, lange vorher aus Sicherheitsgründen (klar, mit Sonnenstich können sie dann nicht attentätern);
- keine Schirme, nicht Kaltes, nur Plastikbecher mit warmem Wasser ohne Kohlensäure;
- warten lassen...
- vorher tagelang über Presse und Rundfinck Kennedy beschwören, der nicht nur tatsächlich eine tolle Rede gehalten hatte, sondern diese auch in weniger als zehn Minuten, einschließlich Konsekutivübersetzung und Beifallsstürmen etlicher Hunderttausende, die nicht geladen, sondern einfach gekommen waren; Kennedy, der nicht nur in einer wirklich dramatischen Situation da war, sondern auch kurz darauf ermordet wurde, was die Mythenbildung erleichterte - da hatte Obama keine Chance;
- warten lassen...
- halbe Stunde Rede, gut, aber eben nicht Kennedy...
- sollte noch irgendwelche Stimmung vorhanden gewesen sein - Wowereit sichtbar plazieren; der wirkt (auf einem Foto) etwa wie der Hausmeister, der erstens sauer ist, dass er gerade heute arbeiten muss, zweitens den Kerl sowieso nicht ausstehen kann (Obama hat weniger Flughafen, aber mehr Publikumserfolg. Und die Familie kommt auch irgendwie besser rüber), und drittens nicht ganz sicher ist, ob der Typ eigentlich wirklich da auf dem Pariser Platz reden darf, ohne grüne Abgasplakette und so; linkisch wirkt er inzwischen immer - letzte Stimmung ist weg...
Kurz und gut, lieber Wilhelm - Reden von US-Präsidenten in Berlin sind nicht nur nicht mehr, was sie mal waren. Sie sind auch kein wirkliches Neuland. Chantk (Diskussion) 23:06, 19. Jun. 2013 (CEST)
"Hausmeister Wowereit" - das ist gut... Ich dachte zunächst, das ist der vom Protokoll, der die Sitzplätze zuweist.--Antemister (Diskussion) 23:27, 19. Jun. 2013 (CEST)
Nicht, dass wir uns missverstehen. Es gibt auch gute Hausmeister.
Die Sache mit dem Protokoll - das Problem ist wie immer, wenn jemandes Ehrgeiz und Fähigkeiten irgendwie gar nicht zusammenpassen. Und wenn der Hausmeister glaubt, Protokollchef spielen zu dürfen, obwohl er das partout nicht kann - ja, dann sieht das so peinlich aus, dass selbst der Lach-Erfolg so aufregend ist wie neunundneunzig büroschlafende BeamtInnen. Chantk (Diskussion) 00:31, 20. Jun. 2013 (CEST)
Die Gäste waren handverlesen (z. B. Schüler der deutsch-amerikanischen John-F.-Kennedy-Schule). --Komischn (Diskussion) 12:20, 20. Jun. 2013 (CEST)
Natürlich war die Stimmung erbärmlich. Beim letzten Besuch waren Hunderttausende da, weil der Obama so schön "Yes we can" sagen konnte. Nach einer Amtszeit haben halt auch die Letzten in Deutschland begriffen, dass ein amerikanischer Präsi nicht plötzlich eine komplett andere amerikanische Politik macht, nur weil er eine andere Hautfarbe hat. Jetzt sind alle vom Obama enttäuscht und schämen sich wegen Ihrer rassistischen positiven Diskriminierung und keiner will Ihn mehr sehen. Diese Entwicklung war aber schon vor fünf Jahren abzusehen, als Obama vor deutschem Publikum das erste mal ywc gesagt hat und die verwirrte Masse ihm zugejubelt hat. Ich fürchte mich vor dem Tag, an dem mal einen deutschen Politiker ein cooler Spruch einfällt. --JaMitKeksBitte (Diskussion) 12:50, 20. Jun. 2013 (CEST)
So ein Häufchen handverlesener auf einem großen Platz sieht wohl immer erbärmlich aus. Das Netrebko-Konzert mit angeblich "zehntausenden" Zuschauern auf dem Roten Platz imn Moskau am gleichen Abend auf Arte sah auch aus wie ein Dorfmusikfest, nur das es bei solchen nicht so viele leere Stühle gibt... --AndreasPraefcke (Diskussion) 15:26, 20. Jun. 2013 (CEST)
"Dorffest" wäre eh eine Verbesserung gegenüber Obamas letztem Besuch; damals war es ja nur ein "Treffen der Wohngemeinschaft Wilhelmstraße 48 [8]" gewesen. --Stubfighther (Diskussion) 20:17, 20. Jun. 2013 (CEST)

Taubenverteilung in Ankara

Auf diesem Bild sind die Tauben ähnlich gleichmäßig verteilt wie die Menschen dahinter (Güven-Park, Ankara, wohl gestern). Wie kriegt man das hin? Ernst gemeinte Frage, normalerweise knubbeln Tauben doch ständig. Gibt's eine Fototechnik, die die immergleichen Tauben in Intervallen aufnimmt? Aber dann müssten die Menschen verwackelter sein. Oder sind die Tauben von den Menschen so irritiert, dass sie sich auseinanderziehen? Viele scheinen ja auch eher zu stehen als zu schreiten. Irgendwas stimmt da nicht, oder wenigstens ist da was ungewöhnlich. Aber was? --Aalfons (Diskussion) 13:32, 20. Jun. 2013 (CEST)

Spontan hätt ich gesagt, die haben denen was zu essen hingeworfen, aber erkennen kann ich davon nichts. --Eike (Diskussion) 14:00, 20. Jun. 2013 (CEST)
Das ist Ankara im Sommer. Es ist *heiß*. Vögel können nicht schwitzen. Es gibt drei Dinge, die eine Taube dazu bringen könnten, sich in dieser Situation zu bewegen: Feind, Futter, Gefühl, sich baden zu müssen. Sonst nix. Scheint alles nicht verhanden zu sein. -- Janka (Diskussion) 14:02, 20. Jun. 2013 (CEST)
Sie knubbeln dann, wenn da ein Sandwich liegt. Entspannte Zufriedenheit/Ruhe ergibt sich da, wo die Taube unter ihresgleichen ist und genügend Raum zum gleich wieder Abheben hat.
Einen ganz ähnlichen Abstand halten sie auch beim Fliegen - und man weiss mittlereweile auch, wie sie ihre Position in der Masse der anderen Tauben "koordinieren". Scheissviecher, die ... GEEZER... nil nisi bene 14:18, 20. Jun. 2013 (CEST)
Siehe auch Individualdistanz. -- Density Disk. 16:49, 20. Jun. 2013 (CEST)
Dolle. Die Fotos und die Begründung sind überzeugen. Dann müssen sich die Ratten der Lüfte bei den Demonstranten entspannt fühlen. --Aalfons (Diskussion) 16:55, 20. Jun. 2013 (CEST)
Das Bild ist auch in der Hinsicht interessant (vielleicht ist DAS ja das Ungewöhnliche), weil die Menschen endlich mal stille stehen - auf Linie. Das machen dann auch die Tauben - und das lässt den Toleranzruheabstand Taube => Mensch (nicht Mensch => Taube; Menschen wollen ja immer alles befingern...) abschätzen. Nur eine Taube, Chuck, traut sich näher heran. Es könnte sein, dass sie bei den beiden Damen etwas Essbares vermutet.
Auch hier (zirkulär): "Vorsicht, Männer! Abstand! Er hat Lauerstellung eingenommen und die Plüschkühe sind nicht essbar... sagt Chuck!" GEEZER... nil nisi bene 18:13, 20. Jun. 2013 (CEST)
Leittauben, da müsste das Vergrämen doch einfacher sein. --Aalfons (Diskussion) 19:08, 20. Jun. 2013 (CEST)

Wer ist Graf Fito?

Hier haben wir einige Zitate von einem gewissen "Graf Fito". Aber wer ist Graf Fito? Ich habe ihn gegooglet, es gibt keinen WP-Artikel über ihn (nicht mal auf en-WP), ich fand nichts bei Wolfram Alpha oder im dnb, es gibt kein Eintrag auf Wikiquote oder Stupidedia, ich habe es mit "Lord Fito" oder "Graffito" versucht... Entweder er existiert nicht oder stehe mächtig auf dem Schlauch...--84.149.162.133 15:13, 20. Jun. 2013 (CEST)

Ich glaube Klaus Klages schreibt unter dem Pseudonym. --Dos Vientos (Diskussion) 15:19, 20. Jun. 2013 (CEST)
+1 © Graf Fito (*1938), eigentlich Klaus Klages, deutscher Gebrauchsphilosoph und Abreißkalenderverleger. GEEZER... nil nisi bene 15:20, 20. Jun. 2013 (CEST)

Graf Fito Klaus Klages - falls wir mal den Beleg brauchen. --Eingangskontrolle (Diskussion) 13:11, 21. Jun. 2013 (CEST)

Mathematische Funktion zu einer Grafik gesucht

Gibt es zur Datei:Valuefun.jpg, verwendet in der enlg. Wikipedia, eine Funktion, die diese Grafik beschreibt? Die dort enthaltene Funktion U ist es nicht.--141.76.111.103 16:04, 20. Jun. 2013 (CEST)

Sicher gibt es eine Funktion, die diese Grafik beschreibt: man nimmt für jeden Punkt auf der x-Achse den entsprechenden Abstand (in Pixel) des punktes der Grafik von der y-Achse. Zusätzlich gibt es wegend er beschränkten Auflösung der Grafik auch unendlich viele Funktionen, darunter auch unendlich viele analytische Funktionen, die Werte liefern, die mit der Grafik übereinstimmen. --MrBurns (Diskussion) 16:15, 20. Jun. 2013 (CEST)
Doch, für eine unendliche Zahl von Funktionen der Art v(x) und w(x) ist genau das vermutlich die Grafik, bloß kennst Du weder v noch w. Es sieht aber eher so aus, als sei die Grafik gezeichnet oder mittels einer esoterischen Lowess-Regression durch eine Datenpunktewolke entstanden. fossa net ?! 17:19, 20. Jun. 2013 (CEST)
Das Bild sieht nach nem' freihändig gezeichnetem Arkustangens aus, oder eine geglättete Sprungfunktion.--Svebert (Diskussion) 22:27, 20. Jun. 2013 (CEST)
Eher nicht, das entscheidende an der Funktion ist ja gerade die Asymmetrie, d.h. dass die auf der negativen Seite steiler ist als auf der positiven.[9] Ob es da irgendeine konkrete mathematische Darstellung gibt, kann ich aber nicht sagen. -- Jonathan 23:03, 20. Jun. 2013 (CEST)
Für mich sieht das aus wie ein Funktionsgraph, den der Autor in irgendeiner Economics-Vorlesung von der Tafel abgemalt hat. Der Dozent hat sie garantiert auch freihand gezeichnet. Im Zweifalsfall würde ich beim Urheber des Schaubilds, en:User:Rieger nachfragen. --Rôtkæppchen68 23:22, 20. Jun. 2013 (CEST)
+ 1. Als Bewertungsfunktion wird hier angesetzt: v(x) = x^α für x > 0, v(x) = -λ(-x^α) für x < 0 (mit α = 0.88 und λ = 2.25). Diese hat den Graph [10]. --Pp.paul.4 (Diskussion) 23:31, 20. Jun. 2013 (CEST)
In der BWL werden gerne Terme benutzt, die wie mathematische Gleichungen aussehen, bei denen jedoch weder die Funktionen noch die Randbedingungen genau bekannt sind. Dafür bekommt man vielleicht den Nobelpreis, aber ausrechnen lässt sich damit nichts. --Optimum (Diskussion) 08:47, 21. Jun. 2013 (CEST)

Tierschutz bei Ungezieferbekämpfung

Wo wird geregelt, welche Methoden zur Ungezieferbekämpfung zulässig sind und welche nicht? Ob beispielsweise eine neu erfundene Methode verwendet werden darf. 46.115.124.224 18:24, 20. Jun. 2013 (CEST)

Welche Art "Ungeziefer"? 6-beiniges, 4-beiniges? GEEZER... nil nisi bene 18:28, 20. Jun. 2013 (CEST)
Z.B. 6-beiniges staatenbildendes Ungeziefer oder auch nesterbauende Langschwanznager, beide nicht unter Naturschutzt oder so. .46.115.124.224 18:57, 20. Jun. 2013 (CEST)
Eben doch. Tierschutz gilt gesetzlich für Wirbeltiere. Darunter fallen auch die Kanalratten, egal welcher Schwanzlängenkategorie sie angehören. Für sämtliche Invertebraten gilt er aber nicht. Chantk (Diskussion) 20:09, 20. Jun. 2013 (CEST)
+0,5: § 1 Tierschutzgesetz gilt für alle Tiere, die hier weiters einschlägigen §§ 4 und 13 (1) beziehen sich dann allerdings tatsächlich nur auf Wirbeltiere. Je nach Methode könnten auch Biozid-Richtline bzw. Chemikaliengesetz relevant sein, siehe dazu hier ausführlich bei der zuständigen Bundesanstalt für Arbeitsschutz und Arbeitsmedizin. --Rudolph Buch (Diskussion) 20:20, 20. Jun. 2013 (CEST)

Nachrichtensperre nach 911?

Mir wurde vor einiger Zeít mal erzählt, am 11. September habe es nach dem dem Einschlag des zweiten Flugzeugs in Deutschland eine Art "Nachrichtensperre" gegeben. Neuere Entwicklungen sollen da da eine Zeit lang nicht berichtet worden sein. Kann sich jemand daran erinnern? (Selber habe ich es damals zu spär erfahren). Denkbar erscheint das, nachdem zu dieser Zeit online-Medien, vor allem aber Web 2.0, noch nicht so verbreitet waren.--Antemister (Diskussion) 20:37, 20. Jun. 2013 (CEST)

Ich habe damals zwischen n-tv, CNN, ARD und anderen europäischen und US-Kanälen (Satellit, nicht on line) hin- und hergewechselt und kann mich nicht an wesentliche Unterschiede im Videomaterial und in den Informationen erinnern. Die kommentierenden Journalisten hatten anfangs unterschiedliche Meinungen, aber das ist nur verständlich. Eine Nachrichtensperre hätte keinen Sinn gehabt. Chantk (Diskussion) 21:12, 20. Jun. 2013 (CEST)
Nachrichtensperre ist Unfug, CNN war doch ununterbrochen auf Sendung. Ich war damals selbst Redakteur bei einer Tageszeitung, die Agenturmeldungen kamen fortlaufend rein. Nur war es wie immer in solchen Situationen so, dass es auch ein paar ungeprüfte Aussagen gab, zum Beispiel von Schießereien oder explodierenden Autos in Washington. Kann sein, dass mal jemand gesagt hat, dass es darüber keine weitere Berichterstattung gebe (weil der Anlass nämlich nicht stimmte), und unsere Freunde von der Verschwörungsfront machen da gleich wieder ein Gewese drum. Aber interessant, dass du dir die schnelle Verbreitung solcher Nachrichten ohne 2.0 nicht vorstellen kannst. --Aalfons (Diskussion) 21:28, 20. Jun. 2013 (CEST)
Was es gegeben hat, war ein Meinungsumschwung. Am Anfang wussten die verschiedenen Stationen nichts bis wenig und haben in alle möglichen Richtungen ihre Vermutungen frei von der Leber weg live auf Sendung erzählt. Ab einem gewissen Zeitpunkt, europäischer Zeit am Abend, US-Ostküstenzeit früher Nachmittag, haben dann die US-Behörden offizielle Statements über die Agenturen geschickt und von da an war da so ziemlich einheitliche Berichterstattung auf allen Kanälen. Ab dann war nur mehr von entführten Flugzeugen die Rede, die ersten Namen von Entführern sind genannt worden, die Pennsylvania-Geschichte und die Pentagon-Geschichte wurde zusätzlich bekannt. Und so weit ich mich erinnern kann, sind noch in der selben Nacht die ersten Marschflugkörper nach Afghanistan losgeschickt worden. Da bin ich mir aber nicht mehr ganz sicher. Unser Artikel Krieg in Afghanistan seit 2001 sagt eher, dass es erst nach der UN-Resolution zwei Wochen später dazu kam, obwohl so genau steht es da nicht. Ich war jedenfalls überrascht, wie schnell der "Gegenschlag" losging. --El bes (Diskussion) 21:42, 20. Jun. 2013 (CEST)

Es geht darum, das deutsche Sender so etwa eine gute halbe Stunde nach dem zweiten Einschlag nichts neues berichtet hätten. Nicht jeder schaut da CNN. Auf Twitter etwa hätte sich das doch wie ein Lauffeuer verbreitet. Aber damals gabs das noch nicht, auch keine Smartphones, mit denen man immer und überall online sein kann. Aber auch die Verwirrung, und die zahlreichen Gerüchte sind da eine plausiblere Erklärung. Und, nein, die Person kommt von weit außerhalb der Verschwörerszene.--Antemister (Diskussion) 21:53, 20. Jun. 2013 (CEST)

Zu El Bes: Ich habe mir das noch mal im Ticker angeschaut. Der erste Einschlag wurde bei dpa um 14:57 Uhr, der zweite Einschlag um 15:06 unserer Zeit gemeldet. afp berichtete um 15:30, die US-Regierung gehe von einem Anschlag aus. dpa zitiert um 15:43 Bush umschreibend mit "wahrscheinlicher Terrorakt". Um 15:47 berichtet reuters, bei Abu Dhabi TV habe sich die DFLP zu dem Anschlag bekannt, um 16:02 via reuters von der DFLP dementiert. AP berichtet um 17:04 von "Spekulationen über Osama bin Laden als Drahtzieher des Terrors". ((Kann nicht weiter kompilieren, muss jetzt wegen Gewittern vom Netz)). Fortsetzung vllt morgen... --Aalfons (Diskussion) 22:13, 20. Jun. 2013 (CEST)
Im South Tower war damals auch die Telekommunikationsfirma Verizon auf einigen Stockwerken eingemietet, die dort einiges an Internet-Backbone stehen hatte. Das Hauptquartier war gleich daneben im Verizon Building, das ebenfalls stark beschädigt wurde. Beim Einsturz ist auf einen Schlag eine beträchtliche Bandbreite im Internet eingebrochen und das gesamte Verizon-Handynetz war plötzlich offline. Auch Verizon-Kunden in Europa konnten nicht mehr telefonieren. Vielleicht war das mit ein Grund, warum danach die Infos einen "Lag" hatten, bis alternative Leitungen aufgebaut, bzw. freigeschalten waren. --El bes (Diskussion) 22:19, 20. Jun. 2013 (CEST)

Worin besteht denn der fundamentale qualitative Sprung, ob man das Ereignis live im Fernsehen oder Web verfolgt oder mit sechs Stunden Verzögerung in einer zusammenfassenden Nachrichtensendung erfährt? Mal so als Beispiel in einem Vergleich von 911 und dem 26./27. Oktober 1962 (Höhepunkt/Ende der Kubakrise)? Ich kann (von den unmittelbar Betroffenen mal abgesehen) den für ein besseres Leben notwendigen Unterschied nicht erkennen (außer in der unmittelbaren Befreidigung von Sensationsgier per Stream tausende von Meilen entfernt von diesem Ereignis). --84.191.135.38 00:51, 21. Jun. 2013 (CEST)

Es geht nicht darum, aus Sensationsgier möglichst in Echt-Zeit über jedes Detail informiert zu sein, sondern um die unterschwellig formulierte Vermutung, dass in den ersten Stunden die Berichterstattung noch ziemlich chaotisch, aber auch pluralistisch und frei war und ab einem gewissen Zeitpunkt alle TV-Sender und Nachrichtenagenturen "gleichgeschalten" waren und nur mehr die US-Regierungslinie berichtet haben. Das obwohl bei anderen Verbrechen die Behörden aus ermittlungstechnischen Gründen oft sehr mit Information geizen. Jener Personen, die nicht an die offizielle Verschwörungstheorie glauben (eine Verschwörung war es ja in jeden Fall), äußern auch immer wieder den Verdacht, dass das Material der Fernseh-Archive im Nachhinein manipuliert oder geschnitten wurden. Die ersten Stunden der Berichterstattung waren somit die interessantesten, wenn man das in Anbetracht dieser Tragödie überhaupt so sagen darf. Bezüglich der Kuba-Krise haben wir übrigens erst 30 Jahre später die ganze Wahrheit erfahren, nämlich dass es dabei auch um die US-Mittelstreckenraketen in der Türkei gegangen ist, die Kennedy dann abziehen ließ. --El bes (Diskussion) 01:26, 21. Jun. 2013 (CEST)
Nun, es gibt (noch immer) eine sehr starke Bewegung, die der offiziellen 911-Geschichte mißtraut und alles sehr akribisch hin und her wendet. Wenn Material manipuliert wurde, würde bei einem seriösen Nachweis sicher ordentlich darauf rumgeritten und wir hätten schon längst davon erfahren. Wenn es den Nachweis nicht gibt, ist das Spekulieren eher müßig. Mir persönlich (um mich da zu positionieren) ist das eigentlich auch völlig wurscht. Viel wichtiger ist doch, daß so viele Leute ihrer Regierung oder bestimmten politischen Seilschaften das Allerschrecklichste zutrauen, daß das Undenkbare denkbar ist, daß es da (sagen wir mal im Vergleich zu 1962) viel viel weniger Vertrauen gibt. Das drückt ja auch was aus, wenn kaum jemand seiner Regierung noch glaubt. Die berühmte "Schere im Kopf", also die vorauseilende eigeninitiative, oft flächendeckende Selbstzensur ist ein uraltes Phänomen und Problem der Presse. Da muß keiner kommen und manipulieren und schneiden, das machen die schon von selbst. Man denke an die Berichterstattung während der Schleyer-Entführung als Beispiel oder bei dem Anschlag auf den Boston-Marathon, da hat sich auch keiner getraut, mal darauf hinzuweisen, daß solche und noch viel schrecklichere Anschläge alle zwei Wochen im Irak stattfinden und daß deshalb noch lange nicht tagelang sondergesendet wird. Aber auch, wenn wir nicht alles wissen, kann man "bei uns" noch öffentlich fragen, was zum Beispiel die US- und Bundesregierung denn in Sachen Murat Kurnaz vereinbart haben oder wer in der deutschen Regierung wußte, wieviel Geheimflüge mit von US-Geheimdiensten Verschleppten über das Bundesgebiet genehmigt und gedeckt wurden, wenn die zu ihren "Verhör"kerkern in Osteuropa unterwegs waren. Wir werden wohl nie alles erfahren, aber einiges sickert eben durch in die Öffentlichkeit und man kann öffentlich fragen. Und in 911 sind wirklich sehr viele Leute involviert und einige reden ja auch Sachen, die mit der offiziellen Version nicht übereinstimmen. Und deshalb kann man vom "gleichgeschaltet" nicht reden. Das ist eine schlimme Erfindung der Nazizeit und zwischen "gleichgeschaltet" und einer extrem unfreien Presse, wie wir sie in z.B. Weißrusssland, dem Iran oder Saudi-Arabien (um bei 911 zu bleiben) erleben, sehe ich noch einen Unterschied. Einzig bei Nordkorea würde ich von "gleichgeschaltet" reden und in den USA gibt es sicher jede Menge Probleme und die Gegenöffentlichkeit mußte, was die landesweite Reichweite angeht, fast völlig auf das Internet ausweichen, aber es gibt trotz schlechter Platzierung auf Rang 32 im Press Freedom Index (hinter Ghana und Surinam) dennoch genug Raum, um auch im Land der Freien und der Heimat der Tapferen "dumme" Fragen zu 911 zu stellen - und auch zu verbreiten. Die NSA macht dann den Eckermann und hebt alles schön für die Nachwelt auf. :-) --84.191.135.38 02:28, 21. Jun. 2013 (CEST)
Es gab am 9.11. eine ganze Reihe Fehlinformation. So wurde lange Zeit berichtet, dass es neben den Flugzeuganschlaegen in Washington auch noch Autobomben gegeben habe. Daher ist es plausibel, dass die Nachrichtensender etwas vorsichtiger geworden sein koennten und nicht mehr alles gleich brandheiss servierten. Ein weiterer Aspekt, an den ich mich erinnere, war, dass am 9.11. in den Nachrichten Bilder von Menschen gezeigt wurden, die aus den brennenden oberen Stockwerken herabfielen. Diese Bilder wurden dann spaeter nicht mehr gezeigt. -- Arcimboldo (Diskussion) 03:28, 21. Jun. 2013 (CEST)
Die springenden Menschen sind ein interessantes Beispiel. Als etwas älteres Semester bin ich mir bewusst, dass weder schriftliche Berichterstattung, noch Bilder und auch nicht Augenzeugenberichte die völlig objektive allumfassende Wahrheit darstellen. Die Intenet- und Handygeneration sieht das scheinbar teilweise anders. Ich persönlich habe kein Bedürfnis, Menschen per Fernsehen beim Sterben zuzusehen. Auch wenn das Ereignis schon fast 12 Jahre zurückliegt. Es ist eine Frage der Pietät und wie man die Grenze zwischen berechtigtem Informationsbedürfnis und Sensationsgier zieht. Für mich ist es ausreichend, dass mir bewusst ist, dass es diese Verzweiflungstaten gab. Guter Journalismus zeichnet sich auch dadurch aus, dass eingehende Informationen - soweit mögllich - Wahrheit geprüft und bewertet werden. Darauf könnte die vermeintlich beobachtete Nachrichtenverzögerung zurückzuführen sein. --84.152.48.222 06:50, 21. Jun. 2013 (CEST)
So etwas wie eine Nachrichtensperre gab es definitiv nicht. z.B. Spon hat mehr oder weniger im Minutentakt jedes neue Gerücht getickert das irgendwie aufkam. Auf N24 lief das genauso. Das Problem war eher das jedes Nachrichtenmedium jeden Informationsfetzen sofort veröffentlichte und man am Ende des Tages nicht so richtig wusste was jetzt eigentlich alles passiert war. Das Web 2.0 wird bei sowas komplett überschätzt. Da sowieso jeder jeden über einige Personen kennt, hat sich die Nachricht auch damals nahezu in Echtzeit verbreitet. Den ersten Einschlag habe ich Minuten später über SMS erfahren und die weitere Entwicklung wurde über Messenger oder Email bzw. auf Foren diskutiert. Der große Unterschied zu heute war nicht, dass es kein Facebook und Twitter gab, sondern der das heute jedes Handy eine Kamera dabei hat. Das ist die tatsächliche nachrichten-technische Revolution. Früher gab es fast nur Bilder die Journalisten erstellt und veröffentlicht haben, heute gibt es solche Ereignisse aus 1000 verschiedenen Perspektiven. Und zu Twitter und Facebook. Wann hast du denn deine letzte wichtige Nachricht über eins dieser Medien erhalten? :) --85.181.220.49 06:52, 21. Jun. 2013 (CEST)
Ja, es gab partiell eine Nachrichtensperre im Pentagon. Da sollten die Schäden nicht nachvollziehbar sein. Da ist bis heute nicht alles bekannt. Ansonsten waren lange das Internet und das Mobilnetz nach USA wegen Überlastung zusammengebrochen. Gerade in NY saß der Schrecken tief, da war man vordringlich mit den Rettungsarbeiten beschäftigt, Information war nachrangig, aber eine Nachrichtensperre gab es nicht.--79.232.217.224 07:39, 21. Jun. 2013 (CEST)
Naja, ganz so das Journalisten mit Rettungsarbeiten beschäftigt gewesen wären war es nicht :) Das Gebiet um die Türme war nur relativ schnell weiträumig gesperrt und der Luftraum war ebenfalls 8 KM um die Türme herum gesperrt. Trotzdem war jeder Nachrichtencopter in der Luft. Direkt an diesem Tag hat man hauptsächlich Fernaufnahmen der Türme gesehen und ein zwei Videos von Journalisten die zufällig in der Nähe der Türme waren. Vom Pentagon hat man wenn ich mich recht erinnere längere Zeit berichtet das eine Autobombe davor explodiert wäre. Übrigens, oben wird behauptet das noch am selben Tage eine Cruise Missile losgeschickt worden währe. Das stimmt so nicht. Genaugenommen war es verblüffend wie lange es danach gedauert hat, bis Amerika irgendjemand angegriffen hat. Nur das es der Osama war, das wurde glaube ich schon ne Stunde nach dem zweiten Einschlag durch die Medien geschleift. Wie man das so schnell raus gefunden hat frage ich mich bis heute. --85.181.220.49 08:23, 21. Jun. 2013 (CEST)
Also das war nun schon damals keine allzu anspruchsvolle geistige Transferleistung, auf bin Ladin zu kommen. All Qaeda war schon recht aktive, hatte kurz zuvor wohl im Jemen zugeschlagen und in verschiedenen Teilen der Welt Einrichtungen der USA und ihrer Alliierten angegriffen. Aufs WTC hatte ebendiese Organisation schon einige Jahre vorher einen Bombenanschlag veruebt. Relativ schnell fand man dann unter den Passagiernamen auch Leute wie Mohammed Atta und ihre Verbindungen nach Afghanistan, die Hamburger Zelle war wohl bereits einen Tag nach dem Anschlag in den Medien namentlich bekannt. -- Arcimboldo (Diskussion) 08:36, 21. Jun. 2013 (CEST)

Hier liegt ein manifester Irrtum vor, der sich durch alle Edit in diesem Thread zieht. „Nachrichtensperre“ bedeutet nicht, daß keine Nachrichten mehr kommen. Das war niemals zu verhindern, es hatte niemand ein Interesse daran, es zu verhindern. „Nachrichtensperre“ bedeutet nur und lediglich, daß offizielle Stellen zunächst keine Erklärungen abgeben. Ob es eine solche gab, weiß ich nicht - vernünftig wäre es gewesen. Wenn beispielsweise der für Feuerwehren zuständige Verwaltungschef von Washington D.C., der vollauf mit dem Einschlag des Flugzeugs ins Pentagon befaßt war, dazu eine Pressekonferenz gegeben hätte, ohne von den beiden Anschlägen in New York zu wissen, dann hätte er sich bestenfalls blamiert. Eine Nachrichtensperre ist also ein behördeninterner Vorgang, der für keinen Journalisten irgendeine Auswirkung hat - außer der, daß er von offiziellen Stellen nichts zu hören bekommt. --Freud DISK Konservativ 13:30, 21. Jun. 2013 (CEST)

Unterschriften von fiktiven Personen

Guten Abend. Mir ist heute etwas sehr unangenehmes widerfahren. Ich schreibe gerade an einem Buch (Roman) und möchte auf die Titelseite die Unterschrift der Hauptperson setzen. Die Hauptperson ist eine fiktive Person, also von mir für das Buch erfunden. Da wir in meiner Ausbildungsstätte auch problemlos mal etwas privat einscannen durfen (ausdrücklich im Maßen erlaubt) habe ich mir kurz vor Feierabend ein Blatt Papier geschnappt und darauf die Unterschirft der Person "geübt" (amerikanischer Name) um das dann einzuscannen. Das hat mein Chaf gesehen und fragte ob ich Unterschriften fälschen würde. Ich hab das zwar verneint, aber kann das irgendwelche rechtlichen Konsequenzen oder zumindest Probleme zur Folge haben? Ich meine es gibt die Person ja nicht und außerdem ist ja kein Dokument signiert. --78.52.120.125 21:23, 20. Jun. 2013 (CEST)

(IANAL) Fiktive Unterschriften, sollten soweit das eindeutig "Kunst" ist, kein Problem darstellen. Problem könnte werden, wenn es eine Person diesen Namens wirklich gibt, und die Unterschrfit auch noch ähnlich erscheint.--Antemister (Diskussion) 21:28, 20. Jun. 2013 (CEST)
Wenn man einien gängigen Vor- und Nachname nimmt gibt es die Person immer, also Clark Douglas in meinem Fall. Es garantiert Menschen die so heißen, aber ich kenne die natürlich nicht und weiß auch nicht wie deren Signatur aussieht. Zumal es ja nur auf einem Schmierblatt war. --78.52.120.125 21:36, 20. Jun. 2013 (CEST)
Wenn ich du wäre würde ich das Buch (Roman) mit zusätzlichen Kommas anreichern weil vor allem ältere Leser das oft erwarten.
Dass der Chaf ausnahmsweise die Verwendung technischer Geräte für private Zwecke gestattet, heißt nicht, dass er auch private Schreibübungen in der Arbeitszeit toll findet.
Das Titelbild vor der Fertigstellung des Textes auszuarbeiten, ist etwa wie das Grillen des Bärenfleisches (ich verweise auf unlängst hier diskutierte Fragen zum Thema "Frisst mich der Bär oder fresse ich ihn, und wenn ja, in Norwegen?") vor dem Erlegen von Meister Petz. Chantk (Diskussion) 21:35, 20. Jun. 2013 (CEST)

Rechtlich duerftest Du, wenn es so ist wie beschrieben, nichts zu befuerchten haben. Dein Hauptproblem ist eher der Eindruck, den der Chef hat. Wenn die Zusammenarbeit generell vertrauensvoll ist, dann sollte es nicht viel ausmachen. Wenn es aber eher von Misstrauen gepraegt ist, dann koennen solche Ereignisse Misstrauen noch verstaerken, denn es ist eine Sache, dass Du nichts Widerrechtliches getan hast, eine andere Sache, ob der Chef Dir glaubt, dass Du unter die Schriftsteller zu gehen versuchst (wenn er vorher davon noch nichts gehoert hat). Das haengt natuerlich auch davon ab, ob Du in einem Kunstverlag taetig bist (da ist es eher plausibel) oder vielleicht als Kfz-Mechaniker, wo literarische Ambitionen doch etwas seltener sind. -- Arcimboldo (Diskussion) 04:45, 21. Jun. 2013 (CEST)

Wie werden Pseudonyme eigentlich aufgedeckt?

Immer wieder lese ich, dass z. B. Fechner unter dem Pseudonym "Dr. Mises" schrieb. Auch wird immer wieder erwähnt, dieser oder jener Autor habe anonym eine Schrift publiziert. Wie kann man solche Werke posthum einem Urheber zuordnen? Findet man da Hinweise im Nachlass? Enden mit dem Tod Schweigevereinbarungen zwischen Verleger und Autor oder reden eingeweihte? Oder macht man das wirklich über Analyse des Wortschatzes, des Satzbaus usw, ist diese Methode so zuverlässig? --188.101.88.2 23:51, 20. Jun. 2013 (CEST)

Vermutlich durch Spekulation oder Offenbarung. – Cherryx sprich! 23:52, 20. Jun. 2013 (CEST)
Viele Wege führen zum Ziel. Eine Textanalsye ohne weitere Beweise bzw. Nachweise oder Spuren reicht aber meist nur für Spekulationen aus und nicht für eine sichere Zuordnung. Das wird dann aber auch normalerweise als Vermutung benannt und begründet. Weil die Umgangsweisen mit solchen Fragen in der Wissenschaft sorgfältiger seriöser sind als im Web 2.0 sind die Nachweise und Zeugnisse (von welchem Weg auch immer) in der Regel publiziert, nachprüfbar und öffentlich diskutierbar. Es gibt aber auch Fälle mit einem Rest Unsicherheit. B. Traven ist ein spannendes Beispiel. Hier geht man zwar überwiegend davon aus, daß Rolf Recknagel mit den Ergebnissen seiner stilistischen Untersuchung richtig liegt, aber wer seriös über die Identität von B. Traven und Ret Marut spricht, wird auf den dennoch bestehenden winzigen Rest an Ungewißheit hinweisen. --84.191.135.38 00:32, 21. Jun. 2013 (CEST)
Schriftsteller, Musiker und Schauspieler der Gegenwart haben ja einen Bekanntenkreis, der Realnamen und Pseudonym kennt. So dürften viele Pseudonyme in der Liste von Pseudonymen bekannt geworden sein, ähnlich wie die schon seit Jahrhunderten zusammengetragenen Pseudonymen-Lexika. Wenn das Pseudonym wirklich verborgen bleiben soll, sind detektivische Methoden erforderlich, um Hinweise zu finden. --Pp.paul.4 (Diskussion) 00:36, 21. Jun. 2013 (CEST)
Aus der Einleitung des Dictionary of Pseudonyms (ab S. 3) geht hervor, dass die meisten der Pseudonyme gewählt wurden, weil der eigene Name uncool erschien (ich habe gerade einen in die obige Liste nachgetragen, der zu jedem Thema, über das er schrieb, einen "geeigneten" Namen wählte - auch mal den des anderen Geschlechts). Decknamen von Spionen oder Verbindungsleuten sind irgendwo dokumentiert und kommen meist nach Jahren ans Licht. Autoren wählen auch of Namen, die sich durch Übersetzungen, Anagramme oder anderes erklären lassen. Und dann gibt es noch den Ansatz, für welche Person die Wahrscheinlichkeit für ein bestimmtes Pseudonym am höchsten ist. Da werden verschiedene Ansätze kombiniert und man sieht nach, auf wen die meisten hinweisen.
Selbst amerikanische Generäle haben es mittlerweile schwer, ihr Pseudonym geheim zu halten... ;-) GEEZER... nil nisi bene 07:51, 21. Jun. 2013 (CEST)
... und der Bambi-Autor Felix Salten hat seine "Lebenserinnerungen" angeblich auch unter einem Pseudonym verfasst - sich aber wohl nicht dazu bekannt. ;-) -- Aerocat 13:27, 21. Jun. 2013 (CEST)
Es sind vielleicht nicht die Fälle, die dich in erster Linie interessieren, aber oft geht es bei Pseudonymen einfach nur darum, dass nicht sofort ins Auge fällt, was alles vom selben Autor stammt. Echte Geheimhaltung spielt dabei keine große Rolle. Berufsschriftsteller, die hauptberuflich davon leben wollen aber nicht zum sehr kleinen Kreis der glücklichen Superstars gehören, schreiben oft ganz verschiedene Dinge für verschiedene Verlage und Zielgruppen, ohne Rücksicht auf ein irgendwie stimmiges Gesamtwerk. Da will es die Leserschaft gar nicht so genau wissen, wenn "Ferien auf dem Ponyhof", "Delta Force Ninja" und "Werwolf-Bondage" alle aus der selben Feder stammen. Das hält einen Schriftsteller aber nicht davon ab, in geschäftlichen Zusammenhängen oder im privaten Umfeld ganz offen damit umzugehen. KarlLohmann (Diskussion) 13:51, 21. Jun. 2013 (CEST)

Hast Du ein Problem, und versuchst, es durch reguläre Ausdrücke zu lösen, hast Du zwei Probleme.

Hi, ich stehe gerade auf dem Schlauch, wie man das mit RegEx formuliert: Ich habe eine dreispaltige ASCII-Text-Tabelle:

|Maier          Mueller     Huber       |
|Schultze       Wachtendonk Luedenscheid|
|Gschachtlhuber                         |

Die "|" sind nicht Bestandteil der Ausgabe, sondern sollen nur Zeilenanfang und -ende symbolisieren.

Ich möchte erreichen, dass ich einen "Treffer" gemeldet bekomme, wenn der gesuchte Begriff - aber nur genau der, und nicht in einer Zusammensetzung - in der Zeile auftaucht. Das ganze muss case-insensitiv sein.

Mein Problem in diesem Beispiel ist der "huber". Den bekomme ich einmal als alleinstehenden "Huber" und einmal als "Gschachtlhuber" als Treffer ausgegeben.

Überlegung:

  • Entweder der Begriff steht am Anfang der Zeile (^) und hat danach eine Leertaste (wie bei Maier, Schultze, Gschachtlhuber),
  • oder er steht am Ende der Zeile, hat hinten ein Zeilenende ($) und vorne eine Leertaste (Luedenscheid),
  • oder er hat davor und danach eine Leertaste (restliche Begriffe).

Aber wie sag' ich "Zeilenanfang ODER Leertaste" bzw. "Leertaste ODER Zeilenende" in RegEx? "[^ ]" geht nicht, weil es - zumindest in dem RegEx-Dialekt, den ich habe, mit dem ^ innerhalb der [] die Negierung der Klasse einleitet, und es nicht als Zeilenanfang liest.

--149.172.200.27 00:48, 21. Jun. 2013 (CEST)

Modifier/Option i für case insensitiv und m für ^/$ als Zeilenbegrenzer sind anscheinend bekannt. Mir würde einfallen:
Hat leider beides nicht geklappt, da der Windows/DOS-Befehl "findstr" in seinem RegEx-Vokabular wohl diese Befehle nicht unterstützt (ich habe versucht, eine "net group /Domain GRUPPENNAMEHIER" - Ausgabe zu verarbeiten, um zu prüfen, ob der aktuell angemeldete Nutzer Mitglied der Gruppe ist. Der Befehl "ifmember" ist unter Win7 leider je nach UAC-Einstellung problematisch.).
Was aber (unter Windows 7 zumindest) geht: "whoami /groups" - das lässt sich bequem filtern.
Es listet eine Gruppe pro Zeile, beginnend immer mit %USERDOMAIN%\ (das "\" muss man in der Suchabfrage verdoppeln, also "%USERDOMAIN%\\") und am Ende des Gruppennamens kommt (mindestens) ein Blank.
Somit lässt sich auch zwischen Gruppe1 und Gruppe12 unterscheiden.
-- 149.172.200.27 11:36, 21. Jun. 2013 (CEST)
Also so wie ich das jetzt verstehe ist das das gesuchte findstr /R /I "\<huber\>" (laut der Doku von findstr unter Win7), was bei ganz obigen Beispiel die gesamte Zeile ausgibt. Ganz kann ich das aber nicht nachvollziehen, da ich hier gerade weder ein net group noch whoami habe, das gültigen Output liefert--se4598 / ? 12:19, 21. Jun. 2013 (CEST)
Ja, das scheint auch zu funktionieren. Danke! -- 188.105.141.207 15:45, 21. Jun. 2013 (CEST)

Augenringe vs. Blaues Auge

!!Die Unterscheidungsmerkmale von "Augenringe" versus "Blaues_Auge#Brillen-_und_Monokelh.C3.A4matom" erschliesst sich mir nicht!!

??Sind womöglich Augenringe nur per Ausschluss des Blaues_Auge identifizierbar bzw zu diagnostizieren ??

--85.183.86.31 06:50, 21. Jun. 2013 (CEST)

Satzzeichen sind keine Rudeltiere das nur nebenbei... --Ironhoof (Diskussion) 06:52, 21. Jun. 2013 (CEST)

Augenringe <=> Blaues Auge erklärt es doch ganz schön !?! Einfach mal die Einleitungen (Grunderkrankung <=> Verletzung) analysieren ... GEEZER... nil nisi bene 08:27, 21. Jun. 2013 (CEST)

Ergebnisse Vera 3

Hallo, mein Sohn hat mit dem Zeugnis die Auswertung der Vergleichsarbeit Vera 3 bekommen. Leider ist dort nur sein Ergebnis und das Ergebnis seiner Klasse angegeben. Ich würde sein Ergebnis gern im Verhältnis zu anderen Schülern aus Brandenburg oder Deutschland sehen um ggf. noch gezielt mit ihm lernen zu können. Wie komme ich an diese Zahlen? --141.15.33.1 08:28, 21. Jun. 2013 (CEST)

Hier Info und Kontaktadressen. Gezieltes Lernen bekommst du auch hin, wenn du mit Junior kommunizierst (Was haben sie gemacht. Was hat er verstanden und was nicht. Lass ihn seinen Krempel formulieren etc.) und du dann eher Richtung Denken und Nachdenken und "Erdenken" gehst als Richtung Lernen. Meist traut man Kindern zu wenig zu.... GEEZER... nil nisi bene 09:35, 21. Jun. 2013 (CEST)
Mein Junior ist gut. Jedenfalls im Vergleich zur Klasse. Er über 90%, die Klasse zwischen 80 und 90%. Ich möchte es aber im Vergleich zu anderen Schulen einschätzen können. Kann ja sein, dass die Schüler seiner Schule generell Defizite habe. Danke für die Internetseite. Ich werd dort mal anfragen. --141.15.33.1 09:59, 21. Jun. 2013 (CEST)
Über 90 % ist doch sehr erfreulich. Mal weg von der Schule - wie beurteilst du selber oder andere Personen (z.B. sein Klassenlehrer [in Abwesenheit von Junior fragen]) seine Pfiffigkeit? Die relativ guten Ergebnisse zeigen doch, dass ihm (a) Lernen/Mithalten nicht schwer fällt und/oder er (b) beim Lernen Spass hat. Wie du zwischen den Zeilen lesen kannst; halte ich nicht viel von globalen Vergleichen Einer vs. eine Gruppe. Es zählen ja im Leben auch noch andere Faktoren, die in der Schule nicht gemessen werden. :-) GEEZER... nil nisi bene 10:53, 21. Jun. 2013 (CEST)
Ich bin stolz auf ihn und mit seinen Leistungen (auch beim um-die-Ecke-Denken) mehr als zufrieden. Sicher hat er bis zur irgendwelchen Auswahlverfahren (Wunschgymnasium, Job, Studium) noch ne Menge Zeit. Aber ich denke: Zu wissen wo man steht ist nicht falsch. Und irgendwelche Auswahlverfahren beschränken sich ja später nicht auf seine Klasse, sondern zumindest auf die Schüler der Schulen in der Umgebung. Daher das Interesse an diesen Zahlen. Und wenn die 90% eben doch nicht so toll sind, weil der Brandenburger Durchschnitt bei 97% liegt und seine Klasse nun gerade besonders schlecht abgeschnitten hat, weil die Lehrerin ... Egal, hab per Mail mal freundlich angefragt. --141.15.33.1 11:29, 21. Jun. 2013 (CEST)

Bewerbungen Eingangsbestätigung was tun

Hallo, Ich bin derzeit in der bewerbungsphasd für Ein Diplom Praktikum, was soll ich tun wenn eine Firma mir eine Eingangsbestätigung Sendet, mich artig bedanken oder schweigen bis ich erfahre ob mehr geht Vorstellungsgespräch etc....?--84.59.25.12 11:50, 21. Jun. 2013 (CEST)

Abwarten. Steht doch sicherlich auch in dem Schreiben, sowas wie "...danken Ihnen und kommen nach Sichtung aller Bewerbungen ggf. auf Sie zurück!" 31.16.204.27 12:07, 21. Jun. 2013 (CEST)

Rechtswidrige Anordnungen von Gerichten

Hier ordnet ein Amtsgericht einen Hausdurchsuchung an, die eine höhere Instanz als rechtswidrig einordnet. Mir geht es nicht um den speziellen Fall, zumal man ja nicht wissen kann, ob die Augsburger alle Begrifflichkeiten richtig verwendet, sondern ganz allgemein. Eine festgestellte Rechtswidrigkeit ließe auf eine jenseits des Interpretationsspielraums liegende Fehleinschätzung schließen. Liegen Rechtsbeugung o.ä. Schwerwiegendem wird - mutmaße ich als blutiger Laie - sich wohl sich ein Staatsanwalt mit der Sache beschäftigen. Was geschieht bei geringerem "Verschulden"? Ist das ein Antragsdelikt? Kann ein Richter bei geringem Verschulden strafrechtlich überhaupt belangt werden? Ich habe den Eindruck, dass Gerichte (inzwischen?) recht großzügig Hausdurchsuchungen anordnen und Artikel 13 GG Unverletzlichkeit der Wohnung im Zweifel für weniger wichtig erachten, als die gefühlte Möglichkeit ev. eine Straftat aufklären zu können. // Dieser Satz ist eine Meinungsäußerung, darf entfernt werden: Nicht, dass die WP demnächst zur Herausgabe meine Daten aufgefordert wird - Man muß sich absichern ;-)//ThomasStahlfresser 12:22, 21. Jun. 2013 (CEST)

Der Artikel Rechtsbeugung erklärt doch eigentlich recht ausführlich, dass Richter hier ganz gut geschützt sind. Hätte in der Praxis auch eigenartige Folgen, wenn jede höherinstanzlich verworfene Entscheidung eines Richters zu dessen strafrechtlicher Verfolgung führen würde: Bei den rund 50.000 Verfahren, die pro Jahr bei den Landgerichten als Berufungsinstanz allein im Strafbereich eingehen, werden (sehr grob gezählt) 20.000 Entscheidungen der Vorinstanz aufgehoben oder abgeändert. Wenn Du so willst, hat in jedem dieser Fälle also ein Gericht "rechtswidrig" entschieden. Und man könnte es ja kaum als wünschenswert betrachten, wenn binnen kurzer Zeit die gesamte deutsche Amtsrichterschaft in den Knast käme. (Ja, ich weiß, der letzte Satz ist POV und manche fänden es im Gegenteil hochgradig erfreulich...) --Rudolph Buch (Diskussion) 13:54, 21. Jun. 2013 (CEST)

Lies doch bitte mal den Artikel Rechtsbeugung. Dort ist eigentlich die Problematik des Tatbestandes ziemlich gut erklärt. Konkret würde ich bei Durchsuchungsanordnungen einen relativ hohen Ermessensspielraum für den Richter sehen. Eine Verurteilung wegen Rechtsbeugung kommt da imho nur bei krassen Fällen in Betracht. --Archwizard (Diskussion) 13:40, 21. Jun. 2013 (CEST)

bevor das strafrechtlich relevant wird wäre allenfalls noch auf ziviler ebene staatshaftung (in österreich amtshaftung) für den ersatz eines allfälligen schadens denkbar. aber auch nur dann, wenn man der richter über eine vertretbare rechtsmeinung hinausgegangen ist. dafür brauchts auch schon eine ordentliche portion "mistbauen" durch einen richter. lg, --kulacFragen? 14:01, 21. Jun. 2013 (CEST)
ich hab mal gehört, dass ne Beschwerde gegen einen Bescheid eines Staatsanwalts als Dienstaufsichtsbeschwerde empfunden wird... vllt regeln die Juristen das ja irgendwie untereinander, wenn kein richtig grober Verstoß vorliegt... :-) --Heimschützenzentrum (?) 15:23, 21. Jun. 2013 (CEST)

Seitenwandinfarkt

Wie lautet nochmal das Kommando zur Einstellung der Gesundheitsthemen-Info?
In unserem Artikel Myokardinfarkt taucht das Wort „Seitenwandinfarkt“ nicht auf (und in der restlichen WP auch nicht). Nun wurde aber gerade dieses Wort von Ärzten gebraucht, als ein sehr naher Verwandter dieser Tage damit ins Krankenhaus kam. Nachdem via Katheter ein Stent („nehmen wir mal den 10er“) gemacht wurde, soll, so ebenfalls die ärztliche Auskunft, der Zustand danach an sich besser sein als der Zustand davor, weil die verstopfte Arterie ja nun wieder frei sei. Wie verträgt sich das mit der Aussage in Myokardinfarkt, daß bei jedem Infarkt immer ein Teil des Muskelgewebes unwiderruflich untergehe? --Freud DISK Konservativ 13:19, 21. Jun. 2013 (CEST)

Der Begriff findet sich in med. Lehrbüchern von 2013. Man sollte das der Redaktion Medizin vorlegen zur Beurteilung einer Einarbeitung. GEEZER... nil nisi bene 13:29, 21. Jun. 2013 (CEST)
Danke. Ich übertrage das dorthin. --Freud DISK Konservativ 13:32, 21. Jun. 2013 (CEST)

Gesamtfläche aller 962 UNESCO-Weltkultur- und naturerbestätten

Frage oben ist wohl selbsterklärend. Es werden vermutlich mehrere 100 km² sein, aber ich möchte eine genaue Angabe haben. Viele Grüße --84.149.163.189 14:21, 21. Jun. 2013 (CEST)

Ja, etwas mehr. Aber mir ist deine Definition noch nicht ganz klar.
Die erste Suche mit => UNESCO world heritage surface <= liefert z.B. : "Since the early 1980s, forty-three marine sites have been inscribed on the World Heritage List, covering about 1.4 million km2 of ocean surface." GEEZER... nil nisi bene 15:17, 21. Jun. 2013 (CEST)
(BK) es wird viel deutlich mehr sein als ein paar hundert km2. allein das Wattenmeer (Nordsee) hat 9000km2, das Great Barrier Reef hat knapp 350.000km2. Für das Zusammenrechnen ergibt das allerdings den Vorteil, dass man den gesamten Kleinkram weglassen kann, weil er von der Fläche her irrelevant ist. Ein paar große Nationalparks und Geotope sollten reichen, um einen recht guten Annäherungswert hinzubekommen. Also Empfehlung für's vorgehen: Liste des UNESCO-Welterbes und in den anderen Listen die Nationalparks und anderen Geotopen raussuchen, zusammenaddieren und den restlichen Kleinkrams ignorieren. Die ganzen Häuser, Burgen etc. haben alle keine relevante Fläche. -- southpark 15:20, 21. Jun. 2013 (CEST)
Auch deren Statistik-Seite gibt keine Oberfläche. Einige Gebiete werden nur vage als "areas" angegeben. GEEZER... nil nisi bene 15:27, 21. Jun. 2013 (CEST)

Verklausulierte Todesanzeigen

Warum wird in Todesanzeigen oder Nachrufen so selten die Todesursache genannt. Warum nicht "an Krebs" statt "nach langer Krankheit", "bei einem Autounfall" oder "an einem Herzinfarkt" statt "plötzlich und unerwartet"? Wird das so gemacht weil es eben Brauch ist oder gibt es betimmte Gründe? --Sakra (Diskussion) 15:24, 17. Jun. 2013 (CEST)

Persönlichkeitsrecht und Privatsphäre. Vermute ich mal. --LimboDancer (Diskussion) 15:28, 17. Jun. 2013 (CEST)
Das Ereignis an sich - und die Angabe der damit emotional belegten Personen (waren sie lange damit konfrontiert? Geschah es plötzlich? etc.) - scheint wichtiger zu sein als der genaue Ablauf des Ereignisses. Bei Geburtsanzeigen ist es ähnlich: Die Zeitdauer der Presswehen oder der um 9:15h erfolgte Kaiserschnitt werden meist verschwiegen. Auch bei Hochzeitsanzeigen wird nur selten geschrieben, ob man mit dem Schnellboot in den Hafen fuhr, oder erst nach mehrmaligem Rammen des Kais den Zutritt erreichte. Details machen diese Ereignisse einfach zu profan und lenken von der Emotionalen Wucht ab (TF). GEEZER... nil nisi bene 15:50, 17. Jun. 2013 (CEST)
Jetzt habe ich ein Bild im Kopf, wie die Wuchtbrumme den armen Kai in den Hafen der Ehe rammt... Da Todesanzeigen wohl älter sind als Persönlichkeitsrechte und Privatsphäre (wie dies heute verstanden wird): Man könnte es wohl unter Pietät subsumieren. Vermutlich gehört(e) es sich nicht, mittels Beschreibungen der Todesursache noch Zweifel am göttlichen Entschluss aufkommen zu lassen, jedenfalls nicht öffentlich (TF). -- Ian Dury Hit me  16:45, 17. Jun. 2013 (CEST)
Diese Zweifel gibt es sehr wohl: Todesanzeigen von Selbstmörder_inne_n erkennt man am "Warum?" in der Titelzeile. --Φ (Diskussion) 16:50, 17. Jun. 2013 (CEST)
Und auch bei vielen anderen gibt's bei der Beerdigung, die ja in der Regel auf noch älteren Traditionen fusst als die Todesanzeige, und 'ne noch pietätvollere Veranstaltung ist, bei der die unterschiedlichsten Gruppen von Trauernden zusammenkommen, dann doch wieder Infos (natürlich nur zu einem gewissen Detailgrad, also keine Angaben, welche Krebszellen sich wohin ausbreiteten, aber durchaus sowas wie "kam am Mittwoch nach einem Herzinfarkt ins Krankenhaus, nachdem er ja schon jahrelang Herzprobleme hatte, und ist dort am Samstag Morgen gestorben"). --YMS (Diskussion) 16:54, 17. Jun. 2013 (CEST)
Wenn ich Krebs hätte oder hohen Blutdruck, dann würde mir eine Angabe von Todesursachen in den Anzeigen doch eher zusätzliche Angst einjagen. "Guck mal, der ist da auch dran gestorben, was ich habe." Also kann man es auch aus Rücksicht auf noch Lebende weglassen. Der Tod ist generell ein Tabuthema, und eigentlich will doch niemand genau wissen, woran fremde Leute gestorben sind. Wer den Toten kannte, erfährt es bei der Beerdigung oder im Gespräch mit den Hinterbliebenen. --Sr. F (Diskussion) 17:39, 17. Jun. 2013 (CEST)
(BK) Der Mensch stirbt, wenn er Glück hat, an Altersschwäche, wenn er Pech hat, an einer körperlichen oder seelischen Erkrankung (GG möge mir den Dualismus verzeihen), wenn er noch mehr Pech hat, an den Folgen eines Unfalles, und vielleicht manchmal aufgrund einer wohl abgewogenen rationalen Entscheidung über Leben oder Tod. Das einzige, das zählt, sind die Hinterbliebenen und ihre Gefühle. Warum sollte man in einer Todesanzeige irgendjemandes Voyeurismus befriedigen? Gibt's die Anzeige dann billiger, oder bekommt man eventuell noch Honorar? 85.180.196.22 17:49, 17. Jun. 2013 (CEST)
Ich gebe meine Theorie (Pathos) auf und schliesse mich - nach Durchsicht mehrerer sehr alter Todesanzeigen - der zweiten genannten (...hat es gefallen, XYZ abzuberufen...) Theorie an. diese hier erinnert ein bissl an Die Brücke von San Luis Rey. GEEZER... nil nisi bene 17:53, 17. Jun. 2013 (CEST)

Gott hat es gefallen... schreibt hier in Oberschwaben nur noch der ehemalige Adel. Dass z. B. die Waldburger ihre Verwandten so großformatig, schwülstig und mit allen ihren angeblichen Titeln und Durchlauchten verabschieden, bringt der Schwäbsichen Zeitung einiges an verkauftem Anzeigenplatz. --FA2010 (Diskussion) 17:57, 17. Jun. 2013 (CEST)

Ich kenne mehrere Fälle, wo der später Verblichene noch zu Lebzeiten die gewünschte Todesanzeige formuliert hat. Aber wie ist es meist? Die (oftmals) geschockten oder zumindest emotional gebeutelten (oder auch unwissenden) Angehörigen bekommen vom Bestatter gängige Beispiele vorgelegt (in den Ausführungen klassisch, religös, modern, sehr modern, poetisch, alter Mensch, junger Mensch, Suizid, Unfall, verschiedene Sprüche, Symbole usw.) und da sitzt man dann und sucht aus. Klar, daß die Musteranzeigen die "erprobten Floskeln für alle Fälle" aufweisen. Aber abgesehen davon: ich möchte dermaleinst von mir nicht lesen, ich sei an "metastasierendem Darmkrebs mit Komplikationen in Lunge und Speiseröhre" gestorben - "schwere Krankheit" genügt mir dann. --Bremond (Diskussion) 18:18, 17. Jun. 2013 (CEST)
Das Argument mit der Privatsphäre habe ich nie verstanden. Was ist so privat daran, dass man an Krebs starb? Dass man - als Lebender - Krebs hat, kann schon privat sein - aus Angst vor Diskriminierung. Aber nach dem Tod?! Im Übrigen, man benennt ja die Todesursache - Kandidaten für "lange schwere" gibt's nicht so viele. Nur hat man irgendwie Angst vor dem Namen...--Alexmagnus Fragen? 19:54, 17. Jun. 2013 (CEST)

Und für mich hat der Wunsch, die Todesursache zu kennen, nichts mit Voyeurismus zu tun. Ich will einfach wissen, woran Menschen sterben (generell, nicht im speziellen Fall). Übrigens, Altersschwäche wird immer seltener genannt - man findet immer öfter eine "richtige" Ursache.--Alexmagnus Fragen? 19:58, 17. Jun. 2013 (CEST)

Du hast nie verstanden, was "so privat daran" ist, welche Krankheit ein bestimmter Mensch hatte?? Wenn es eines Tages mal so weit ist, dass es überhaupt nur noch eine letzte Sache gibt, die üblicherweise noch als "privat" gilt, dann wird das immer noch die Krankenakte sein. Wenn du "generell" wissen willst, woran Menschen sterben, dann musst du seriöse Statistiken und Studien zum Thema bemühen: Todesursache#Statistiken als Einstieg... Sich darüber aus personenbezogenen Todesanzeigen schlaumachen zu wollen, hat keinerlei Wert außer den voyeuristischen. --178.202.30.217 23:01, 17. Jun. 2013 (CEST)
ein Punkt, der bisher noch nicht erwähnt wurde, ist das postmortale Persönlichkeitsrecht des Verblichenen. Es muss also auch dem mutmaßlichen Willen des Verstorbenen entsprechen, wenn Details in die Todesanzeige kommen. In vielen Fällen ist sogar die auf dem Totenschein eingetragene Todesursache geschönt. Ich kenne Fälle, wo Suizid plötzlich zu einer unheilbaren Krankheit wurde oder wo auf Wunsch der Hinterbliebenen offiziell ein anderer Ort als Sterbeort eingetragen wurde. Eine Todesanzeige ist kein amtliches Dokument. Sie zu schönen ist nicht strafbar. @Fragesteller, würdest Du über Dich beispielsweise als Todesursache lesen wollen: „Ist auf der Autofahrt von seiner Lieblingsschwulenkneipe zu seiner Ehefrau in alkoholisiertem Zustand bei Dunkelheit und Nebel wegen überhöhter Geschweindigkeit von der Straße abgekommen und gegen einen Betonpfeiler geprallt“? --Rôtkæppchen68 23:11, 17. Jun. 2013 (CEST)
Du hast Recht – dass der Pfeiler aus Beton war, sollte man wirklich besser weglassen. -- Geaster (Diskussion) 09:54, 18. Jun. 2013 (CEST)
Ähm, jetzt, weil die Freiheitlichen mit der österreichischen Bauindustrie verbandelt sind oder wegen dieser Werbung ? --84.191.136.149 18:42, 18. Jun. 2013 (CEST)
... ist unser Lebensmensch auf tragische Weise vom rechten Weg abgekommen ... --Zerolevel (Diskussion) 15:07, 19. Jun. 2013 (CEST)

Die Sache ist aber, dass "nach langer schwerer Krankheit" danach klingt, dass derjenige, der die Anzeige geschrieben hat, einen Wunsch, ja gar Drang hatte, die Todesursache mitzuteilen (sonst hätte er die hässliche Formulierung ganz weggelassen), aber Angst hatte, deren Namen zu nennen (so eine Art Mystik). Was ein Vorteil von persönlichen Anzeigen (von berühmten Personen) gegenüber den Statistiken ist: man kennt den Lebenswandel des Verstorbenen zumindest im Ungefähren. So dass man auf Zusammenhänge Lebenswandel/Todesursache kommen kann, die in den Statistiken gar nicht auftauchen. @Rotkäppchen: in diesem Fall würde mir die Angabe "Autounfall" genügen. Genau wie bei Krebs mir die Angabe "Krebs" genügen würde, der betroffene Organ interessiert mich nicht.--Alexmagnus Fragen? 23:25, 19. Jun. 2013 (CEST)

Alle, die es etwas angeht wissen bereits die Todesursache, das sind die näheren Angehörigen und Freunde. Die Todesanzeige informiert Ex-Nachbarn, Ex-Arbeitskollegen, ehemalige Schulfreunde und flüchtige Bekannte ebenso wie ganz normale neugierige Mitmenschen und auch den ein oder anderen aus den Augen verlorenen engeren Freund. Letzterer wird vermutlich betroffen zur Beerdigung erscheinen und dort als ehrlich Interessierter näheres erfahren. Alle anderen geht es eigentlich nix an, und auf das angestachelte Getratsche "soso, Krebs, ja, der hat ja auch immer geraucht wie ein Schlot" oder "Autounfall? - Die war bestimmt besoffen!" kann man wohl verzichten. Mitunter läßt sich aus der Kombination "nach langer schwerer Krankheit" und "statt Blumen sind Spenen erbeten an die Organisation zur Bekämpfung von..." eine Vermutung ableiten.feba disk 02:26, 21. Jun. 2013 (CEST)
Die Phantasie mancher Leute ist beachtlich: "Wunsch", "Drang", "Angst", "Mystik" nur weil in der Todesanzeige steht "nach langer schwerer Krankheit". Dieser Satzbaustein ist keine geheime Botschaft, mit der Hinweise auf eine mystische, unaussprechliche Todesursache gegeben werden soll.
Eine Todesanzeige informiert die Öffentlichkeit über den Tod, weil sich in der Öffentlichkeit Menschen befinden können, für die die Tatsache des Todes dieses Menschen von wichtigem Interesse ist. Das sind nicht nicht nur die von Feba genannten Schulfreunde oder Arbeitskollegen und Nachbarn, sondern auch z.B. aktuelle Vertragspartner, Gläubiger, Schuldner, unbekannte Erben... Dass man "nach langer schwerer Krankheit" in eine solche Todesanzeige schreibt, ist sicherlich dadurch entstanden, damit diese Leute nun auch wissen, warum der Verstorbenen wohl schon seit längerem vor seinem Tod auf keine Briefe (oder gar Mahnungen) mehr reagiert hatte. Ebenso gibt es den anderen Satzbaustein "plötzlich und unerwartet verstorben", damit die Leute, die die Person erst vorgestern noch putzmunter auf der Straße gesehen haben, die Nachricht überhaupt glauben können. Ob einer fremden Person die Information "Krebs" reichen würde und der Fremde das genaue Organ gar nicht wissen will, ist sowas von irrelevant. Die ärztliche Schweigepflicht bezieht sich auch nicht nur darauf, welche Organe betroffen sind, sondern auf die Art der Krankheit überhaupt. Das geht schlicht und ergreifend niemanden was an, schon gar keine neugierigen Verschwörungstheoretiker. --178.202.30.217 12:33, 22. Jun. 2013 (CEST)

Südosten Deutschlands

In einer Dokumentation über die Lausitz wurde diesselbe gerade als im Südosten Deutschlands liegend bezeichnet. Ich dachte immer Bayern wäre quasi Südosten. Aber wie kommt man darauf die Lausitz als im Südosten Deutschlands zu suchen? Ist das irgendwas nicht-geografisches sondern etwas geschichtliches? Als ein Fehler kann ich mir das schlicht nicht wirklich vorstellen (Hallo, Arte, das ist Qualitätsfernsehen ;-)). --Friechtle (Diskussion) 19:36, 17. Jun. 2013 (CEST)

Sie liegt im Süden des Ostens. Also quasi Südosten. --FA2010 (Diskussion) 19:52, 17. Jun. 2013 (CEST)

Hallo Friechtle, ich tippe mal auf einen "Versprecher" (das kann auch bei Arte passieren ;-). Die Lausitz liegt ja im Süden Brandenburgs und im Osten Sachsens. Vielleicht dachte der Moderator auch im Südosten der neuen Bundesländer, wurde sich dann aber bewusst, das man das heutzutage nicht mehr so bezeichnen würde und hat es spontan umformuliert. --Merrie (Diskussion) 19:53, 17. Jun. 2013 (CEST)
Wieso? Die Lausitz ist genausoviel und genauswenig Südosten Deutschlands wie Bayern. Wenn man von einem beliebigen Punkt irgendwo in der "Mitte" Deutschlands erst ganz nach Süden geht und dann ganz nach Osten, dann ist man in Freilassing. Und wenn man erst ganz nach Osten geht und dann ganz nach Süden, dann kommt man in Görlitz raus. Niederschlesischce Oberlausitz im Gebiet des Mitteldeutschen Rundfunks, also eindeutig Nordwestdeutschland. Oder eben nicht. --FA2010 (Diskussion) 19:56, 17. Jun. 2013 (CEST)
Siehe Südostdeutschland. --Pp.paul.4 (Diskussion) 20:03, 17. Jun. 2013 (CEST)
Wesentliche Fragen kommen immer wieder. -- Aerocat 22:18, 17. Jun. 2013 (CEST)
Es waere halt schoen, wenn Deutschland rechteckig (an den Laengen- und Breitengraden ausgerichtet) waere. Dann haette man einen wohldefinierten Suedwesten und Suedosten. Nun haben wir leider diese Knicke dort, wo Tschechien und Elsass/Lothringen liegen. Und dann noch Nord- und Ostsee, so dass man im Prinzip auf in Schleswig-Holstein noch eine Nordwest- und Nordostecke Deutschlands definieren kann. -- Arcimboldo (Diskussion) 03:14, 18. Jun. 2013 (CEST)
Nö, ich verstehe es einfach nur nicht wieso die Lausitz als im Südosten liegend bezeichnet wird, ist sie es doch scheinbar nicht. Sie liegt ja auch nicht südlich und östlich von Deutschlands Mitte, sondern wohl eher nördlich, aber vor allem östlich. Wesentliche Gebietsänderungen Deutschlands sind ja auch schon mind. 20 Jahre her (steht ja auch in Südostdeutschland, bis zum Ende des 2. WK benutzt). --Friechtle (Diskussion) 11:36, 18. Jun. 2013 (CEST)
Nach Deiner Überlegung wäre dann aber auch Bayern nicht Südosten, sondern einfach Süden, da selbst Bautzen östlicher liegt als der gesamte Freistaat Bayern (vgl. z. B. Niederdorla - Zittau [ist südlicher gelegen] bzw. Bayern [das Bundesland!]: Würzburg liegt westlicher als Niederdorla).--IP-Los (Diskussion) 14:30, 18. Jun. 2013 (CEST)
Ich glaube Bayern liegt südlich und östlich von Niederdorla (zumindest großteils). Aber das war ja auch gar nicht mein Vorschlag, sondern ein Vorschlag aus der oben verlinkten früheren Auskunftsfrage. --Friechtle (Diskussion) 15:26, 18. Jun. 2013 (CEST)
Vom Staatsgebiet der ehem. DDR ausgehend liegt die Lausitz tatsächlich im Südosten. Vielleicht eine alte Sendung oder ein alter Moderator? --Optimum (Diskussion) 18:45, 18. Jun. 2013 (CEST)
Das ist immer so eine Sache. Nach einer Definition war ich im äußersten Südwesten Deutschlands, nach anderer Definition liegt diese Ecke ganz woanders. Jeder hat da einen eigenen Standpunkt. Eindeutig kann man das nur bei den afrikanischen Staaten sagen, die in der Regel eckige Grenzen haben. -- Liliana 20:50, 18. Jun. 2013 (CEST)
Die USA haben auch rechteckige Staaten Colorado, Wyoming und sogar ein rechtwinkliges Vierländereck Four Corners. --Rôtkæppchen68 00:36, 19. Jun. 2013 (CEST)
Wegen meiner besseren Ortskenntnis spreche ich nicht den Südosten, sondern den äußersten Südwesten an. Ich war am vergangenen Sonntag in Rilchingen-Hanweiler. Es gibt in Deutschland keinen Ort, der zugleich weiter westlich und weiter südlich liegt. Das gleiche gilt für Perl, Weil am Rhein und bestimmt noch für einige weitere Orte, übrigens nicht für das weiter oben verlinkte Fischbach bei Dahn, da Weil am Rhein zugleich südlicher und westlicher liegt. Das heißt, mehrere Orte erfüllen diese Bedingung des äußersten Ortes im Südwesten. Es wäre doch mal interessant, diese Extrem-Orte zu benennen. --Pp.paul.4 (Diskussion) 00:03, 21. Jun. 2013 (CEST)
Das kannst Du mit Google Earth leicht selbst bestimmen. Von Deinen Kandidaten ist es derjenige Ort, der am weitesten vom Flächenmittelpunkt Deutschlands entfernt ist. --Rôtkæppchen68 00:11, 21. Jun. 2013 (CEST)
Ich hab bei Google Earth nur mal die Reißnägel gesetzt, ohne das Lineal auszupacken. Danach sind nach Augenmaß Perl und R.-H. von Landstreit bzw Ober/Niederdorla nicht weiter weg als Karlsruhe. Weil am Rhein oder Grenzach-Wyhlen sind deutlich weiter von Landstreit/Dorla entfernt. --Rôtkæppchen68 00:19, 21. Jun. 2013 (CEST)

Es gibt immer mal wieder richtig gute Fragen, nämliche solche, die das Denken anregen. Die Ausgangsfrage ist so eine. Es ist im Prinzip das Gleiche als wenn man fragt, ob Norwegen westlich, nördlich oder östlich von Schweden liegt. Für jede der drei Himmelsrichtungen gibt es gute Argumente. --Dioskorides (Diskussion) 00:02, 22. Jun. 2013 (CEST)

Getriebefehlfunktion nach Zahnriemenwechsel

Kann nach einem fehlerhaften Zahnriemenwechsel eine Fehlfunktion beim Getriebe eintreten, wie etwa dass nur noch zwei Gänge verfügbar sind, wie das hier [11] von einem Autoverkäufer beschrieben wird? Oder ist das technisch nicht möglich und liegt da eine falsche Beschreibung einer Fehlfunktion vor? Normalerweise dürfte ein Zahnriemen doch nur den Motor und nicht das Getriebe betreffen. 46.115.106.231 21:33, 17. Jun. 2013 (CEST)

Lies den verlinkten Text genau, dazu die Angaben im Artikel Volvo S80. Erkenne, dass da weit mehr gemacht wurde als nur ein Zahnriementausch. Besagtes Auto hat obendrein ein Automatikgetriebe. --Rôtkæppchen68 21:40, 17. Jun. 2013 (CEST)
Ja, aber losgetreten durch einen regulären Zahnriemenwechsel. Die erwähnten Teile gehören mit in die Baugruppe des Zanriemens bzw. sind bei vielen Modellen mit zu tauschen. 46.115.106.231 21:46, 17. Jun. 2013 (CEST)
Ich glaube auch, da kann irgendwas nicht stimmen. Die aufgelisteten Arbeiten sind Kleinigkeiten und Standard beim Zahnriemen. Was da gemacht wurde, betrifft das Getriebe gar nicht. Es könnte sich aber um ein verbreitetes Phänomen der menschlichen Beobachtung handeln: wenn etwas zeitgleich passiert, muss dies nicht ursächlich oder im Zusammenhang sein...Hm...allerdings hätte ich die Werkstatt verklagt, wenn ich einen Wagen zu einer Standartreparatur bringe und mit Getriebeschaden zurück bekomme. Klingt alles merkwürdig. --Wicket (Diskussion) 22:48, 17. Jun. 2013 (CEST)
Eben, es stimmt da was nicht. Die Baugruppe wird standardmäßig als Verschleißteile gewechselt. Es könnte auch ein Fehler an der Elektrik sein, aber den hätten sie schon gefunden, bevor sie ans "Ende vom Latein" gekommen sind. 46.115.106.231 22:57, 17. Jun. 2013 (CEST)
Tja, das läßt sich aus der Ferne nicht beurteilen. Bei den VWs, die ich früher hatte, löste sich mit 100%iger Sicherheit irgendwann ein Metallplätchen am Schalthebel, was auch dazu führte, dass sich nur noch zwei Gänge schalten ließen. Das kostete nur 20 Euro. Aber wenn hier angeblich Fachwerkstätten den Fehler schon nicht finden...--Wicket (Diskussion) 23:06, 17. Jun. 2013 (CEST)
Hör auf mit Werkstätten! Letztens ist eine Citroën-Vertragswerkstatt daran gescheitert, die Hydropneumatikfedern an meiner Mistkröte auszuwechseln. Nach dem richtigen Tipp von hier habe ich das dann selbst geschafft. Das die Werkstatt nicht wusste wie es geht ist ein Armutszeugnis. (Ist aber nicht symptomatisch für Citroën, dass eine Werkstatt mal eins meiner Autos *nicht* kaputter gemacht hätte als es vorher war kann ich mich nicht erinnern.) -- Janka (Diskussion) 00:36, 18. Jun. 2013 (CEST)
http://www.motor-talk.de/forum/volvo-s80-t6-automatikgetriebe-schaltprobleme-t2782894.html Oh noch einer? Sieht so aus als ist die Wartung nicht ganz so einfach. Kabel ab, Stecker ab. Sensoren zur Raddrehzahlkennung nicht richtig motiert, Ölkühler scheint zuzusetzen. D.h. raus damit und Wartungsintervall verkürzen (Filtersatz und Automatikgetriebeöl ATF / DEXRON). Es ist heute normal, dass Motor- und Getriebesteuergeräte zusammengeschaltet sind bzw. kommunizieren. Grund ist Energieeinsparung und Drehmomentabsenkung im ersten Gang. Das Motorsteuergerät gibt dann nie Vollgas, da das Getriebe dieses Drehmoment nicht mehr übertragen könnte, es würde beschädigt werden. Sitzen die Sensoren nicht oder ist das Kabel ab, wird die Drehzahl nicht oder falsch registriert. Das dürfte auf Deinen Fehler passen. Die Motordrehzahl kann an Riehmenrädern des Zahnriemen erfasst werden. --Hans Haase (Diskussion) 00:18, 18. Jun. 2013 (CEST)
Ein Elektronikdefekt wird es vermutlich sein. Dritter Gang ist bei Automatikgetrieben das Notprogramm, denn damit kann der Wagen zur Not noch anfahren und andererseits kommt man schneller als im Schneckentempo voran. -- Janka (Diskussion) 00:29, 18. Jun. 2013 (CEST)
Die Frage war genau ander zwei Gänge verfügbar. Der von Dir beschriebe Notlauf (je Modell/Hersteller) tritt ein, wenn die Motordrehzahl nicht erfasst werden kann. OBD sagt Dir wo der Schuh drückt bzw. welcher Wert aus dem Range läuft. Der Zahnriemenwechsel und danach der Fehler sind eindeutig. --Hans Haase (Diskussion) 00:48, 18. Jun. 2013 (CEST)
Ich kenn es so, dass der OT-geber auf der Kurbelwelle gleichzeitig zur Erfassung der Motordrehzahl dient. Wenn der weg ist sollte aber nicht nur das Getriebe meckern, sondern auch das Motorsteuergerät, da der Zündzeitpunkt nicht mehr erfasst werden kann. Wenn eine Werkstatt *das* nicht herausfindet, sollte man die Jungs da drin einsperren und das Tor zuschweißen. -> Die Geschichte des Verkäufers ist unglaubwürdig. Finger weg von dem Fahrzeug! -- Janka (Diskussion) 21:36, 18. Jun. 2013 (CEST)
Ergänzung: Die Automatikgetriebe von AW (Toyota, steckt aber auch in VW und einigen anderen Marken drin), deren Konstruktion ich mal genauer untersuchen durfte, haben übrigens einen *eigenen* Drehzahlsensor am Getriebeeingang. Die Erklärung aus Japan war, dass das Drehzahlsignal für die Getriebefunktion zu wichtig sei, als dass man sich auf die per K-Line oder per CAN übetragenen Werte verlassen würde. Ich denke die anderen Getriebehersteller werden das ähnlich sehen. (Unabhängig davon kann das Getriebe natürlich trotzdem in Notlauf gehen, wenn die beiden Werte nicht übereinstimmen. Ich würde persönlich ja nur 'ne Warnlampe angehen lassen.) -- Janka (Diskussion) 21:46, 18. Jun. 2013 (CEST)
Allgemein gesagt (nicht auf das Modell oder den Hersteller bezogen): Viele Geräte erfassen nicht, wenn ein Wert aus dem Bereicht (Range) läuft. Das deutet auf einen Kabelbruch, Wackelkontakt und diesen ganzen Unfug hin. „Quasi das Rascheln in der Mikrofonleitung raushören.“ Und wenn sie der Controller kann, kommt nach Murphy OBD-Software zum Einsatz, die das verschweigt. So und den gelernten Mechanikern, die heute dieser unzureichenden Mechatronik ausgesetzt werden, dürfen diese Zumutungen von Kunde, Chef und Werk ausbaden. Wundert Dich das unter diesen Umständen? --Hans Haase (Diskussion) 14:50, 21. Jun. 2013 (CEST)
Nenn es Deja-Vu, aber irgendwie kommt mir diese Geschichte so seltsam bekannt vor, als ob der anfangs erwähnte Volvo seit Jahren mit diesem Bug im Netz herumschwirrt. -- Janka (Diskussion) 20:26, 21. Jun. 2013 (CEST)

Abgrenzung Psychotiker Neurotiker

Ich weiß, dass der Begriff Neurotiker offiziell nicht mehr eingesetzt wird und man Psychose nur noch in Zusammenhang mit psychotischen Episoden mit Wahnvorstellungen usw. verwendet. Im Artikel Borderliner steht allerdings, Borderlin sei im Sinne der Psychoanalyse eine Art Grenzphänomen zwischen Neurose und Psychose. Deshalb frage ich mich: Wie ist Psychose und Neurose eigentlich definiert und wie grenzen sich diese beiden Krankheitsbilder voneinander ab?(nicht signierter Beitrag von 188.100.84.44 (Diskussion) 21:52, 19. Jun. 2013 (CEST))

das ist noch Gegenstand der Forschung... irgendwie ist in Psychiatrie/Neurologie gar nix ordentlich definiert: bewusstlos/wach, Epilepsie/Schizophrenie/Mobbing, Neurose/Psychose, ... lässt sich aber trotzdem Geld mit verdienen, auch wenn der Therapieerfolg ausbleibt... --Heimschützenzentrum (?) 22:29, 19. Jun. 2013 (CEST)
Ganz grob und nichtwissenschaftlich gesagt ist ein Neurotiker ein Typ, der in bestimmten Situationen unangepasst oder falsch reagiert, er kann aber am öffentlichen Leben teilnehmen und "funktioniert" mit gewissen Einschränkungen in Familie und Gesellschaft. Dazu gehören z. B. Leute mit Spinenphobie oder Platzangst, Waschzwang etc. Behandlung meist mit Gesprächen, Medikamente nur belgeitend oder vorübergehend. Ein Psychotiker hingegen ist ernsthaft krank und sollte in psychiatrische Behandlung kommen, er bringt sein Leben nicht auf die Reihe und ist nicht fähig einen Beruf auszuüben oder in der Familie Verantwortung zu übernehmen. Kinder großzuziehen etc. Dazu gehören z.B. Leute mit Wahnvorstellungen, Paranoia etc. Da hilft keine Gesprächstherapie, da braucht man Psychopharmaka oder Psychiatrie. Ein Boderliner ist eine Person, die je nach den Bedingungen, in der sie sich befindet, entweder ein normales Leben führen kann, oder unter ungünstigen Bedingungen z.B. Berufsstress, Beziehungsstress, tragische Verluste, Schicksalsschläge etc. auch völlig abdrehen kann und wie ein Psychotiker wirken kann. Borderliner waren früher die "Grenzfälle" zwischen Neurose und Psychose, heute gilt es als eigene Krankheit. Borderliner können in einem gesunden Umfeld ohne Medikamente leben und sind dann nur in akkuten Phasen unter medikamentöser Behandlung, sie sollten aber regelmäßig untersucht werden.--Giftzwerg 88 (Diskussion) 23:23, 19. Jun. 2013 (CEST)
Wenn du weißt, dass der Begriff der Neurose (i.d.F.) im psychiatrischen Umfeld heute nicht mehr verwendet wird, jener der Psychose (aber kaum der eines "Psychotikers") jedoch sehr wohl, leuchtet mir nicht ein, weshalb du dann trotzdem noch nach einer Abgrenzung "beider Krankheitsbilder" fragst, bzw. was für eine Antwort du auf diese Frage nun erwartest. Das ist die Abgrenzung. Und gemäß heutiger Sichtweise gibt es ein Krankheitsbild, welches einer klassischen Neurose entspricht, einfach nicht. Sondern die Leiden von Menschen, denen man früher etwa noch eine "Neurose" nachsagte, werden heute schlicht anders interpretiert, und weiter ausdifferenziert, d.h. sie erhalten alternative Diagnosen (die Borderlinestörung ist eine mögliche davon). Analog dazu ist genauso der Begriff "Psychose" kein Krankheitsbild im Sinne einer präzisen, abschließenden Diagnose (wobei deine Eingrenzung dieser unzutreffend ist), sondern das ist im Grunde ein recht diffuser Über- oder Sammelbegriff (ähnlich "Krebs") für verschiedene Erkrankungen u. Störungen mit jeweils unterschiedlichen Ursachen, Ausprägungen, Prognosen und Interventions-/Behandlungsmöglichkeiten; gleichwohl natürlich auch mit gewissen, charakteristischen Parallelen (eben das teilen Psychosen, bzw. psychotische Störungen). Aber ob jemand nun an einer akuten, wenn auch ggf. vorübergehenden Drogenpsychose leidet, oder an einer Alkohol-Halluzinose, oder an psychotischen Schüben in Verbindung mit einer depressiven Erkrankung/Persönlichkeitsstörung/usw., oder aber ob jemand chronisch betroffen ist, wie in den meisten Fällen der schizophrenen und affektiven Psychosen, ist offensichtlich ein Unterschied. Ebenso muss unterschieden werden zwischen einerseits organisch begründeten Psychosen (etliche somatische Erkrankungen können mit psychotischen Symptomen einhergehen, genauso Vergiftungen, ...) und den "endogenen" oder psychotischen Störungen i.e.S., d.h. solche, die sich nicht auf ein körperliches Leiden zurückführen lassen. Lange Rede, kurzer Sinn: Du siehst, es handelt sich tatsächlich mehr um ein ganzes Begriffsbündel und weniger um ein klar definierbares Krankheitsbild. (Schon deshalb ist deine Frage nicht zu beantworten.) Dasselbe gilt und dann erst recht für den noch viel schwammigeren Begriff der "Neurose", nur ist es in dem Fall heute eben irrelevant, vielleicht abgesehen von psychiatrie-historischen Interessen. Unser Verständnis von der Welt und uns selbst darin ist nun mal kein Zustand, sondern ein Prozess. --ZT (Diskussion) 22:50, 21. Jun. 2013 (CEST)

Partei Deutsches Reich

Was hat es mit der Partei Deutsches Reich auf sich, die an der Bundestagswahl teilnehmen will? Quelle:http://www.wahlrecht.de/doku/presse/20130618-1.htm

--79.255.24.85 15:25, 20. Jun. 2013 (CEST)

(Quetsch) Es ist gemein, auf eine externe Liste zu verlinken, nachdem ich mir gestern so viel Mühe gemacht habe, Liste der Parteien mit Beteiligungsabsicht an der Bundestagswahl 2013 als Tabelle zu bauen... --Rudolph Buch (Diskussion) 17:23, 20. Jun. 2013 (CEST)
Partei die eher dem rechten Spektrum zuzuordenen ist. Wobei Spinner auch ganz gut passt. Zitat Ziel aus dem Wahlprogramm: "allgemeine Gewichtsreduzierung (75 % der Deutschen sind zu dick)" Hier die Homepage von dennen: Klick --Lidius (Diskussion) 15:41, 20. Jun. 2013 (CEST)
Na, mal kucken, ob die teilnehmen dürfen... --Eike (Diskussion) 16:09, 20. Jun. 2013 (CEST)
Wohl ein Wiederbelebungsversuch der Sozialistischen Reichspartei. —PοωερZDiskussion 16:54, 20. Jun. 2013 (CEST)
Das Programm hört sich tatsächlich etwas national-sozialistisch an und hat eiene tolle Stilblüte: Offenbar soll die Abschaffung von Hühnerbatterien und ähnlichem unter Strafe gestellt werden.--SFfmL (Diskussion) 18:18, 20. Jun. 2013 (CEST)Mit national-sozialistisch meine ich im übrigen, dass das Progamm sowohl nationalistische als auch sozialistische Züge hat.
Nach Überfliegen deren Netzseite kann ich die in dieser Diskussion bereits vorgenommenen Etikettierungen (noch?) nicht ganz nachvollziehen:
Wiederbelebung der Sozialistischen Reichspartei?? Laut der Partei-Netzseite ist diese Partei alles andere als offen neonazistisch.
Nationalsozialistisch?? Formulierungen wie "unverantwortliche Machtpolitik des dritten Reiches", "durch die Diktatur (des dritten Reiches) vergiftetes geistiges Potential", "wir betrachten mit Sorge den Neuaufbruch der rechten Szene", (für eine)"ungestrafte Möglichkeit, sich der Landesverteidigung mit Waffengewalt zu entziehen" hören sich wohl kaum national-sozialistisch an.--Stubfighther (Diskussion) 20:11, 20. Jun. 2013 (CEST)
Nationalismus hat meistens sozialistische Züge, denn die Volksgemeinschaft muss zusammenhalten. —PοωερZDiskussion 18:49, 20. Jun. 2013 (CEST)
„Daher ist es zwingend erforderlich, die BRD gegen alle Bestrebungen verschiedener Gruppen zu verteidigen, die sie zugunsten des Wiederaufbaus des Deutschen Reichs in seinen Grenzen von 1937 beseitigen wollen.“ Häh? Wenn die mit den Grenzen von heute zufrieden sind, warum reden die dann die ganze Zeit von den Grenzen von 1937? --Chricho ¹ ² ³ 19:32, 20. Jun. 2013 (CEST)
(BK) @23PowerZ Nö, schau Dir mal die DNVP an.--IP-Los (Diskussion) 19:35, 20. Jun. 2013 (CEST)
@Chricho Lies einfach den nächsten Absatz: "Das Deutsche Reich, welches 1945 nicht untergegangen ist (siehe auch Urteile Karlsruhe), findet seine Bestätigung selbst bei der UNO, und zwar in den Grenzen von 1937. Wann immer diese Wiedereinsetzung geschieht, für die wir ringen, die BRD muß und wird das Kernland des Deutschen Reichs bleiben." Sie wollen schon noch diese Grenze, nur eben dafür die Demokratie nicht aufgeben, da sie diese für die "die Überlegenheit der jetzigen Regierungsform" halten. Die politische Einstellung ist schon am ersten Satz ablesbar: "Der Erste Weltkrieg, der aus der falsch verstandenen Treue zu einem Bündnis entstand und den die Führung des Deutschen Reichs zu keiner Stunde gewollt hatte [siehe dazu Fischer-Kontroverse], sowie die Folgen des Diktats, mit dem er beendet wurde, bereiteten den politischen Boden für den Zweiten Weltkrieg vor [...] [meine Hervorhebungen]"--IP-Los (Diskussion) 19:48, 20. Jun. 2013 (CEST)

Verwandt ist die Kommissarische Reichsregierung. Das letzte Wort des ersten Absatzes fasst es ganz gut zusammen; der Artikel weist in seinen Details einen großen Nutzwert auf, so wie übrigens auch Chemtrail. --Aalfons (Diskussion) 19:57, 20. Jun. 2013 (CEST)

Und auf dieser Webseite sind die Zusammenhänge zwischen dem KRR- und dem Reichspartei-Personal dargestellt. Vor allem die Sache mit dem Orden von Saddam Hussein und den 60 Prozent Gewinn pro Monat mit Wertpapieren ist wirklich beeindruckend. --Aalfons (Diskussion) 20:16, 20. Jun. 2013 (CEST)
Nun sind aber die allermeisten siebenunddreißiger Gebiete rund ums kernige Deutschland von heute nicht gerade unbewohnte Saharadünen oder planktonbetonte Atlantikkubikmeter. Da wäre es schon interessant, zu wissen, wie diese Leutchen denn ohne Krieg diese Räume heim ins Reich holen wollen. Will man die den Polen und Tschechen und Franzosen abkaufen? Oder versetzt man einfach tausend Jahre lang every Samstagnacht die Grenzsteine um ein paar Millimeter? Oder koppelt man das an Fußballspiele (1 gewonnenes Länderspiel = 11 Quadratkilometer zurück)? Oder ist der Deutsche in der EU einfach so nett, dass er jedes Jahr ein paar Grenz-Auen zum Geburtstag bekommt von den Um-Europäern? Chantk (Diskussion) 20:17, 20. Jun. 2013 (CEST)
Ein total tuaregfeindlicher Affront. Es stimmt eben nicht, dass die Saharadünen unbewohnt sind. Sie werden nur nicht so intensiv genutzt wie das Ackerland links und rechts der Memel! Und wie das wieder heim ins Reich kommt? Durch Einsicht in die Überlegenheit der weißen Rasse, die hier zu bewundern ist. Das mit der Gewichtsabnahme im Parteiprogramm wird so auch verständlich; vermutlich wurde dieser Beschluss einvernehmlich gefällt. --Aalfons (Diskussion) 20:33, 20. Jun. 2013 (CEST)
Es steht ja immer in den Grenzen von 1937, daher man müsste "nur" einige Gebiete östlich der Oder-Neiße-Linie von Polen und Russland zurückholen und z.B. keine Gebiete von Frankreich oder der Sahara, was aber natürlich nicht viel daran ändert, dass diese Vorstellung schwer durchzusetzen wäre. --MrBurns (Diskussion) 20:38, 20. Jun. 2013 (CEST)
Na komm, wer von 1937 spricht, muss sich dann auch mal mit dem Schanddiktat von Versailles auseinandersetzen. Das heißt: Wiederherstellung des Kaiserreichs, Nordschleswig, Eupen-Malmedy und Elsass-Lothringen inklusive. Polen wieder an Russland, die Kolonien auch wieder zurück, das wird schon ... (Aber jede Wette, dass sie in ihren Machtfantasien nicht von den Hohenzollern regiert werden wollen.) --Aalfons (Diskussion) 20:49, 20. Jun. 2013 (CEST)
Wieso will eigentlich niemand das Erste Kaiserreich wiedererrichten, mit Kurfürsten und Wahlkaiser und so? --El bes (Diskussion) 22:44, 20. Jun. 2013 (CEST)
Ein relativ schwaches, zerstrittenes Gebilde von Staaten (als Beispiel: Schmalkaldischer Krieg), dessen Grenzen z. T. noch nicht einmal richtig fest waren (siehe Reichsmatrikel) bzw. mit Mitgliedern, die nur zum Teil dazu gehörten (siehe z. B. Preußen)?
@Alfons: Na ja, sie wollen eine Demokratie, was auch immer sie sich darunter vorstellen: "Die in der Folge unter Einwirkung der Alliierten entstandene demokratische Regierung war auf Jahre dem durch die vorangegangene Diktatur vergifteten geistigen Potential nicht gewachsen und brachte die Verbrüderung demokratischer Kräfte mit den Relikten nationalsozialistischer Ideenvielfalt, die sich bis heute trotz der verstrichenen sechs Jahrzehnte währender demokratischen Bemühungen beharrlich gehalten haben." Was die Grenzen betrifft: es steht zwar etwas von den Grenzen von 1937, aber als Karte dient das Deutsche Reich von 1914 - die Karte von 1937 hat man wohl nicht auftreiben können ;-) --IP-Los (Diskussion) 23:19, 20. Jun. 2013 (CEST)
Die Partei, um die es hier geht, hat doch klipp und klar geschrieben, dass sie an der "BRD" - insbesondere an ihrer derzeitigen demokratischen Regierungsform, die sich seit 60 Jahren bewährt habe - festhalten will. Diese BRD ist übrigens gerade dabei, ganz Europa zu bevormunden und zu beherrschen und zwar mithilfe des Konstrukts "Europäische Union" weit über die Grenzen von 1937 hinaus. Wenn die BRD sich stattdessen unter der Bezeichnung "Deutsches Reich" auf die Grenzen von 1937 beschränken würde, wäre das so gesehen mal ein wohltuend bescheidener Rückzug aus den Angelegenheiten fremder Völker wie den Griechen etc. --Jahrmillionenflut (Diskussion) 23:36, 20. Jun. 2013 (CEST)
Weil der Wiener Kongress zu der Einsicht gekommen ist, dass das eine verdammt schlechte Idee ist. —PοωερZDiskussion 07:24, 21. Jun. 2013 (CEST)
@Jahrmillionenflut: Daß Staaten versuchen, Einfluß auf andere Staaten ausüben, ist doch nichts Neues. Die Probleme in Griechenland sind aber z. T. nicht nur rein griechische (ohne jetzt die Maßnahmen der EU bewerten zu wollen).--IP-Los (Diskussion) 13:47, 21. Jun. 2013 (CEST)
Dem stimme ich zu: der Einflkuss der Deustchen ind er EU ist nur deshalb so groß, weil die anderen EU-Staaten das zulassen, im Prinzip hat in der EU jeder Staat die gleichen Rechte und das einzige Gremium, wo Deutschland allein von den EU-Verträgen her Einfluss hat als andere Staaten ist das EU-Parlament, aber nru wiel die Zahl der Mandatare dort einwohnerabhängig ist. Was das Zitat in deinem Beitrag von 23:19, 20. Jun. 2013 betrifft: ich würde das eher als Angriff gegen die jetztige deutsche Bundesregierung sehen, also auf das derzeitige demokratische System. Und selbst, wenn dem nicht so wäre, würde das nicht unbedingt bedeuten, dass sie gegen die demokratie wären, sie könnten auch einfach für ein anderes demokratisches System sein als das derzeitige. --MrBurns (Diskussion) 15:22, 21. Jun. 2013 (CEST)
Die anderen lassen sich von Deutschland bevormunden, weil sie sich in der jüngeren Vergangenheit von Deutschland wirtschaftlich haben korrumpieren lassen (per Euro und Euro-Schuldensalden). Und nun müssen sie die Abhängigkeit ausbaden, in die sie sich dadurch begeben haben. Ich sage nicht, dass es was Neues ist, wenn Staaten versuchen, Einfluss auf andere Staaten auszuüben. Ich weise nur darauf hin, dass es inzwischen weniger ein Zeichen von altem Größenwahn, sondern eher ein Zeichen von Bescheidenheit wäre, wenn Deutschland sich auf das Verwalten des Territoriums von 1937 konzentrieren und beschränken würde anstatt wohl spätestens nach einem EU-Beitritt der Türkei bis nach Asien hineinregieren zu wollen. Ach, da fällt mir ein - Zypern ist ja schon Asien... --Jahrmillionenflut (Diskussion) 17:49, 21. Jun. 2013 (CEST)
Griechenland hat vor dem Eurobeitritt falsche Angaben gemacht, wenn sie korrekte Angaben gemaht hätten, hätte man sie garnicht beim Euro aufgenommen. Dei anderen Länder sind auch "selbst schuld", dass sie jetzt Hilfen brauchen, weil sie sich entweder bewusst zu stark verschuldet haben (z.B. Italien), grobe Fehler in der allgemeinen Wirtschaftspolitik gemacht haben (z.B. Spanien) oder bei der Bankenregulierung versagt haben (Zypern und wohl auch Spanien). Generell zeigt sich bei den ganzen Euro-Krisen, dass eine gemeinsame Währung ohne Gemeinsame Wirtschafts- und Fiskalpolitik gefährlich sein kann. --MrBurns (Diskussion) 17:57, 21. Jun. 2013 (CEST)

Sterilisation Instrumente

hier[12] steht zur Deaktivierung von Prionen: " Dampfautoklavieren von autoklavierbarem Material bei 134°C für eine Stunde, bei 121°C für 4,5 Stunden oder bei 136°C in zwei aufeinanderfolgenden Zyklen von je 36 Minuten" Andere Quellen empfehlen, Instrumente, die bei Risikopatientgen eingesetzt wurden, nicht mehr bei Gesunden einzusetzen. Ich frage mich: Warum wird da mit so niedrigen Temperaturen herumgepfuscht? Warum nicht Autoklavieren bei 200 Grad? Natürlich sind optische Instrumente mit Gummidichtungen empfindlich, aber bei allen anderen, klassischen Edelstahlinstrumenten könnte man sicher über 150 Grad gehen. Warum wird das nicht empfohlen? --77.3.165.179 21:05, 20. Jun. 2013 (CEST)

Bei 134 °C müssen die Geräte für 2 bar Überdruck ausgelegt werden, bei 121 °C für 1 bar Überdruck. Bei 200 °C müsste Dein Dampfsterilisationsapparat für 14 bar Überdruck ausgelegt sein, also das siebenfache. [13] Dazu steigen die Anforderungen gemäß der EU-Druckgeräterichtlinie. --Rôtkæppchen68 22:45, 20. Jun. 2013 (CEST)

Warum mit Wasserdampf? Ginge nicht auch ein anderes Gas? Heißluft etwa. --Eingangskontrolle (Diskussion) 17:12, 21. Jun. 2013 (CEST)

Das macht es nicht besser Gas ist gleich Gas nur das Wasser aufgrund der hohen Wärmekapazität viel besser geeignet ist.--Saehrimnir (Diskussion) 18:28, 21. Jun. 2013 (CEST)
Dampf hat den Vorteil, dass er sich von selbst ohne Gebläse o.ä. gleichmäßig verteilt. Durch den festen Zusammenhang zwischen Dampfdruck und Temperatur erreichen alle Teile des Sterilisationsgutes die gleiche, vom Druck abhängige Temperatur. Es bilden sich also keine „kalten Nester“, die unsterilisiert bleiben. Der kritische Punkt der Lufthauptbestandteile Stickstoff und Sauerstoff liegt bei −146,9 °C und 33,9 bar (Stickstoff) bzw −118,6 °C und 50,50 bar (Sauerstoff). Die notwendigen Sterilisationstemperaturen liegen weit darüber, sodass sich bei Sterilisation mit Luft nicht von selbst überall im Sterilisationsgut die gleiche Temperatur einstellt. Die heiße Luft müsste aktiv umgewälzt werden. Alternativ müsste der Sterilisationsraum direkt aufgeheit werden, was je nach Beschaffenheit des Sterilisationsguts deutlich länger dauern kann. --Rôtkæppchen68 19:21, 21. Jun. 2013 (CEST)

problem mit Netzwerk

wie kann es sein, dass ich trotz vorhandener Netzwerkverbindung keine seiten aufrufen kann? bekomme immer nur seiten ladefehler von Firefox angezeigt. passierte aus dem nichts. alles hochfahren und vom strom nehmen half nicht.bin verzweifelt. schreive jetzt per Handy, --Marschwiese (Diskussion) 21:29, 20. Jun. 2013 (CEST)

mal n anderen Browser probieren? wurde vor Kurzem am Firewall gespielt? :-) --Heimschützenzentrum (?) 21:38, 20. Jun. 2013 (CEST)

nichts dergleichen. wie gesagt aus heiterem Himmel. eine Minute noch gearbeitet, dann ging nix mehr. --Marschwiese (Diskussion) 22:11, 20. Jun. 2013 (CEST)

Das innere LAN oder das WAN, Router rücksetzen, aber nicht löschen, nur neustarten. --Hans Haase (Diskussion) 22:56, 20. Jun. 2013 (CEST)

habe Router neu gestartet und auch vom Netz genommen, half alles nicht. --Marschwiese (Diskussion) 00:11, 21. Jun. 2013 (CEST)

beim Provider anrufen und ne Störungsmeldung aufgeben? --Heimschützenzentrum (?) 08:45, 21. Jun. 2013 (CEST)
DNS prüfen, anderen Browser probieren, ob der Fehler daran liegt. Versehentlich einen Proxy-Server eingestellt? Infizierter Computer? Rootkit? --Hans Haase (Diskussion) 19:10, 21. Jun. 2013 (CEST)

kopfnicken

Wenn ein grieche mit dem kopf nickt,bedeutet das nein oder ja ? --178.200.80.195 13:30, 21. Jun. 2013 (CEST)

Das steht im Artikel Kopfnicken. Bei Kretern wird es schwieriger (Zitat: Alle Kreter lügen). --Pp.paul.4 (Diskussion) 13:39, 21. Jun. 2013 (CEST)
es ist bei den Giechen weniger ein Nicken, als ein den Kopf leicht nach oben bewegen, manchmal auch nur mit den Augen ein bißchen nach oben schauen, schweigend, beides bedeutet "ochi" = "nein" - StephanPsy (Diskussion) 17:16, 21. Jun. 2013 (CEST)

Werden bei Meiose die Erbanlagen der Großeltern oder Eltern unterschiedlich auf die Geschlechtszellen verteilt???

Im Wiki Artikel über Gene steht beim Unterpunkt genetische Variation: "Als genetische Variation wird das Auftreten von genetischen Varianten (Allele, Gene oder Genotypen) bei individuellen Lebewesen bezeichnet. Sie entsteht durch Mutationen, aber auch durch Vorgänge bei der Meiose („Crossing over“), durch die Erbanlagen der Großeltern unterschiedlich auf die Geschlechtszellen verteilt werden." Ist das ein Druckfehler? Sollten nicht die Erbanlagen der Eltern auf die Geschlechtszellen verteilt werden, oder habe ich da was nicht verstanden?

--Tobiask2001 (Diskussion) 16:01, 21. Jun. 2013 (CEST)

Stimme dir zu. Extrem merkwürdige Verformulierung. Ich kann sie nur so erklären, dass man ausdrücken möchte, dass durch diese Vorgänge die absolute Gleichverteilung der grosselterlichen Anlagen (N-2) beim Enkel (N) nicht gewährleistet ist - also nicht 25 % + 25 % + 25 % + 25 %. Frag da mal in der Disk. nach. GEEZER... nil nisi bene 16:11, 21. Jun. 2013 (CEST)
Komisch formuliert, stimmt aber. Vater und Mutter des Kindes haben von jedem ihrer Eltern einen halben Satz Chromosomen. Diese Stammen also von den Großeltern des Kindes. Im Körper von Mutter und Vater werden die Keimzellen gebildet, diese haben nur einen halben Chromosomensatz. Welches Chromosom (von Großvater oder Großmutter) dabei ausgewählt wird ist zufällig und kann ungleich ausfallen. -- 78.42.119.215 11:53, 22. Jun. 2013 (CEST)

Wert einer Hamer Les Paul Spezial Seriennr. 2 5060

Hallo,moechte gern den Wert der obengenannten Gitarre erfahren,soll mal Gary Moore gehoert haben.LG Suzy Qu (nicht signierter Beitrag von 80.171.70.234 (Diskussion) 16:53, 21. Jun. 2013 (CEST))

Versuch macht kluch - einfach mitsteigern. --Eingangskontrolle (Diskussion) 17:27, 21. Jun. 2013 (CEST)

Wieso entstehen Windgeräusche

Ablösung von Wirbeln hinter einem bewegten Stab

(die ja durch "Hindernisse") erzeugt werden? Also Luft + Energie trifft auf Fensterritzen, Strommasten usw. Ich meine, wie geht das physikalisch vor sich, das sind ja nur Moleküle und "Molekülklumpen" mit ein ein paar Schwebstoffen im Mikrobereich (selten auch mal größere Objekte wie Sandkörnchen und Planzenteilchen, aber die lassen wir alle hier mal weg). Ganz blöde Frage: Hat das war mit der Energieabgabe zu tun (schließlich wird der Windstrom abgebremst)? --188.193.25.228 17:38, 21. Jun. 2013 (CEST)

Vereinfacht ausgedrückt: Die Moleküle haben einen Impuls, wenn der übertragen wird wirkt eine Kraft, die erzeugt eine erzwungene Schwingung. --MrBurns (Diskussion) 17:49, 21. Jun. 2013 (CEST)
Ergibt sich aus der Strömungslehre bzw. genauer der turbulenten Strömung. Siehe Animation. Dabei entsteht häufig ein meist hochfrequentes "flattern" der Strömung, ganz ähnlich wie auch "niederfrequent" bei einer Fahne in starkem Wind, an der man das "flattern" auch als solches warnimmt. --Kharon 00:48, 22. Jun. 2013 (CEST)

Zum Wesen der Integrationskonstanten

Hallo. Ein wenig rechnen kann ich eigentlich, aber wieso und wo wird eigentlich begründet, dass viele Rechenoperationen die ich an der Integrationskonstante durchführe sozusagen "verpuffen"? Im Beispiel entstehen streng genommen zunächst "zwei" Integrationskonstanten. Diese fasst man oft "unbemerkt" und ohne Begründung zu einem C auf der rechten Seite zusammen. Ähnliches gilt für oder . Per Bauchgefühl ist klar, wenn ich zu meiner unbekannten C noch 2 draufaddiere, ist sie immer noch unbekannt und ich kann es genauso gut bei "nur C" belassen. Wie drücke ich das mathematisch gegründet aus?

Vielen Dank, --WissensDürster (Diskussion) 18:43, 21. Jun. 2013 (CEST)

Dein Beispiel passt aber irgendwie nicht zur Frage, denn da kommen doch 3 verschiedene Konstanten vor, die auch 3 unterschiedliche Bezeichnungen haben: nämlich C, C1 und C2. Offenbar ist dabei C=C2-C1. Somit ist da eigentlich alles klar definiert und man deklariert nicht einfach so ein C zu einem anderen C um. --178.202.30.217 19:04, 21. Jun. 2013 (CEST)
Ich glaub er hats so gemeint: wie kann man das Beispiel so formulieren, dass man statt C, C1 und C2 nur C braucht und es trotzdem mathematisch korrekt ist? Ich denke, das geht nicht. --MrBurns (Diskussion) 19:31, 21. Jun. 2013 (CEST)
Das war von mir schon etwas aufgeschlüsselt. Ich würde eigentlich nur schreiben:

Subtrahieren, Exponentieren und Plus/Minus-Vorzeichen ändern das C nicht. Ich erhalte so auch ein nettes Ergebnis. Ist das nun mathematisch nur ungenau oder direkt falsch? Gruß, --WissensDürster (Diskussion) 19:49, 21. Jun. 2013 (CEST)

Wenn man als Physiker korrekt sein will, ohne mehrere Konstanten zu verwenden, schreibt mans nach meinen Erfahrungen so:

Ob das wirklich mathematisch korrekt ist, weiß ich aber nicht. --MrBurns (Diskussion) 20:14, 21. Jun. 2013 (CEST)
Die Variante des erklärten Weiterverwendens gefällt mir :) Ich bin dann auch auf Rechnungen gestoßen, die das C wirklich modifizieren, etwa:

Also wird eine Unterscheidung mindestens dann nötig, wenn C nicht mehr beliebig sein darf? --WissensDürster (Diskussion) 20:31, 21. Jun. 2013 (CEST)

Meine Physikstunden sind schon einige Zeit her, aber ich glaube mich daran erinnern zu können, dass mein Lehrer gerne "+const" schrub (statt +C), wenn er damit sagen wollte: "irgendeine konstante Zahl, die uns hier nicht weiter interessiert und die wir deswegen auch nicht korrekt äquivalenzumformen; ihr werdet sehen, am Ende der Rechnung fällt sie sowieso raus (oder ist zumindest immer noch irgendeine langweilige Konstante), also spar ich uns die Kreide und für mich ist ja schließlich auch schon die fünfte Stunde..." Klang nachvollziehbar, aber nicht gerade nach mathematischer Strenge. Man wusste halt, dass dasselbe "const" in jeder Zeile was anderes bedeuten konnte, falls es nicht sogar einfach ganz weggelassen wurde... --178.202.30.217 21:04, 21. Jun. 2013 (CEST)

Without Apparent Motive

hat jemand eine erklärung, warum die imdb diesen film nicht listet? ich kenne die imdb bisher als lückenlos. --Sophia 19:56, 21. Jun. 2013 (CEST)

Auch die IMDb folgt gewissen Wiki-Prinzipien. Du kannst also auch bei IMDb mutig sein und diesen Stub erweitern. Wie es geht steht hier. --Rôtkæppchen68 20:21, 21. Jun. 2013 (CEST)
Ich denke, der Film ist nie gedreht worden. Bille August, Richard Gereund Julianne Moore hatten ihre Beteiligung an dem Projekt abgesagt. [14] morty (N) 20:25, 21. Jun. 2013 (CEST)

Nachname »beim Graben«

Vor einigen Monaten haben wir hier mal die Frage erörtert, woher das »Schmoll genannt Eisenwerth« als Nachname kommt. Das Geheimnis wurde unter dem Stichwort Genanntname längst aufgedeckt. Jetzt habe ich einen neuen Namen entdeckt –»Günther beim Graben«. Woher hat dieser Mann denn seinen Nachnamen?

--vigenzo (Diskussion) 02:22, 22. Jun. 2013 (CEST)

Na doch wohl von dem Graben bei dem er oder seine Familie gelebt hat. Bahlow gibt ein ähnliches Beispiel: Beimfohr (Westf.) wird deutlich durch BeimForde "bei der Furt", wie Tomfohr oder TomForde! Vgl. Hans vorm Fohrde 1602 in Stemmen. Ähnlich Beimbrinke, Beimgraben, Beimdieke.
Bei uns in der Gegend gibt es als weiteres Beispiel den Nachnahmen tum Suden, bei dem ich mir allerdings nicht sicher bin, ob "Suden" hier auf die Himmelsrichtung weisen soll. Geoz (Diskussion) 07:39, 22. Jun. 2013 (CEST)

"Konvertiten" in den antiken Philosophieschulen?

In der Antike gab es ja 4 große Philosophieschulen, die Epikureer, die platonische Akademie, die Stoa und die damals weniger beachteten Aritstoteliker. Kam es denn damals auch vor, dass jemand von, sagen wir, einem Stoiker ausgebildet wurde, dann aber zum Platonismus überwechselte oder das ein Aristoteliker Anhänger Epikurs wurde? --188.100.28.221 08:07, 22. Jun. 2013 (CEST)

Alles was möglich ist passiert auch irgendwann. Im Zeitraum der Antike, der ja nicht gerade kurz war, ist davon auszugehen. Sucht du nach einer Liste? —PοωερZDiskussion 08:47, 22. Jun. 2013 (CEST)
Dionysios Metathemenos trägt's im Spitznamen. Grüße Dumbox (Diskussion) 09:59, 22. Jun. 2013 (CEST)
Eklektizismus, also die Übernahme von Elementen anderer Schulen, war in der Antike kein Einzelfall. Cicero wird, siehe Artikel, häufig in diesem Kontext genannt. --Aktionsheld Disk. 12:23, 22. Jun. 2013 (CEST)

Sind Goji-Beeren die gleichen wie Gou-Qi-Beeren, wie wir letztere! Beeren im chinesischen Geschäft kaufen können?

--89.183.80.16 11:33, 22. Jun. 2013 (CEST)--

Ja. Siehe Gojibeere und auch hier. GEEZER... nil nisi bene 11:48, 22. Jun. 2013 (CEST)

Word 2003: Grafik und Bildunterschrift in Textfeld platzieren

Textfeld mit Abbildung innerhalb eines längeren Textes

Liebe Auskunft, innerhalb eines mehrseitigen Textes möchte ich manche Textpassagen besonders hervorheben, indem ich sie umrandet in der Ecke einer Seite platziere. Dafür verwende ich ein Textfeld. Der übrige Text fließt um das Textfeld herum. In dem Textfeld möchte ich zusätzlich eine Grafik mit Bildunterschrift (in einem zweiten Textfeld innerhalb des ersten) platzieren. Beim Einfügen der Grafik sind die Textfluss-Optionen deaktiviert; die Grafik kann nur "Mit Text in Zeile" platziert werden. Wie kann ich Textfeld und Grafik so aussehen lassen wie in der Abbildung? Vielen Dank für Antworten! --BlackEyedLion (Diskussion) 17:23, 20. Jun. 2013 (CEST)

Ich habe das mal gemacht (aber Open Office...), indem ich eine Tabelle eingefügt habe, die aber nur 1 Zeile und 1 Spalte hat = nur 1 Feld. Dieses Feld kann man jetzt positionieren, mit Text auffüllen (= vergrössern) und auch noch ein odere mehrere Bilder einfügen, die mit den Bild-Tools gerahmt, positioniert und unterschriftet werden können. Aber es gibt sicherlich elegantere Lösungen... GEEZER... nil nisi bene 18:21, 20. Jun. 2013 (CEST)
In Word 2003 gibt es die nicht, soweit ich das sehe. Word scheint prinzipiell nicht die Möglichkeit zu kennen, dass Text innerhalb eines Textfelds um eine Grafik umbrochen wird (anders als zum Beispiel TextMaker, wo das problemlos geht). Da bleibt wirklich nur der Trick mit der Tabelle. --Jossi (Diskussion) 23:59, 22. Jun. 2013 (CEST)

Festnetz und DSL funktioniert nicht mehr

Ich habe einen Laufzeitvertrag für beides seit fünf Jahren. Nun geht seit 30.05. keins von beiden mehr. Ich habe nichts an der Konnektivität oder Software oder Interneteinstellung geändert. Der Dienstleister sagt, er habe nichts gesperrt oder abgeschaltet. Wenn ich mich auf dem Festnetz anrufe kommt die Ansage, dass die Nummer nicht bekannt ist. Internet geht auch nicht mehr. Beides über Tage hinweg probiert. Jetzt gibt es natürlich einen Streit wegen dem Geld. Seit Tagen surfe ich über UMTS-Surfstick, was bei meinen 18 Std. Onlinezeit täglich richtig teuer kommt (prepaid, weil die alte Leitung könnte ja jederzeit wieder gehen). Wie beweise ich, dass mein DSL und Internet nicht mehr geht? Muß ich mir einen Sachverständigen kommen lassen? --80.226.24.13 22:20, 20. Jun. 2013 (CEST)

Wenn Du bei der Telekom bist oder die Leitung der Telekom gehört, rufe 0800-330-1000 und frage was mit Deinem Anschluss (Rufnummer) los ist. Melde die Störung.
Wenn Du bei einem anderen Provider bist und die Leitung der Telekom gehört frage diesen über den Auftragsstatus. Melde die Störung dort. Frage nach ob es am Provider liegt. Dieser hat in wenn es so zutrifft einen Vertrag mit der Telekom, die ihm ihre Leitung zu Dir zur Verfügung stellt. Sollte dies fälschlich gekündigt sein, schadet es nicht, wenn Dein Provider noch eine Schriftlichkeit von Dir hat, mit der er dem Leitungseigentümer juristische Schritte gehen kann. --Hans Haase (Diskussion) 22:31, 20. Jun. 2013 (CEST)
(BK)Ich würde es als allererstes mit dem Entstördienst des DSL-Anbieters versuchen. Die müssen zwar oft auf den Entstördienst der DTAG zurückgreifen, können aber messen, wo die Leitung unterbrochen ist, also ob unterwegs das Kabel durchgehackt wurde oder ob Dein DSL-Router eine Macke hat. Im Vorfeld würde ich aber selbst überprüfen, ob die Kabel von Deiner 1. TAE-Dose zum Splitter, Router und zur Telefonanlage in Ordnung sind. Im Zweifelsfall leihst Du Dir von Freunden Geräte, von denen Du weißt, dass sie funktionieren, um einen Hardwareausfall auf Deiner Seite auszuschließen. Was für einen Anschluss hast Du denn? Mit Splitter, ohne Splitter, VoIP, Analogtelefon, ISDN? Wenn Du einen Spannungsmesser hast, kannst Du an der 1. TAE-Dose mal die Spannung messen. Bei einem Analoganschluss sollten ca. 60 Volt anliegen, bei ISDN 96 Volt und bei VoIP weiß ich es nicht. Wenn Du ISDN hast: Leuchtet auf dem NTBA das grüne Lämpchen? Wenn Du einen Analoganschluss hast: Schließe ein Analogtelefon (kein DECT) direkt an den Anschluss an. Knackst es im Hörer, wenn Du den Gabelumschalter des Telefons manuell betätigst? --Rôtkæppchen68 22:33, 20. Jun. 2013 (CEST)
VOiP wie DSL, das sind nur Daten. Der Router wurde mal stromlos gemacht, nein, so mach das zuerst. Danach kannst Du die Hotline getrost nerven, die können die Leitung teilautomatisch testen/messen und sehen was los ist. --Hans Haase (Diskussion) 22:38, 20. Jun. 2013 (CEST)
Noch eine Möglichkeit: Dein Provider hat den Leitungseigentümer nicht bezahlt. Das wird Dir der Leitungseigentümer nicht sagen, aber auf das Vertragsverhältnis bzw. die Bereitstellung kannst Du mal anklopfen. Wenn Du bezahlt hast und Dein Teil hinter der Dose/oder Übergabepunkt auf Deine Infrastruktur ok ist, mache es sobald Du das sicherstellen kannst und keine Auskunft bekommst schriftlich. Frage, und prüfe die Antworten bzw. hinterfrage sie, wenn Du nicht weiterkommst. Sei grausam. --Hans Haase (Diskussion) 22:45, 20. Jun. 2013 (CEST)
Nochwas: Fehler die Administrativ liegen kommen auch gerne vor, wenn Du gekündigt hast und die Kündigung wiederrufen, also weiter dabei bleibst. Dein Provider reicht die Kündigung der Leitung gerne recht früh weiter und die nächste Schriftlichkeit geht schonmal verloren. --Hans Haase (Diskussion) 22:49, 20. Jun. 2013 (CEST)
Noch etwas: auch wenn Du garantiert nichts an der Verkabelung geändert hast, ist es empfehlenswert, jeden Stecker zu ziehen und wieder einzustecken, da sich irgendwo eine kleine Oxidschicht oder ein Wackelkontakt gebildet haben könnte. Durch die Reibung beim Aus- und Einstecken können sich die Kontaktverhältnisse verbessern. --Rôtkæppchen68 22:57, 20. Jun. 2013 (CEST)
Worin der Schwachpunkt der TAE-Dose liegt. Bei 3fach Dosen wie NFN oder NFF oder in Reihe geschaltete Dosen liegt innen je Steckplatz der Kontakt nur auf. Pins 1-6 und 2-5. Der Stecker reibt bei ein und Ausstecken, aber die Durchschleifung ist ein gegen Staub, Wandfarbe usw. offener Schalter und eine Fehlerquelle. Modularstecker haben diese Eigenschaft nicht, sie schleifen überhaupt nicht durch. Die Reihenschaltung muss dort über die Geräte hergestellt werden (2 Buchsen: LINE, PHONE an Fax, Modem, AB). --Hans Haase (Diskussion) 09:06, 22. Jun. 2013 (CEST)
Modularstecker sind darüberhinaus in den allermeisten Fällen vergoldet, TAE-Stecker nicht. Das macht viel aus. --Rôtkæppchen68 12:45, 22. Jun. 2013 (CEST)
Klar bei der Fläche der Kontakte. --Hans Haase (Diskussion) 14:08, 23. Jun. 2013 (CEST)

Kotzbrocken ...

.... fehlt noch in der Kategorie Schimpfwort. Ich interpretiere es mal als "Jemand der einem immer wieder mal unangenehm aufstösst". Wenn ich suche, komme ich nur in die 1980er Jahre. Mache ich etwas falsch oder ist es wirklich erst ein so relativ neuer Begriff? Kennt jemand einen Urheber dieser anschaulichen Beschreibung? Wussten Goethe oder Luther - der Mann des drastischen Wortes - , was ein Kotzbrocken ist? GEEZER... nil nisi bene 11:24, 21. Jun. 2013 (CEST)

Ich meine mich zu erinnern, das Wort öfters im Umfeld der Schnodderdeutschsynchronisationen ala Rainer Brandt und Co. der 1960er/70er gehört zu haben, hab allerdings gerade kein Beispiel parat. --213.39.225.44 12:14, 21. Jun. 2013 (CEST)
Der Ngram Viewer führt uns an den Anfang der 1960er Jahre... --84.191.139.148 15:16, 21. Jun. 2013 (CEST)
GB-Suche mit "kotzbrocken 1964" ergibt Hinweise auf Der Spiegel, , Band 26,Ausgaben 24-27, S. 64, Spiegel-Verlag, 1972 und möglicherweise Elfriede Jelinek, Die Liebhaberinnen, 6, Neuauflage, Rowohlt, 1975. --84.191.139.148 15:33, 21. Jun. 2013 (CEST)
DER SPIEGEL 49/1970, 30.11.1970, Über den Film "Das Biest muß sterben" von Chabrol, der im "Ersten Programm" ausgestrahlt werden soll: "Er wählt dazu stets Sujets aus dem "moribunden Bürgertum". In seinem "Biest" beispielsweise agiert ein gutverdienender Provinz-Unternehmer, der im Monat "allein 1000 Franc für den Fleischer ausgibt", als "Karikatur des vollkommen schlechten Menschen, wie man ihn nie anzutreffen hofft". Dieser "Kotzbrocken" (Dialog) hat den einzigen Sohn eines Schriftstellers totgefahren und noch dazu Fahrerflucht begangen. Der haßerfüllte Vater plant und bewerkstelligt nach minuziösem Plan die Ermordung des Verbrechers, indem er dem bourgeoisen Unhold Rattengift in dessen "Scheißeritis"-Arznei schüttet." --84.191.139.148 15:46, 21. Jun. 2013 (CEST)
In DER SPIEGEL 17/1972, 17.04.1972 wird Klaus Rainer Röhl von seinem Kumpel Peter Rühmkorf als "Kotzbrocken" geschildert. Zitat:
Rühmkorf schildert Ulrike Meinhof-Röhls einstige -- von ihr durchaus bewältigte -- Doppelrolle als "unerbittliche Gesellschaftskritikerin" einerseits und als "Lieblingskind", als "verhätschelte Ausnahmeerscheinung" und "gern herumgereichtes Exotikum" eines Hamburger "'Establishment' etwas links vom ESTABLISHMENT" andererseits: "Während man ihn (Röhl) als unvermeidlichen Kotzbrocken mit in Kauf nahm, zog man sie liebreich an die Brust und schmückte sich mit ihr; und sie schmückte sich für die Gesellschaft und trug zum Gloria-Modellkleid gern das handgehämmerte Skoluda-Gehänge." --84.191.139.148 15:52, 21. Jun. 2013 (CEST)
Die Suche im Spiegel ab 01.01.1960 ergibt den Chabrol-Artikel als frühesten Treffer. --84.191.139.148 15:56, 21. Jun. 2013 (CEST)
Dürrenmatt wird schon am 07.02.1964 in der Zeit als Kotzbrocken tituliert. In der Zeit offenbar der früheste Treffer. Autor des Briefes ist wohl Italo Svevo, wie sich aus dem ersten Brief ergibt (DIE ZEIT, 24.1.1964 Nr. 04) --84.191.139.148 16:10, 21. Jun. 2013 (CEST)

(Ich bin Psychologe von Beruf) Als deutscher Muttersprachler hat man es bei der Betrachtung von Schimpfworten einerseits relativ schwer, weil die deutsche Sprache vergleichsweise wenige Schimpfworte kennt (anders als z.B. die slavischen Sprachen, und damit meine ich nicht nur die an Schimpfworten sehr reiche russische Sprache). Auf der anderen Seite hat man es bei der Interpretation der Schimpfwörter relativ leicht, weil die Deutschen die Schimpfworte relativ passgenau einzusetzen, d.h. mit dem Schimpfwort nicht nur eine Herabsetzung, sondern (unbewusst) eine ziemlich genaue Beschreibung vorzunehmen scheinen.

Nehmen wir mal das Beispiel "Arschloch" oder andere, auf anales hinweisende Schimpfwörter. Hiermit werden meiner Beobachtung nach Personen beschimpft, die vor allem analfixierte oder analregredierte Eigenschaften aufweisen, also ihren Gegenüber mit überzogener Kleinlichkeit, Regel-Reiterei, Kleinkariertheit, aber auch Rücksichtslosigkeit aufweisen oder z.B. für einen eigenen winzigen, meist irrationalen Vorteil einen großen Nachteil beim Schimpfenden inkauf nehmen. Also eine Regelreiterei, die als eine gewisse Graumsamkeit" empfunden wird. Das aber, wenn der Beschimpfte durch seine Handlung keinen tatsächlichen oder materiellen Vorteil erlangen will, sondern wenn er eher auf kleinliche Regeln besteht, die vom Schimpfenden in der speziellen Situation als vernachlässigbar angesehen werden.

Beim "Kotzbrocken" ist aus dieser Sicht eine orale" Eigenschaft nicht übersehbar, also wenn z.B. der Beschimpfte eher eigensüchtig handelt, als aus tatsächlich eigenen materiellen Motiven handelt. Er erzielt durch seine Handlung einen echten Vorteil. Er handelt (wiederholt) selbstsüchtig, und das empfindet der Schimpfende "zum Kotzen".

Kann sein, dass ich mich irre, aber nach vielen Jahren der Beobachtung habe ich es oft so vorgefunden.176.5.138.51 16:51, 21. Jun. 2013 (CEST)

Tip bei tieferem Interesse: Martin Ebel: Fäkal, nicht sexuell. dlf Büchermarkt, 8. Januar 2013 (über Hans-Martin Gauger: "Das Feuchte und das Schmutzige", C.H.Beck). Dort heißt es sehr schön: „Auch das vulgäre Vokabular gehört zu unserem kulturellen Rucksack.“ --84.191.139.148 17:14, 21. Jun. 2013 (CEST)
Das Wort war schon vor dem 2. Weltkrieg bekannt:
Johannes R. Becher, Levisite oder der einzig gerechte Krieg (1925):
"So ein Krieg..."
"Kotzbrocken..."
"Klamottenkrieg..."
Erich Kästner, Drei Männer im Schnee: "Es waren zwei Kotzbrocken, zugegeben."--IP-Los (Diskussion) 22:20, 21. Jun. 2013 (CEST)
Danke für die Hinweise, ich werde speichern und weitersammeln. Mit "vor dem Krieg" ist interessant. Ich hatte wegen des drastischen Ausdruckes auch Nähe zum Militär vermutet. Dass Schimpfworte ihr kulturelles Eigenleben haben, ist ein interessantes Feld. Bei K. assoziiere ich auch, dass man es zwar runterschlucken kann, es kommt aber praktisch unverändert (Brocken) wieder hoch. Übers Reden gibt es ja viele Schimpfwörter aber K. passt da nicht rein und hat irgendwie Sonderstellung. Speichellecker kommt in den Sinn, meint aber etwas ganz anderes. GEEZER... nil nisi bene 13:55, 22. Jun. 2013 (CEST)
IP-Los ist bei solchen Wettbewerben nur sehr selten zu schlagen. Spannend wäre sicher noch, ob das Wort aus den Schützengräben des Weltkrieges mitgebracht wurde, also aus der Soldatensprache gewachsen ist. Was aber die Beschreibung von "Kotzbrocken" angeht, bin ich mit der Formulierung: "Jemand der einem immer wieder mal unangenehm aufstösst" nicht konform. Von "immer wieder mal" kann bei einem Kotzbrocken keine Rede sein. Das ist ein sehr unangenehmer Mensch und das ist deutlich zu erkennen. Die bildhafte Genesis dieser (Ab)Qualifizierung (nach meinem Sprachgefühl eher nicht Beschimpfung) ist ja so deutlich wie etwas nur sein kann. Es geht um eruptiv ausgewürgte unverdaute Speisereste, also die Brocken in dem, was sonst noch so hochkam. Die (Ab)Qualifizierung geht (der Scherz sei erlaubt :) noch ein Stückchen weiter als: "Der ist zum Kotzen", hin zu einem signifikanten, deutlich erkennbaren Teil oder Bestandteil des unangenehmen Resultates der Übelkeit. Ich behaupte mal theoriefindend, daß es genau diese noch identifizierbaren Teile oder Brocken des Mageninhaltes sind, die das Gesamtergebnis noch unangenehmer erscheinen lassen als eine nur diffuse übelriechende Brühe. Dies umso mehr, wenn man noch Rückschluß auf die letzte Mahlzeit ziehen kann ("Diese Pizza hat er wohl nicht so gemocht"). In Two and a Half Men, in der Staffel 6, Folge 3 (Damn You, Eggs Benedict, deutsch: "Rührei" wird dieser Umstand sogar noch auf die Spitze getrieben, wenn der zum ersten Mal betrunkene, vor der Toilettenschüssel knieende minderjährige Jake Harper gegenüber seinem Onkel sinniert: "Sieh dir den Shrimp an, den könnte man abwaschen und für neu verkaufen." (etwa ab 1:00). --84.191.135.123 21:46, 22. Jun. 2013 (CEST) Da ich kritisiert wurde: Mit "Weltkrieg" ist selbstverständlich der Erste Weltkrieg gemeint, kein anderer Weltkrieg war "der Weltkrieg"... --84.191.187.101 13:06, 23. Jun. 2013 (CEST)

Profilzylinder

Schlosszylinder werden bei den Händlern immer mit zwei Angaben für die Länge vor" und hinten" versehen, z.B. 30/55 für einen Zylinder, der auf der einen Seite mit 30mm, auf der anderen Seite mit 55mm bündig abschließt. Um ein optimal passendes Teil bestellen zu können, habe ich meinen derzeit im Schloss steckenden Zylinder ausgemessen. Dabei bin ich aber auf ungerade Ergebnisse gekommen.

Von w0o aus muss gemessen werden? Von der Mitte der Bohrung der Befestigungsschraube oder vom Ende des beweglichen inneren "Rotors" des Zylinders? Ich frage deshalb, weil das derzeitige Schloss bei beiden Messungen nicht auf eine Zahl kommt, die serienmäßig angeboten wird. Es dürfen außen beispielsweise nur 27mm werden, damit das Schloss noch in das Schließblech passt. Die geringste Zahl bei den Händlern beträgt aber 30mm. 176.5.138.51 16:31, 21. Jun. 2013 (CEST)

Es gibt aber auch für besonders kurze Abmessungen Kurzzylinder in den Längen 22/22mm, 27/27mm, 27/33mm, 27/35mm und selten auch in der Länge 27/40mm. Mal einen anderen Händler fragen. Hier noch ein Tip zum ausmessen A+B --Dos Vientos (Diskussion) 16:56, 21. Jun. 2013 (CEST)
Die Länge wird immer von der Mitte des Stulpschraubengewindes gemessen. --Rôtkæppchen68 17:30, 21. Jun. 2013 (CEST)
Lieben Dank für Eure Antworten. :-) Also gemessen in meinem Fall sind am alten Zylinder 27 mm am - von Bohrungsmittelpunkt zum Ende des Zylinders -, der hier plan abschließt. Meine Frage ist nun, welches der generelle standardmäßig angegebene Endpunkt bei "jedem" Zylinder ist. Bei dem von mir favorisierten Produkt "BKS Janus" - dem m.E. einzigen (?) Produkt mit Stiftung-Warentest-Bewertung "sehr gut" - ist es so, dass die Enden der "statischen Fassung" im Ggs zu allen anderen Produkten bautechnisch gerundet sind, wobei der innere drehbare Teil auf den Abbildungen wieder herum noch einmal weiter vor steht. Die Differenz sieht nach ca. einigen mm aus. Ich frage deshalb, weil mein Schließblech außen die maximale Länge begrenzt und wahrscheinlich keine noch so kleine Toleranz zulässt. Soll ich also 30mm oder 27 mm bestellen? (Das ist ein Preisunterschied von 62,- zu 113,- € bei ebay, im Fachhandel bis 142,-€ ) 176.5.138.51 18:04, 21. Jun. 2013 (CEST)
Lad Dir von der Herstellerwebsite die passende Maßzeichnung runter. Aus dieser sollte die Antwort auf Deine Frage hervorgehen. --Rôtkæppchen68 18:18, 21. Jun. 2013 (CEST)
Hat der Zylinder eine Not- und Gefahrfunktion, also kannst du damit zB von außen aufschließen obwohl innen der Schlüssel steckt? Nette Funktion das. 85.177.234.245 18:48, 21. Jun. 2013 (CEST)
Ja, Gefahrenfunktion hat er serienmäßig, weil sonst Schlüsseldienst-Einsätze bei der statistisch häufigen Ursache (innen stecken gelassener Schlüssel) zu häufig und zu häufig teuerst wären. Ohne das würde ich ihn auch nicht kaufen. Aber ich will im Urlaub einfach sicher sein. Der Fachmann sagt, dass der Zylindertyp faktisch unknackbar sei und dass er Janus-Zylinder gesehen habe, bei denen Einbrecher vergeblich versucht hätten, ihn mit der Bohrmaschine zu zermüllern.[15]. Er ist eben nur recht teuer. Picken könne man ihn generell nicht wegen der (dem Einbrecher im konkreten Fall unbekannten) 3d-Anordnung der Stifte. Außerdem sei er aufgrund des runden Innenlebens auch unempfindlich gegen den Einsatz von Hebelwerkzeugen, weil die durchdrehen würden. Die Einbrecher versuchen es außerdem bei diesen Türen seltener, weil sie in Eile sind und Mehraufwand scheuen. Sie rechnen wohl auch ökonomisch und gehen lieber zum Nachbarn, der keinen solchen Zylinder haben und bei dem sie deshalb schneller drin sind. 176.5.138.51 19:41, 21. Jun. 2013 (CEST)
Kommen Einbrecher nicht bevorzugt durchs Fenster? -- Janka (Diskussion) 20:36, 21. Jun. 2013 (CEST)
Ach, das gekippte? Oder die Balkon/Terasssentür, die sicherheitstechnis noch schlechter ist? Mit derHaustür fängt man wohl als Einbrecher-Azubi an. -84.148.29.18 21:18, 21. Jun. 2013 (CEST)
Vergiss das Blafasel Deines Händlers. Wer reinkommen will, kommt rein - und zwar innerhalb von Sekunden (https://www.youtube.com/watch?v=LrkcGgGwQKc) und ohne grobes, lautes Werkzeug wie eine Bohrmaschine. Bohrmaschinen verwenden nur die Möchtegern-Schlüsseldienst-Notfall-Abzocker, die Dir danach einen neuen Zylinder verkaufen wollen. -- 149.172.200.27 22:53, 22. Jun. 2013 (CEST)

Religionsfreiheit mal andersrum

Es gibt recht viele Fälle im Internet zu finden, die sich damit beschäftigen ob ein Arbeitsnehmer ein religiösen Objekt (z. B. Kette) während der Arbeit tragen darf, respektive umgekehrt, ob der Arbeitsgeber ihm das verbieten kann.

Nun mal die umgekehrte Situation: Kann der Arbeitsgeber eine Arbeitskleidung anordnen, die dem Glaube des Arbeitsnehmer widerspricht?

Um das Ganze nicht so allgemein zu halten: Beim Arbeitsgeber handelt es sich um eine Brockenstube der Heilsarmee, beim Objekt um einen Arbeitskittel mit Heilsarmee Logo und das Arbeitsverhältnis basiert auf einen Zivildiensteinsatz. Kann der Zivildienstleistende mit Begrüdung auf die Religionsfreiheit das Tragen der Arbeitskleidung verweigern? --77.239.58.89 22:04, 21. Jun. 2013 (CEST)

Mal vorsichtig zurückgefragt: Auf welches Land bezieht sich die Frage? --Rudolph Buch (Diskussion) 22:29, 21. Jun. 2013 (CEST)
Und welches religiöse Gefühl / welches Gebot wird durch diese Arbeitskleidung konkret verletzt?feba disk 22:34, 21. Jun. 2013 (CEST)
Es handelt sich um die Schweiz. Betreffend religiöses Gefühl: Die Heilsarmee ist eine Freikirche, der Zivildienstleistende jedoch religionslos und er möchte nicht mit der Religion in direktem Zusammenhang gebracht werden. --77.239.58.89 22:42, 21. Jun. 2013 (CEST)
das widerspricht sich aber etwas: ein bekennender Religionsloser kann per definitionem ja gar keine "religiösen Gefühle" haben. - andy_king50 (Diskussion) 22:47, 21. Jun. 2013 (CEST)
(BK) das wär doch aber unlogisch oder inkonsequent: Wenn der Religionslose kein Problem damit hat, aktiv für diese Freikirche zu arbeiten, dann kann er doch auch kein Problem damit haben, deren Arbeitskleidung anzuziehen... Das wäre ja sonst wie ein Kriegsdienstverweigerer, der aus Gewissensgründen nur die Arbeitskleidung "Uniform" verweigert, ansonsten aber kein Problem hat, mit den anderen Soldaten mitzuballern .. --178.202.30.217 22:56, 21. Jun. 2013 (CEST)
@Andy Religionsfreiheit beinhaltet nicht nur Freiheit der Religion sondern auch Freiheit von Religion. —PοωερZDiskussion 23:33, 21. Jun. 2013 (CEST)
Eine Brockenstube der Heilsarmee ist ja einem Wohltätigkeitsbasar einer Kirchengemeinde ähnlich - da werden alte Sachen verkauft und der Erlös geht in soziale Zwecke.
Zunächst mal wird der unbefangene Beobachter sich vielleicht denken, dass der dort Tätige ehrenamtlich tätig ist. Aber mal davon abgesehen, schreiben wir doch 2013 - auch auf dem evangelischen Kirchenbasar wird mal ein Katholik mitarbeiten oder umgekehrt und niemand wird sich heute mehr darüber aufregen, oder? Religionskrieg ist vorbei. Wer nun dennoch auf gar keinen Fall mit dem Organisator des Wohltätigkeitsverkaufs - oder -geschäfts in irgendeine Beziehung gebracht werden will (beispielsweise weil die 98-jährige Großmutter der neuen Freundin so konservativ ist ist, dass sie nie einen Katholiken, Moslem, Heilsarmeeler akzeptieren könnte), der geht da eben nicht arbeiten.
Hier Zivildienst? Da soll er sich nicht so haben, andere Zivis müssen schwerkranke Alte pflegen (waschen, et c.) oder ähnliches. Da ist wohl das Verkaufen in einem Basarladen zumutbar - die Heilsarmee wird ihn nicht missionieren. Und es gibt immer eine prima Alternative - ab zum Militär! Csedfa (Diskussion) 23:02, 21. Jun. 2013 (CEST)

Das dürfte mal wieder von der Gesinnung des Richters abhängen. Letztlich ist nur so viel zu sagen: Du solltest nicht in einem Betrieb arbeiten, dessen Geschäftspraktiken du ablehnst. Geh halt dann zum Roten Kreuz.--Antemister (Diskussion) 23:16, 21. Jun. 2013 (CEST)

Laut Wikipedia darf sich übrigens der Schweizer Zivi seine Einsatzbereiche selbst aussuchen (das Glück hatte und hat man nicht in jedem Staat!) und - hopp Schwyz! - er bekommt eine Uniform: Hier steht das.
Und es gibt heute keinen Grund mehr für diese religiösen Berührungsängste. Du kannst heute auch zum Papst sagen Hallo Herr Bergoglio, ich freue mich, Sie kennenzulernen, und bin Atheist. Csedfa (Diskussion) 23:28, 21. Jun. 2013 (CEST)
Das konnte man immer schon, die Frage ist nur wie oft. —PοωερZDiskussion 23:36, 21. Jun. 2013 (CEST)
Ja, natürlich nur einmal - der Mann hat ja eine Menge zu tun... ...abgesehen davon, dass keiner der Vorgänger Herr Bergoglio hieß. Csedfa (Diskussion) 23:38, 21. Jun. 2013 (CEST)
Muss man denn alles erklären? Die Frage ist, ab wann man nicht mehr dafür hingerichtet wurde. —PοωερZDiskussion 02:20, 22. Jun. 2013 (CEST)
Machen wir's doch so: Du sagst uns, wann man für Hallo Papst, ich bin Atheist noch hingerichtet wurde. Nicht mehr hingerichtet dafür wurde man ziemlich sicher so in den letzten 50 Jahren - beim Konzil lag der Dialog nicht nur mit den anderen Konfessionen und Religionen, sondern auch mit atheistischen Weltanschauungen schon sehr in der Luft, wenn man ihn damals auch noch eher vorsichtig aufs Papier tropfen ließ. Nur der Schweizer Zivi hat es noch nicht so ganz mitbekommen, dass wir heute volentes oder nolentes in einer multireligiösen Welt leben, in der die Religionen einander und die Atheisten tolerieren müssen, aber eben gelegentlich auch mal umgekehrt. Csedfa (Diskussion) 19:30, 22. Jun. 2013 (CEST)

film brennen für eine zweite person

Darf man für eine zweite person ein film brenne wenn man für den film bezahlt hat?

wybi (nicht signierter Beitrag von 79.226.243.54 (Diskussion) 22:28, 21. Jun. 2013 (CEST))

In Deutschland: Ja. Es muss dein Freund sein (für "irgendjemand" ist wohl verboten), du darfst den Film nicht öffentlich anbieten, du darfst kein Geld verlangen, und du darfst keinen Kopierschutz umgehen. Siehe Privatkopie. --78.34.9.197 22:45, 21. Jun. 2013 (CEST)
Ich glaub die Kosten vom Rohling darf man sich schon erstatten lassen. Man darf nur nichts dran verdienen, de jure ist es wohl schon illegal, 50 Cent zu verlangen, wenn der Rohling 49 Cent gekostet hat (außer vielleicht wenn man sich den Energiebedarf fürs Brennen ausgerechnet hat und der eine Cent dafür ist). --MrBurns (Diskussion) 11:43, 23. Jun. 2013 (CEST)

Dänischer "Stinkekäse"

Warum riechen die meisten dänische Käsesorten so streng? --80.226.24.6 23:01, 21. Jun. 2013 (CEST)

Scheinen bevorzugte traditionelle Lebensmittel der skandinavischen Länder. Siehe auch Surströmming oder Hákarl. --Kharon 00:34, 22. Jun. 2013 (CEST)
Die meisten? Welche der vier Wikipedia bekannten dänischen Käsesorten Danablu, Esrom, Havarti und Samsø (Käse) stinken denn? --Rôtkæppchen68 00:58, 22. Jun. 2013 (CEST)
Also, Esrom hieß in meinem Haushalt immer nur "Stinkerkäse".feba disk 02:55, 22. Jun. 2013 (CEST)
Gestank ist leider eine sehr subjektiv gefärbte Wahrnehmung. Eine vernünftige Antwort könnten höchstens systematische Testreihen liefern, in denen speziell geschultes Testpersonal die dänischen Käsesorten mit genormten Geruchsobjekten "eichen" (siehe Olf) und dann auch mit Sorten aus anderen Ländern vergleichen. Rein anekdotisch: In Frankreich soll es Käsesorten geben, über denen die kreisenden Fliegen betäubt abstürzen... ;-) Geoz (Diskussion) 07:55, 22. Jun. 2013 (CEST)
Im Volksmund tue-mouche :-) Doch wir haben einen Superhelden, der damit fertig wird - die Dänen nicht .... GEEZER... nil nisi bene 09:54, 22. Jun. 2013 (CEST)
Den hier nicht zu vergessen - Käse hilft auch gegen Piraten --Potisiris (Diskussion) 10:28, 22. Jun. 2013 (CEST)
BK Das mit dem Stinken ist relativ. Jede Gegend riecht anders, die Nase stellt sich auf diesen "Grundgeruch" ein. Aus diesem Grunde riecht ein Käse (Mensch, Tier, Blume etc.) je nach Umfeld anders. Diese Konditionierung der Nase ist auch dafür verantwortlich, dass eigene Ausdünstungen und Auslässe beim Verursacher selbst besser ankommen. Will damit sagen, dass ein Harzer (Käse) im Bereich eines Güllesilos ganz passabel riechen kann. --79.232.206.173 10:42, 22. Jun. 2013 (CEST)

Um die Ehre der Dänischen Küche zu retten, mein Käselexikon kennt 10 Dänische Käsesorten. Das Lexikon definiert das Aroma von Esrom als "kräftig pikant". Ansonsten gibt es kein gemeinsames "Merkmal" oder eine Zutat, welche zu erhöhter Geruchsentwicklung gerade dieser nationalen Käsesorten führt. Ich vermute eher, daß der Fragesteller durch die industrielle Einheitssuppe von Gouda/Emmentaler/Butterkäse keine Erfahrung in der Charakteristik von gutem Käse hat. Und in einem Land, daß Produkte wie Tilsiter, Limburger und Harzer hervorgebracht hat, braucht man sich eigentlich nicht über die Nachbarländer Gedanken machen.Oliver S.Y. (Diskussion) 10:49, 22. Jun. 2013 (CEST)

Noch ein Detail: Nach kurzer Recherche dürfte Gamle Ole ost (also Käse der Marke Alter Olaf) das dortige Non plus ultra sein. Wird beim nächsten mal auf dichtem, dunkelen Roggenbrot probiert... GEEZER... nil nisi bene 10:51, 22. Jun. 2013 (CEST)
Ich habe mal gesucht, ob es Möglichkeiten gäbe, herzhaften Käse objektiv (= quqntitativ) nach Geruchsherzhaftigkeit einzuordnen. Eine Gruppe Japaner hat das mit Stinky Tofu ("chinesischer Käse") durchgeführt, indem sie die volatilen Substanzen identifiziert und quantifiziert haben. Jetzt fehlt nur noch jemand der die Grant Application "Analysis, Standardization and Grading of Smelliness of Cheese by GC/MC - A European-wide Study including Denmark." schreibt... GEEZER... nil nisi bene 13:39, 22. Jun. 2013 (CEST)
Hej, Überraschung! Weil das Wort sogar im Duden steht, haben auch wir das Lemma Stinkkäse! Hätte ich jetzt nicht gedacht! Davon abgesehen, dass dem Artikel ein paar intersubjektiv reproduzierbare Daten nicht schlecht bekämen: Dänemark findet in diesem Zusammenhang schon mal keine besondere Erwähnung (in der Tat gar keine, wie auch kein anderes Land). Geoz (Diskussion) 14:21, 22. Jun. 2013 (CEST)
Meinst du, dass der Artikel etwas Chemie vertragen könnte? Z.B. Substanzklassen, die zu herzhaftem Geruch/Aroma beitragen ... und 2-3 Beispielen pro Substanzklasse...? Oder interessiert das Karl-Heinz nicht ...? GEEZER... nil nisi bene 15:45, 22. Jun. 2013 (CEST)
Da ist natürlich ein Hintergedanke dabei: Was ist, wenn es zwar nicht dasselbe Bakterium wie bei den Schweissfüssen ist - aber eine oder mehrere der produzierten Substanzen ist/sind exakt dieselbe(n)? Au weia, der Herzhafte-Käsemarkt schwankt..!? GEEZER... nil nisi bene 16:03, 22. Jun. 2013 (CEST)
Dazu ein Zitat aus unserem Artikel Ekel: Wie Menschen auf einen Geruch reagieren, hängt vor allem vom Kontext und den persönlichen Erfahrungen ab. Die Mehrheit der Forscher geht davon aus, dass die Bewertung von Gerüchen kulturell erworben wird. „[…] smell is in the nose of the smeller, but also in the culture of the smeller.“ Bei Labortests mit unbenannten Geruchsproben kommt es so zu ganz unterschiedlichen Reaktionen auf dieselbe Probe: „Manche Probanden, die nicht wissen, was sie schnüffeln, halten den Geruch von Erbrochenem für Käse – leckeren Käse. Geoz (Diskussion) 18:48, 22. Jun. 2013 (CEST)
Ein gemeinsamer Nenner ist Buttersäure. Es ist auch ein bekanntes Phänomen, dass einige Substanzen in (relativer) Verdünnung als angenehm, bei höherer Konzentration aber als unangenehm wahrgenommen werden. Natürlich auch individuell verschieden... GEEZER... nil nisi bene 11:54, 23. Jun. 2013 (CEST)

GTD umsetzen

Hallo! Als Student fehlt mir momentan noch das passende System zur Arbeitsorganisation. GTD scheint mir da ein guter Schritt in die richtige Richtung zu sein. Mir stellt sich nun die Frage, wie ich, ohne Unsummen für Systeme wie Classei auszugeben, meine Sachen geordnet bekomme. (Die deutsche Übersetzung "Wie ich die Dinge geregelt kriege" ist dazu nicht wirklich brauchbar.) Kann mir jemand empfehlen, wie ich am besten vorgehe? Im Netz finde ich keine vernünftige Anleitung sondern nur fast Diskussionen um Marken und Kleinigkeiten. Eingangskörbe habe ich. Für Links und Tipps wäre ich dankbar. --141.78.40.105 15:41, 22. Jun. 2013 (CEST)

Wie hab ich nur mein Studium ohne "GTD" o. ä. geschafft? Na, egal.
Sowas für DIN A4 hab ich mal im Ein-Euro-Shop fünf mal so viele für ein Bruchteil des Geldes bekommen, das sie im Papierwarenladen wollten. Vielleicht hilft's ja.
--Eike (Diskussion) 17:20, 22. Jun. 2013 (CEST)
Zur Frage: Ich habe meinen Organizer befragt, der verwies mich auf Google ... und da kam => "getting things done" "freeware" => das hier raus - 3 Freeware-Organizer....
Zum Prinzip: Ist Studieren heute ehrlich so viel komplexer geworden? Muss an den Nebengeräuschen liegen ... :-) GEEZER... nil nisi bene 18:56, 22. Jun. 2013 (CEST)
Wie hab ich nur ein Studium geschafft, ohne je von GTD gehört zu haben?--Antemister (Diskussion) 23:09, 22. Jun. 2013 (CEST)

Ich verwende seit über 30 Jahren ein ähnliches (selbsterfundenes) System und verwalte das über einige Word-Dateien. Klappt anscheinend ganz ordentlich.--80.129.213.11 12:00, 23. Jun. 2013 (CEST)

Fernsehbilder aus einem Berliner Café

Vor wenigen Minuten (ca. 19:45) soll auf irgendeinem TV-Sender eine Szene aus einem Berliner Café oder so gezeigt worden sein. Hat das zufällig jemand gesehen und kann mir jemand den Sender nennen? Ein Freund von mir soll darin zufällig gezeigt worden sein. 92.231.188.29 19:55, 22. Jun. 2013 (CEST)

Ich habs nicht gesehen, aber evtl. lohnt sich ein Blick aufs TV-Programm von 19:45. Fiktionale Programme können eigentlich ausgeschlossen werden, da man dort (auch von Statisten) das Einverständnis braucht.
  • ARD: Liga-Fieber – Unsere Bundesliga wird 50 (unwahrscheinlich)
  • ZDF: Der Bergdoktor (fiktional)
  • RTL: Explosiv – Weekend
  • Pro7: Galileo - Grill-Quiz
  • Sat1: K 11 – Kommissare im Einsatz (fiktional)
  • RTL2: Mel Gibson – Der Patriot (fiktional)
  • Kabel1: Rosins Restaurants – Ein Sternekoch räumt auf! (nicht in Berlin)
  • VOX: Die Pferdeprofis (scheint mir unwahrscheinlich in Berlin)
  • Tele5: Smallville (fiktional)
  • arte: 360° – Geo Reportage (Thema: Tonga, Paradies der Südsee? -> unwahrscheinlich)
  • 3sat: Exclusiv – Die Reportage
  • SuperRTL: Übergang von Disney Phineas und Ferb zu Disney Meine Schwester Charlie (beides fiktional)
  • ServusTV: Weltwunder der Technik (Thema: Der Panama Kanal -> unwahrscheinlich)
  • Das Vierte: Teleshopping (unwahrscheinlich)
  • RBB: Abendschau bzw. Brandenburg aktuell sind natürlich Kandidaten
  • Dritte Programme: Größtenteils Lokalnachrichten, natürlich möglich, dass da ein Beitrag zur bundesdeutschen Politik gesendet wurde (mit Einspieler aus Berlin)...
Vieles fällt also weg, weils fiktional ist oder thematisch nicht passt. Neben einem regionalen Nachrichtenprogramm (vor allem Abendschau) bleiben eigentlich nur 3sat mit Exclusiv – Die Reportage und RTL mit Explosiv – Weekend. Aufgrund der Zuschauerverteilung tipp ich also mal auf Explosiv ;). --APPER\☺☹ 00:08, 23. Jun. 2013 (CEST)
Ergänzung: In der Abendschau gabs nen Beitrag zum Smiley-System der Restaurantbewertungen in Berlin, ungefährt 19:45 - vllt. passt das, den Beitrag gibts unter [16]. --APPER\☺☹ 00:19, 23. Jun. 2013 (CEST)
Du hast noch ein paar Sender vergessen:
  • BR: Expeditionen ins Tierreich/Kunst & Krempel
  • BR-alpha: Karl Valentin
  • einsfestival: Clipster
  • einsplus: Expeditionen ins Tierreich
  • HR: hessenschau
  • Kika: Checker Can
  • MDR: MDR aktuell
  • NDR: Regionalprogramm
  • ORF2E: Zeit im Bild
  • Phoenix: Mein Freund, das Monster - Haie zum Anfassen
  • RB: Regionalprogramm
  • SR: daten der woche/aktueller bericht am Samstag
  • SRF 1: Tagesschau
  • SRF zwei: Turnen: Eidgenössisches Turnfest
  • SWR BW: Landesschau - KulTour/SWR Landesschau aktuell Baden-Württemberg mit Sport
  • SWR RP: Das Quiz mit Jens Hübschen/SWR Landesschau aktuell Rheinland-Pfalz
  • tagesschau24: Weltreisen
  • WDR: Lokalzeit
  • ZDF.kultur: Delikatessen: Clips
  • ZDF_neo: Terra X: Deutschland - Wie wir leben
  • ZDFinfo: History XXL: Unternehmen Barbarossa - Der Krieg gegen die Sowjetunion
--Rôtkæppchen68 02:13, 23. Jun. 2013 (CEST)

Geometrie: Winkel einer Linie auf eine Ebene

Ok, die meisten werden jetzt lachen, aber ich bin schon einige Zeit aus der Schule raus und komme nicht weiter. Ich habe eine Ebene von der ich den Winkel von Norden aus habe und den Neigungswinkel. Und ich habe eine Linie, von der ich effektiv die gleichen Werte habe. Wie rechne ich jetzt den Unterschied zwischen diesen beiden Winkeln aus. Die Abweichungsrichtung ist mir egal.

Sorry euch mit so ner Kleinigkeit zu belästigen. (und ja Google hat mir nicht helfen können) 212.90.151.90 20:17, 22. Jun. 2013 (CEST)

Die Formel kenne ich nicht, die Herleitung scheint mir nicht trivial zu sein. Mein Ansatz wäre: Gerade als dreidimensionalen Vektor darstellen; eine Koordinate ist 1, die beiden anderen sind jeweils Produkte von trigonometrischen Funktionen (sin, cos) der beiden Winkel, die die Richtung der Geraden beschreiben. Der Winkel zwischen der Geraden und der Ebene sollte der gleiche sein wie zwischen der Geraden und ihrer Projektion auf die Ebene. Also als zweites den Vektor der Geraden bestimmen, die die Projektion der Geraden auf die Ebene ist. Dann mit der Winkelformel arccos(Skalarprodukt der Vektoren geteilt durch Produkt der Beträge) den Winkel berechnen. --BlackEyedLion (Diskussion) 20:56, 22. Jun. 2013 (CEST)
Ich denke dir kann hier niemand helfen, weil nicht klar ist, was du meinst. Winkel von Norden aus? Hä? Neigungswinkel von was zu was? Linie „effektiv die gleichen Werte habe“ Noch größeres hä. Bzw. HÄÄÄ? Zeichne mal dein Problem und lad das Bild irgendwo hoch und verlinke es hier--Svebert (Diskussion) 23:27, 22. Jun. 2013 (CEST)
Winkel zwischen einer Geraden und einer Ebene: arcsin(Skalarprodukt von Geradenvektor und Ebenennormalenvektor geteilt durch Produkt der Beträge) [17]. Den Normalenvektor der Ebene kann man erhalten, indem man zwei linear unabhängige (das heißt nichtparallele) Vektoren, die in der Ebene liegen, miteinander vektormultipliziert. --BlackEyedLion (Diskussion) 23:47, 22. Jun. 2013 (CEST)
danke! 212.90.151.90 15:09, 23. Jun. 2013 (CEST)

Unfall mit Stickstoff bei einer Pool Party

Bei einer Poolparty hat es durch Dummheit Verletzte gegeben: [18]. Meine Frage dazu: Was für eine chemische Reaktion soll Chlor mit Stickstoff eingehen und warum sollte das nicht mit dem Stickstoff in der Luft passieren? Ich vermute eher, dass Sauerstoffmangel die Ursache war. Ich möchte jedoch lieber mal hier nachfragen, da das Gleiche auch von englisch sprachigen Medien behauptet wird. --Trigonomie - 07:26, 23. Jun. 2013 (CEST)

du hättest doch leiber gleich auf den Link von Daily Mail klicken sollen, dort steht:
Officials say that when the liquid nitrogen came in contact with the water, the chemical substance with a low boiling point displaced oxygen, leaving young party goers gasping for breath, Excelsior.com reported.
Daher sollte klar sein, was passiert ist: durch den Kontakt mit der Wasseroberfläche íst der flüssige Stickstoff sehr schnell verdampft und hat den Sauerstoff verdrängt. Das Wasser hatte zwar sicher in etwas die gleiche Temperatur wie die umgebende Luft (z.B. 10°C mehr oder weniegr machen bei Flüssigstickstoff mit -196°C oder weniger kaum einen Unterschied), aber die Wärmeübertragung zwischen Flüssigkeit und Flüssigkeit erfolgt wegen der höheren Dichet viel schneller als zwischen Gas und Flüssigkeit. Daher ist der Stickstoff sobald er mit dem Wasser in Berührung gekommen ist so schnell verdampft, dass er sich nicht schnell genug mit der umgebenden Luft vermischt hat und in der Nähe der wasseroberfläche die Sauerstoffkonzentration zu niedrig für Menschen war. Das mit der chemischen Reaktion mit Chlor zwar auch bei Daily Mail und zwar beim Untertitel zum zweiten Bild von unten, aber das ist wahrscheinlich nur die eigene Interpretation des Journalisten. Daily Mail hat die Nachricht übrigens von hier, eventuell kann sich diese Quelle auch noch wer anschauen, der Spanisch kann. Auf den google Übersetzer ist ja nicht wirklich Verlass. --MrBurns (Diskussion) 07:57, 23. Jun. 2013 (CEST)
Ahh, klasse. Vielen Dank für deine Erklärung. Dass der Sauerstoff verdrängt wurde, hatte ich ja auch schon geahnt. Jedoch habe ich den Zusammenhang mit der Art der Wärmeübertragung nicht bedacht. --Trigonomie - 08:10, 23. Jun. 2013 (CEST)
(BK) Nachtrag: in diesem Chemikerblog steht auch, dass es keine chemische Reaktion war, sondern der Stickstoff einfach nur den Sauerstoff verdrängt hat. --MrBurns (Diskussion) 08:13, 23. Jun. 2013 (CEST)
Im spanischen Beitrag steht: La mezcla entre el químico con el cloro del agua provocó una peligrosa reacción entre los jóvenes inmersos en la piscina, ya que la mayoría se encontraban en estado de ebriedad. Das heißt in etwa: "Die Vermischung der Chemikalie mit dem Chlor im Wasser verursachte eine gefährliche Reaktion bei den Jugendlichen, die in das Schwimmbad eingetaucht waren, da ja die meisten betrunken waren." (Unterstreichung von mir.) Von einer chemischen Reaktion ist da also, genau genommen, gar keine Rede, auch wenn unklar bleibt, warum das Chlor überhaupt erwähnt wird. In den Kommentaren zu dem Beitrag wird die Frage nach der möglichen chemischen Reaktion ebenfalls aufgeworfen und auch dort neigt man dazu, einfach von einer Verdrängung des Sauerstoffs durch Stickstoff auszugehen. Geoz (Diskussion) 08:27, 23. Jun. 2013 (CEST)

20_391_20

Eine 5-stellige Zahl endet mit den Ziffern 20. Wenn man diese beiden Ziffern rechts wegstreicht und links anfügt, wird die Zahl um 18729 kleiner. abc20-18729=20abc. Durch Rumprobieren (38c20-17829=20c91, dann für c durchprobiert) bin ich draufgekommen, wie sieht der Rechenweg aus? -- 2001:4C28:194:520:5E26:AFF:FEFE:8B50 12:36, 23. Jun. 2013 (CEST)

. --Rôtkæppchen68 12:55, 23. Jun. 2013 (CEST)
Das kannst Du umformen zu mit . Nach n aufgelöst ergibt . Das ergibt dann 391191999< n391202099. In diesem Intervall ist nur eine einzige natürliche Zahl, die auf …20 endet. Die muss es sein. --Rôtkæppchen68 13:07, 23. Jun. 2013 (CEST)
Es geht noch einfacher. Wir wissen, dass wir am Ende der Zahl die Stellen ,20 streichen. Damit ergibt sich . --Rôtkæppchen68 14:01, 23. Jun. 2013 (CEST)

Kurze (wahrscheinlich doofe) Seriendokument-Frage

Hallo, was muss ich tun, damit Fußnoten in einem Word-Serienbrief auf jeder Seite dieselbe Nummer haben und nicht ad infinitum durchgezählt werden? Gruß --Dumbox (Diskussion) 10:55, 23. Jun. 2013 (CEST)

' verwenden? :-) *scnr* --Heimschützenzentrum (?) 11:16, 23. Jun. 2013 (CEST)
Ja, eines der Projekte, die in der Ablage "Wenn ich mal massig Zeit habe" schlummern. ;) Jetzt aber nochmal ernsthaft... Dumbox (Diskussion) 11:26, 23. Jun. 2013 (CEST)
Ich wüsste da nichts, außer dass man die Nummerierung manuell erstellt anstatt eine Nummerierungsfunktion zu verwenden. Für zukünftige Antworten könnte es eventuell sinnvoll sein, zu erwähnen, um welche Version von Word es sich handelt, falls es da Unterschiede gibt (bei ganz alten Wordversionen gibts fast sicher Unterschiede, aber ich nehme schon an, dass es sich um eine "moderne" Word-Version handelt, also Word 97 oder neuer). --MrBurns (Diskussion) 11:34, 23. Jun. 2013 (CEST) PS: oder für jede Seite ein eigenes Dokument erstellen un die Fußnoten udn alle anderen identsichen Inhalte per Copy + Paste kopieren. --MrBurns (Diskussion) 11:41, 23. Jun. 2013 (CEST)
+1 Hast du im Fussnoten-Menu nachgesehen? Da, wo man die unterschiedlichen Arten einstellen kann? GEEZER... nil nisi bene 12:01, 23. Jun. 2013 (CEST)
Ah, jetzt, ja! Im Enddokument, nicht in der Vorlage, Fußnoten "jede Seite neu beginnen". *rant Was soll das? Es liegt doch im Wesen eines Serienbriefes, dass jede Seite genauso aussehen soll. *rant aus. Vielen Dank für die Hilfe! Dumbox (Diskussion) 12:12, 23. Jun. 2013 (CEST)
Archivierung dieses Abschnittes wurde gewünscht von: Dumbox (Diskussion) 12:55, 24. Jun. 2013 (CEST)

Interwikiverlinkung: Wikidata

Warum kann ich die en:Category:Relics associated with Jesus nicht mit der Kategorie:Christusreliquie auf Wikidata verlinken ? 178.3.22.4 18:14, 23. Jun. 2013 (CEST)

Du musst zuerst die Wikidata-Verknüpfung von de:Kategorie:Christusreliquie löschen, indem Du auf die Details der Fehlermeldung klickst und auf der „anderen“ Wikidataseite die Kategorie:Christusreliquie löschst. Dann auf der ursprünglichen Wikidataseite hinzufügen. --Rôtkæppchen68 18:18, 23. Jun. 2013 (CEST)
Leere Items können dann gelöscht werden (-> [19]) --тнояsтеn 13:22, 24. Jun. 2013 (CEST)
Danke, Das wusste ich noch nicht. --Rôtkæppchen68 23:43, 24. Jun. 2013 (CEST)
Archivierung dieses Abschnittes wurde gewünscht von: тнояsтеn 13:22, 24. Jun. 2013 (CEST)

buzzer baustelle

Bei manchen Baustellen mit Kränen hört man Schallzeichen von einem Buzzer oder so ähnlich, meistens 3 x. Was bedeuten diese Zeichen? --80.226.24.9 (19:00, 20. Jun. 2013 (CEST), Datum/Uhrzeit nachträglich eingefügt, siehe Hilfe:Signatur)

Der Kranführer hat eine Hupe womit er die Aufmerksamkeit der anderen Arbeiter erlangen kann. Zufällig könnte dein Kranführer diese stets 3x betätigen. --Netpilots -Φ- 19:34, 20. Jun. 2013 (CEST)
Nein, kein Zufall. Oft hat die Kran-/Baufirma da eine eigene Regel. Zum Beispiel einmal=Achtung, Kran bewegt sich; zweimal=Funk war gestört, Anweisung wiederholen; Dauerhupe=Gefahr, alle auf den Kran achten und weg von der Last. -- Janka (Diskussion) 21:04, 20. Jun. 2013 (CEST)
Man lernt nie aus. Dann müssten alle auf der Baustelle diese Regeln kennen, auch die Elektriker. Als ich vor Jahrzehnten auf Baustellen arbeitete hat niemand davon was gesagt, gehupt hat es trotzdem. --Netpilots -Φ- 17:00, 23. Jun. 2013 (CEST)

Finanzierung

Eine etwas unkonventionelle Frage, zugegeben, dennoch hoffe ich auf rege Unterstützung und kreatives Mitdenken - ich wäre euch sehr dankbar. Ich möchte mir als Student eine Eigentumswohnung zulegen, Problem ist die Finanzierung. Die Hälfte des Kaufpreises könnten meine (nicht mehr berufstätigen) Eltern beisteuern, den Rest, immerhin schlappe 100.000 €, müsste ich mir anderswo herholen. Wie kann man das als Student am besten anstellen? Kann ich mir einen Kredit besorgen? Andere Möglichkeiten? Für jenen Impuls wäre ich hoch dankbar! Und, eine große Bitte noch zum Schluss: keine Veralberungen, Aufführungen illegaler Tätigkeiten etc. bitte ;-) --85.178.58.96 20:52, 20. Jun. 2013 (CEST)

Eventuell über einen Hypotheken-Kredit, falls die Bank keine zusätzlichen Sicherheiten verlangt (daher nicht aus der Subprime-Krise gelernt hat). --MrBurns (Diskussion) 20:57, 20. Jun. 2013 (CEST)
Hypothekenkredit ist völlig richtig. Dazu möchte die Bank sich überzeugen, dass die Wohnung wirklich so viel wert ist. Vor allem aber wird sie dir vorrechnen, wie hoch deine monatliche Belastung ist, je nach Laufzeit. Dann möchte sie dein monatliches Einkommen erfahren. Dann zieht sie die monatliche Belastung vom Einkommen ab. Wenn die Bank der Ansicht ist, dass die Differenz dir zum Leben reicht, dann gibt sie dir vielleicht den Kredit. Sonst nicht. Bei vielen Studenten gilt "sonst", also "nicht". Aber wir kennen ja dein Einkommen nicht. Chantk (Diskussion) 21:06, 20. Jun. 2013 (CEST)
50% Barfinanzierung ist eine Menge. Vermutlich wirst du deinen Hypothekenkredit damit schon von irgendeiner Bank bekommen, wenn der Wert der Wohnung korrekt eingeschätzt ist. Die Bank kann dann ja nur gewinnen: entweder sie bekommt pünktlich dein Geld, oder sie bekommt die Immobilie. Überleg dir also gut, was du sicher zahlen kannst. 85.180.192.70 21:15, 20. Jun. 2013 (CEST)
Such Dir doch einen solventen Bürgen, der von Deiner späteren Karriere (gutes Einkommen und damit sichere Rückzahlung) überzeugt ist. Nebenjobs im Studium (abgesehen von den Semesterferien) kannst Du vergessen, das hat meinen Noten den Rest gegeben (über Jahre hinweg todmüde aufpassen und lernen geht nicht). Niere verkaufen bringt wenig. Blut- oder Samenspende bringt keinen nennenswerten Betrag, selbst über Jahrzehnte hinweg. Pornodarsteller: Als Mann wird man da nicht reich von. Geschäftsbeteiligungen, Grundstücks- oder Wertpapierspekulationen wären eine Möglichkeit, um legal schnell and viel Geld zu kommen; Voraussetzung ist aber exzellente Fach- und Branchenkenntnisse sowie Startkapital (das ganze ist wiederum immer mit Glück verbunden). Eventuell könntest. Wenn Dir das Glück obhold ist: Glückspiel, nee lass lieber. Dann noch: eine Prinzessin heiraten, die "gut betucht" ist. Schatzsucher wäre auch noch eine Möglichkeit, die gibt's wirklich noch. Genauso wie Goldsucher. Aber da braucht man auch erstmal Kapital und Zeitaufwand. Dann könntest Du etwas bahnbrechendes erfinden und gleich patentieren lassen. Oder Du schreibst einen Bestsellerroman oder wirst Entertainer oder Musiker im Unterhaltungsbereich :-) --80.226.24.12 21:09, 20. Jun. 2013 (CEST)
Du könntest die Anfrage auch bis zum letzten Satz lesen. --Aalfons (Diskussion) 21:33, 20. Jun. 2013 (CEST)
50% Eigenmittel ist gar nicht schlecht. Selbst ohne Einkommen könnte man da einen Hypothekarkredit bekommen, sogar einen endfälligen (d.h. ohne monatliche Zahlungen, mit Tilgung alles zusammen erst am Ende der Laufzeit). Die Bank hat dabei praktisch kein Risiko, wenn die Hypothek im Grundbuch eingetragen ist. Dann können sie dir im Fall von Zahlungsunfähigkeit die Wohnung unterm Hintern wegpfänden. Sollte die Bank jedoch mehr als 2,5% Zinsen wollen, dann wollen sie dich übers Ohr hauen. Außerdem wird keine Bank auf einen fixen Zinssatz einsteigen. Derzeit sind Kredite extrem günstig, aber was tust du wenn der EURIBOR-Zinssatz steigt? Nach meinem Eindruck, rechnen die Banken mittelfristig (also max. in fünf Jahren) mit Zinssätzen, die deutlich über den jetzigen liegen. Und dann kann so ein günstiger Kredit schnell mal ziemlich teuer werden, wenn im Vertrag ein flexibler Zinsfuss (eben auf EURIBOR-Basis) vereinbart wurde. Man kann Absicherungsgeschäfte machen, aber die kosten auch und zwar gar nicht wenig. --El bes (Diskussion) 21:53, 20. Jun. 2013 (CEST)
Mit den Prinzessinnen ist es so eine Sache - da scheint der Markt gerade ziemlich leergefegt. Eugenie fällt mir ein - wer steht sonst noch auf der Liste lediger Prinzessinnen über 16? Chantk (Diskussion) 22:10, 20. Jun. 2013 (CEST)
Durch die anstehende Gleichstellung der Lebenspartnerschaft mit der Ehe tut es wahrscheinlich ebenso unsere Liste verpartnerungswilliger Prinzen. Mist, auch rot. Also doch arbeiten. --Rôtkæppchen68 00:29, 21. Jun. 2013 (CEST)
Ich finde, wenn man nicht weiß, wie man an Geld kommt ist es auch besser, wenn man nicht allzuviel in die Finger bekommt. Der Fragesteller mag ja noch nicht einmal sein Studienfach nennen (was für die Einschätzung seiner zukünftigen Solvenz und einer Erwartungsanalyse über seine künftigen Rückzahlungsmöglichkeiten von zentraler Bedeutung ist). Einem Philosophiestudenten zum Beispiel (wahlweise auch ein anderes Orchideenfach) würde ich persönlich sicher nicht mit einem Peer-to-Peer-Kredit, z.B. über Auxmoney unterstützen. Und man sollte von dem zukünftigen Kreditnehmer schon erwarten, daß er mit beiden Beinen im Leben steht, sich im Vorfeld bereits überlegt hat, wie und warum er in überschaubarer Zeit 200.000 Euro plus Zinsen abbezahlen will und kann und nicht irgendwelche unseriöse Internetforen für solche Probleme befragen muß. --84.191.135.38 01:13, 21. Jun. 2013 (CEST)
@Rôtkæppchen68: Der/die Fragestellende könnte es auch mit der Liste adoptionswilliger Multimillionäre versuchen... Mist, auch rot. Marcus (MKir_13) (Diskussion) 09:26, 21. Jun. 2013 (CEST)
Liste vergrabener Schätze und wo man sie findet? Och Mensch, hier gibt´s aber auch nix..--Optimum (Diskussion) 12:20, 21. Jun. 2013 (CEST)
Klar haben wir das: List of burried treasures and where to find them. The text is an inventory of 64 locations; 63 of which are treasures of gold and silver, which have been estimated in the tons. Textprobe aus der Einleitung: In the ruin that is in the valley of Acor, under the steps, with the entrance at the East, a distance of forty cubits: a strongbox of silver and its vessels with a weight of seventeen talents. Ihr braucht euch aber nicht mehr zu bemühen. Ich war schon da ;-) Geoz (Diskussion) 12:56, 21. Jun. 2013 (CEST)

Entscheidend sind doch drei Dinge: Wie lange gedenkt der Fragesteller diese Wohnung zu nutzen? Kann der Fragesteller oder auch dessen Eltern die laufenden Zinsen und zumindest eine teilweise Tilgung finanzieren? Und wie stellt sich die Wertentwicklung in Zukunft dar.
Angesichts der bei Immobilien doch recht hohen Transferkosten lohnt das nur bei längerem Behalten.
Wie viel würde eine Wohnung in dieser Stadt zur Miete kosten? Ist dieser Betrag deutlich höher als Zinsen, Tilgung und Nebenkosten, die der Vermieter zu tragen hat? Ließen sich in dieser Eigentumswohnung während der Nutzung durch den Fragesteller Zimmer an andere Studenten untervermieten? Wie hoch wären die Aufwendungen für den Fragesteller, wenn er den auf ihn entfallenden Anteil an seiner Wohnung woanders anmieten würde?
Wie werden die Zukunftsaussichten am Standort eingeschätzt? Kann man das Objekt dauerhaft vermieten oder regelmäßig neu vermieten? Kann man es an einen Nachfolger in änhlicher Lage weiterverkaufen?
Ich erinner mich dunkel, das es z.B. in Freiburg im Breisgau in den 1980 spezielle Förderprogramme für Studentenwohnungen gab, die von ganz ähnlichen Fallkonstellationen ausgingen (Ein Freund hat dort studiert und mich bei einem Besuch auf entsprechende Baustellenschilder hingewiesen). --Eingangskontrolle (Diskussion) 17:09, 21. Jun. 2013 (CEST)

Zwar sind 50 % Eigenmittel erfreulich hoch. Jedoch dürfte keine Bank einen Kredit gewähren, wenn der Kapitaldienst dafür (Zinsen + Tilgung) über dem Budget eines Studenten liegen; da hilft auch ein Bürge nicht weiter (die Bank würde sich dem Bürgen gegenüber sittenwidrig verhalten, wenn sie eh weiß, daß der Kreditnehmer sowieso nicht bezahlen kann). Ob der Kapitaldienst über Studentenbudget liegt, hängt (auch) vom Tilgungsplan des Darlehens ab. Und deswegen geht es wesentlich um die Frage: soll denn die Wohnung selbst genutzt oder (zunächst einmal) vermietet werden? Das Darlehen zu erhalten wird leicht(er) sein, wenn die Wohnung vermietet wird, denn die Netto-Mieteinnahmen einer 200.000-€-ETW dürften höher sein als die Wohnkosten eines typischen Studenten. Wenn Du also die Wohnung für - beispielsweise - 1.000 € (kalt) vermietest, dürften Dir nach Abzug Hausgeld etc. vielleicht noch um die 500 € bis 600 € Ertrag verbleiben. Wenn Du selbst als Student für vielleicht 300 € irgendwo wohnst, kannst Du den Kapitaldienst erbringen. Wenn Du aber Deine 300-€-Bude aufgibst, in die ETW ziehst, dann hast Du keine Mieteinnahmen, mußt dennoch Hausgeld etc. bezahlen - also bei einer Wohnung dieser Preisklasse wohl sogar unterm Strich mehr als eine Studentenbudemiete ausmacht. Dann ist Dein Budget mit ETW schmaler - noch vor dem Kapitaldienst. Ich denke also: Knackpunkt für eine sinnvolle Darstellung der Finanzierung ist die Frage, ob Du selbst einziehst. Das wird -für einen Studenten - ziemlich teuer. --Freud DISK Konservativ 08:54, 22. Jun. 2013 (CEST)

1000€ Kaltmiete (ich nehme an das ist zuzüglich Betriebskosten) kommt mir etwas hoch vor für eine Wohnung, die 200.000€ in der Anschaffung kostet. m.E. bekommt man nur selten eine Miete, die bezogen auf die Anschaffungskosten einem jährlichen Mietzinserlös von 6% des Kaufpreises entspricht. --MrBurns (Diskussion) 09:26, 23. Jun. 2013 (CEST)
Das kommt ganz darauf an, wo. Wenn man mal Immobilienanzeigen durchgeht, lassen sich durchaus noch Mietshaeuser finden, in denen die Mietrendite 8-10% im Jahr betraegt (nicht gerade in den Boomstaedten wie Muenchen, aber andernorts doch schon noch recht regelmaessig). Bei Einzelobjekten/kleinen Wohnungen ist das auch oft der Fall, da Einzelwohnungen fuer groessere Investoren eher wenig interessant sind und der Verwaltungsaufwand im Verhaeltnis zur Groesse des Objekts doch recht gross ist. -- Arcimboldo (Diskussion) 03:20, 24. Jun. 2013 (CEST)
Ich dachte immer, der Verwaltungsaufwand (zumindest die Hausverwaltung) wird ohnehin über die Betriebskosten auf die Mieter abgewälzt. Da bleibt dann wohl nur der firmeninterne Verwaltungsaufwand für die Verrechnung der Einnahmen/Ausgaben übrig. Ansonsten sind einzelwohnungen vielleicht für große Firmen weniger attraktiv, aber dafür attrraktiver für Privatpersonen, die unabhängig von firmen in Immobilien investieren wollen. Ich schätze mal, gerade dieser Personenkreis ist nach der Subprime-Krise größer geworden, weil da ja auch viele dieser Immobilien-Anlagefirmen nbankrott gegangen sind oder wie die österreichische Immofinanz stark an Wert verloren haben. --MrBurns (Diskussion) 11:57, 24. Jun. 2013 (CEST)

Englische Datumsangabe

Wenn ich ein englisches Datum in der Form 1st June, 2013 angebe, lautet es analog für das heutige Datum nun 21st June, 2013 oder 21th June, 2013? Was ist mit dem 11.6., 11th June, 2013? morty (N) 10:28, 21. Jun. 2013 (CEST)

21st June, da gesprochen: twenty-first June; aber: 11th June, da: eleventh June.PοωερZDiskussion 10:35, 21. Jun. 2013 (CEST)
Im Schottischen gäbe as ja noch das Wort "tenty". Nach etwa tenty-one Double Malts könnte man also die Ordnungszahl "tenty first" oder auch "tenty oneth" kreieren. --Cookatoo.ergo.ZooM (Diskussion) 13:38, 21. Jun. 2013 (CEST)
PS: Scotch "tenty" heißt in etwa "aufmerksam".
21 June 2013, international 2013-06-21. "st" und "th" wurde in der Schule zwar beigebracht (und wird auch noch so gesprochen), im Schriftverkehr aber nicht mehr üblich. Siehe auch en:Date and time representation by country m.w.N.. --194.25.103.254 13:54, 21. Jun. 2013 (CEST)
Bedenke auch 1st, (von first = der Erste), 2nd (second) und 3rd, (third), alles weiter ist th von fourth, fifth, bezogen auf den Tag des Monats. Dehlt inden artikeln Datum und Datumsformat --Hans Haase (Diskussion) 14:16, 21. Jun. 2013 (CEST)
Ja, danke. Hatte ich mir schon fast gedacht. Aber ich dachte frag mal besser, denn sprachliche Eigenheiten sind ja nicht immer nach Schema F abzuhandeln. morty (N) 17:46, 21. Jun. 2013 (CEST)
Die Schreibweise mit -st, -nd, -rd, -th scheint veraltet zu sein. Je nach Ländereinstellung schlägt Windows z.B. 21 June 2013 für Großbritannien oder June 21, 2013 für USA vor. In anderen englischsprachigen Ländern gibt es teilweise andere Konventionen. Du kannst sie Dir in den Windows-Ländereinstellungen in der Systemsteuerung anschauen. --Rôtkæppchen68 17:53, 21. Jun. 2013 (CEST)
Falls die Zielgruppe ein internationales Publikum ist, ist das Datumsformat nach ISO 8601 einem länderspezifischen Format vorzuziehen. --Komischn (Diskussion) 08:37, 24. Jun. 2013 (CEST)

Dr. Hittich

Was ist Dr. Hittich´s Asta Maga 3? Soll man das kaufen? --2.171.111.236 15:45, 22. Jun. 2013 (CEST)

einen Dr. Hittich findet Google zum Beispiel hier: http://www.abzocknews.de/2011/08/08/amtlich-wirkender-bestelltrick-des-dr-hittich/ .--Macuser10 (Diskussion) 15:49, 22. Jun. 2013 (CEST)
(BK) Wie bist du denn darauf gestoßen? Und wenn Du nicht weiß, was es ist, wieso fragst du dann ob "man" es kaufen soll? --Zulu55 (Diskussion) Unwissen 15:49, 22. Jun. 2013 (CEST)
Vielleicht hilft diese Seite Nahrungsergänzungsmittel#Empfehlungen --Dos Vientos (Diskussion) 15:51, 22. Jun. 2013 (CEST)
[20]... diese ganzen Erfahrungsberichte ohne vernünftige Studien wirken verdächtig... --Heimschützenzentrum (?) 19:25, 22. Jun. 2013 (CEST)
Wozu muss man Studien zu Nahrungsmitteln machen? Bei Nahrungsmitteln bedarf es nur der richtigen Werbung. Die Bezeichnung dieses Mittels schein schon einmal voll einzuschlagen, obwohl es nur Krillöl ist. Dazu brauch ich keine Studien. --Rôtkæppchen68 01:49, 23. Jun. 2013 (CEST)
wegen der ganzen health claims, die in den Erfahrungsberichten stehen vllt? selbst zu Spinat gibts Studien, auch wenn die teilweise irreführend publiziert wurden... --Heimschützenzentrum (?) 23:09, 23. Jun. 2013 (CEST)
Zu den ganzen Healthclaims von Krillöl hat Romdie Europäische Behörde für Lebensmittelsicherheit gesprochen und ist zu dem Schluss gekommen, dass keine Ursache-Wirkungs-Beziehung vorliegt. Der Verzehr von Krillöl hat keine Auswirkung auf die Blutkonzentration von Triglyceriden.[21] Siehe auch letzter absatz des Artikels Krillöl. --Rôtkæppchen68 00:07, 24. Jun. 2013 (CEST)
siehste! beim Doktore steht es schonmal unter Herzschmerz-Kreislauf... :-) und wenn man sich kostenlos beraten lässt, kommt wahrscheinlich noch mehr ungestütztes Zeug... --Heimschützenzentrum (?) 07:50, 24. Jun. 2013 (CEST)
Quacksalber. —PοωερZDiskussion 19:43, 23. Jun. 2013 (CEST)

Anfrage zu Google Translator

Seit geraumer Zeit weigert sich Google Translator, Websites zu übersetzen. Kennt jemand den Grund? LG;--Nephiliskos (Diskussion) 16:50, 22. Jun. 2013 (CEST)

Fehlbedienung. [22] --Eike (Diskussion) 17:16, 22. Jun. 2013 (CEST)
Blöd Fraach: Wie komm ich dorthin? Ich frage, weil es bis kürzlich noch gereicht hat, eine bestimmte Webadresse einfach in das Übersetzungsfeld zu kopieren, rechts auf das (übersetzte) Gegenstück zu klicken und fertig. Das geht plötzlich nimmer. LG;--Nephiliskos (Diskussion) 17:27, 22. Jun. 2013 (CEST)
Blöde Antwort: Wenn Du rechts ein übersetztes Gegenstück hast, dann erübrigt sich das Klicken. Wenn nicht, dann klicke auf den Button "Translate". Service, damit der die Site nicht heißläuft: "Translate" heißt auf Deutsch "übersetzen"..... --Pyrometer (Diskussion) 17:33, 22. Jun. 2013 (CEST)
Ah, verdammt, da bin ich nicht drauf gekommen! X) --Eike (Diskussion) 17:34, 22. Jun. 2013 (CEST)
(BK) Bei mir blitzt hier, wenn ich eine URL eingebe, unten in der Mitte ab und zu unten ein gelbes Feld auf, in das man den Link ziehen soll - leider lädt dann mein Browser einfach die Seite. Ich hab's jetzt hierrüber gemacht: http://itools.com/tool/google-translate-web-page-translator. Aber bei meinem Link oben kannst du auch einfach eine neue Website eintragen. --Eike (Diskussion) 17:34, 22. Jun. 2013 (CEST)
Danke Eike, besser als das dumme Geschrabbel von Deinem Vorredner. Jetzt plötzlich geht es wieder: die kopierte Web-URL in der Box wird rechts in übersetzter Form geöffnet. Hmmmm... Komisch... LG;--Nephiliskos (Diskussion) 17:37, 22. Jun. 2013 (CEST)
Es mag ein wenig geschrabbelt sein, aber er hat total Recht: Webadresse ins linke Fenster (wie sonst den Text), wenn nicht automatisch übersetzt wird "Translate" anklicken, glücklich sein! --Eike (Diskussion) 18:03, 22. Jun. 2013 (CEST)
Eike, für Drag'n'drop musst Du nicht das gelbe Feld treffen, oder gar zu warten, bis es erscheint. Die ganze Translate-Seite ist ständig sensitig für gedropte URL. Einfach die URL in einem anderen Tab oder Fenster greifen (vorne am Seitensymbol kann man sie greifen) und in das andere Fenster/Tab mit Google-Translate ziehen. --Pyrometer (Diskussion) 17:48, 22. Jun. 2013 (CEST)
Das scheint bei mir (Linux) ein wenig daraauf anzukommen, was ich da drage und droppe. Wenn ich eine URL markiere und d&de, wird das vom Browser (Seamonkey) abgefangen und einfach die Seite geladen. Wenn ich einen Bookmark d&de, wird der ärgerlicherweise mit der textuellen Beschreibung übergeben und es geht auch nicht. --Eike (Diskussion) 12:34, 23. Jun. 2013 (CEST)
Google ergreift bei "exessiver Nutzung" neuerdings Maßnahmen wie captcha-Passwortabfragen und Serviceverweigerung. Im Prinzip eine Konsequenz aus der Nutzerprofilbildung aus ihrem Datamining. Witzig ist dabei das man als angemeldeter Nutzer mit Google account schlechter gestellt wird als der anonyme Nutzer der bei blockierung der Google HTTP-Cookies dieses Datamining per ip-wechsel jederzeit "resetten" kann. --Kharon 19:09, 23. Jun. 2013 (CEST)

Entfernen von Zeilenumbrüchen und Trennungszeichen

Hallo! Ich habe ein sehr großes Textcorpus in drei TXT-Dateien, bei dem ich gern die Zeilenumbrüche und die Trennungszeichen entfernen würde, um einen glatteren Text zu erhalten. Bei den Zeilenumbrüchen müsste außerdem ein Leerzeichen ergänzt werden. Ist das auch ohne Programmierkenntnisse möglich?

Viele Grüße, --[ˈjonatan] (ad fontes) 17:57, 22. Jun. 2013 (CEST)

Mit dem guten alten Microsoft Word und der Funktion "Suchen und Ersetzen" (Strg+F): Suchen nach "^p", ersetzen durch " " (jeweils ohne Anführungszeichen. --тнояsтеn 18:06, 22. Jun. 2013 (CEST)
Das ersetzt allerdings nicht die Trennungszeichen. dafür müsste man z. B. erstmal "-^p" durch "" ersetzen, was aber auch Fehler produziert ("Schiller-Theater" würde zu "SchillerTheater"). Selbst in Word müsste man also etwas komplizierter denken und die dortigen pseudo-regulären Ausdrücke verwenden. Aber wie und mit welchem Tool man so etwas mit reg. Ausdrücken ersetzt, das können andere sicher besser und korrekter erklären als ich.--FA2010 (Diskussion) 18:33, 22. Jun. 2013 (CEST)
Vielen Dank für diese Hinweise! Damit kann ich arbeiten. [ˈjonatan] (ad fontes) 21:33, 22. Jun. 2013 (CEST)
Das Trennzeichen vs. Bindestrich-Problem lässt sich lösen, wenn (in Word 2010) die Option "Platzhalter verwenden" aktiviert, als Suchstring ([a-z])(-^13)([a-z]) und als "Ersetzen" \1\3 verwendet wird. Dann wird Schiller-[Abs]Theater ignoriert, Schiller-[Abs]tage werden zu Schillertage. --Rudolph Buch (Diskussion) 01:00, 23. Jun. 2013 (CEST)
Es ist sogar noch einfacher, weil ich meinem Text nicht das hier das Trennungszeichen ist (-), sondern ein anderer Strich. [ˈjonatan] (ad fontes) 23:52, 23. Jun. 2013 (CEST)

Aceto Balsamico Tradizionale

Wo kann man als Endkunde den echten Aceto Balsamico Tradizionale am günstigsten kaufen? Direkt beim Hersteller vor Ort oder über einen Online-Shop? --84.74.139.84 19:39, 22. Jun. 2013 (CEST)

Faszinierend! Was es alles gibt!
Also
(1) du suchst dir einen bestimmten Hersteller heraus.
(2) Du suchst dir nun einen bestimmten Typ heraus (wenn man so ein Produkt kauft, hat man sicherlich schon etwas darüber gelesen.)
(3) Du gibst die Suche => euro "Aceto Balsamico Tradizionale" HERSTELLERNAME PRODUCTTYP <= in eine dir vertraute Suchmaschine ein.
(4) Du vergleichst die Preise.
(1) Der Hersteller ist mir ehrlich gesagt egal, auch ob er aus Modena oder der Reggio Emilia ist. Wichtig ist mir, dass die Qualität stimmt.
(2) Der günstigste dürfte wohl ein Tradizionale di Reggio Emilia mit rotem Etikett sein.
(3) Been there, done that. Du behauptest also, vor Ort ginge es nicht günstiger als Online?
(4) Weiss dann aber immer noch nicht, ob der Minimalpreis mit dabei ist.
Auf diese Weise findet man Preisunterschiede bis zu 30 % Bestellst du mehrere Liter, solltest du dich direkt an die Hersteller wenden. ;-)
Und wieviel Rabatt erhalten die? Wenigstens ungefähr, sagen wir so ab Abnahme von 10 Litern?
Wozu verwendet man den? GEEZER... nil nisi bene 12:40, 23. Jun. 2013 (CEST)
Aceto_balsamico#Verwendung --84.74.139.84 13:45, 23. Jun. 2013 (CEST)
Ah ... Mode ..! :-) GEEZER... nil nisi bene 07:43, 24. Jun. 2013 (CEST)

Nachwahlen

Wenn es bei der Bundestagswahl zu einer Nachwahl in einem Wahlkreis kommt (wie 2005), sind dannn auch die Personen, die zwischen dem eigentlichen Wahltermin und den Nachwahltermin 18 werden, wahlberechtigt?

--SFfmL (Diskussion) 15:25, 23. Jun. 2013 (CEST)

Siehe §44 Bundeswahlgesetz. --Rôtkæppchen68 15:35, 23. Jun. 2013 (CEST)
§44 regelt die Wiederholungswahl, die Nachwahl wird in §43 behandelt. -- Wiprecht (Diskussion) 12:52, 24. Jun. 2013 (CEST)
Das Bundeswahlgesetz sagt: Nein, sind sie nicht.
Wahlrecht: §12 (1) Wahlberechtigt sind alle Deutschen im Sinne des Artikels 116 Abs. 1 des Grundgesetzes, die am Wahltage 1. das achtzehnte Lebensjahr vollendet haben, ....
Wahltag: § 16 Der Bundespräsident bestimmt den Tag der Hauptwahl (Wahltag). Wahltag muß ein Sonntag oder gesetzlicher Feiertag sein.
Nachwahl: §43 (2) Die Nachwahl soll im Fall des Absatzes 1 Nr. 1 spätestens drei Wochen nach dem Tag der Hauptwahl stattfinden. .... Den Tag der Nachwahl bestimmt der Landeswahlleiter.
Daraus folgt: Das Wahlrecht mit 18 Jahren bestimmt sich nur nach dem Termin des Tages der Hauptwahl, der Nachwahltermin ändert daran nichts mehr. -- Wiprecht (Diskussion) 12:54, 24. Jun. 2013 (CEST)

Suche Familiennamen

Wo kann man erfahren ob es noch bestimmzte Familiennamen in Velatice gibt?

--93.220.230.47 16:10, 23. Jun. 2013 (CEST)

Im Telefonbuch. --Rôtkæppchen68 16:18, 23. Jun. 2013 (CEST)

Fifa 13 PS3 - individueller Kader bei online Freundschaftsspielen

Hallo, ich möchte gerne bei Fifa 13 in einem online Freundschaftsspiel gegen einen bekannten den Kader benutzen, den ich mir im Karrieremodus zusammengestellt habe. Wie kann ich das einstellen ? Leider finde ich über google keine hilfreichen Treffer. Vielen Dank! --79.215.19.167 16:29, 23. Jun. 2013 (CEST)

Suchmaschine für Mailinglisten?

Gibt es eine Suchmaschine für Mailinglisten? Also wo man z. B. Listen über bestimmte Themen aufgelistet bekommt? --188.101.85.4 18:56, 23. Jun. 2013 (CEST)

Newsserver sind sehr ähnlich, enthalten eine Themenkategorisierung und sind bei größeren Internetprovidern oft kostenlos. Geht einfach mit dem Browser. --Kharon 19:18, 23. Jun. 2013 (CEST)
Also mein Browser kann mit NNTP nichts (mehr) anfangen. --79.223.120.138 03:51, 24. Jun. 2013 (CEST)
Öffentliche Mailinglisten sollten sich per Google gut finden lassen, z. B. http://www.google.com/search?q=debian+mailing+lists --Eike (Diskussion) 20:42, 23. Jun. 2013 (CEST)

https://groups.google.com/ sollte helfen. --тнояsтеn 13:26, 24. Jun. 2013 (CEST)

Film gesucht

Aus folgendem Clip: http://www.youtube.com/watch?v=e_IkZtaJQPg --91.32.54.100 22:46, 23. Jun. 2013 (CEST) 1:55-2:00 und ab 2:20-2:24. --91.32.54.100 22:49, 23. Jun. 2013 (CEST)

Bei 1:55-2:00 (und nur dort) sieht man links oben das Logo eines – mir unbekannten – TV-Senders (eine „1“ im Fernsehkasten). --Pp.paul.4 (Diskussion) 23:22, 23. Jun. 2013 (CEST)
Abgleich mit den dort erwähnten Video-Clips:
Valkyrie
Der Untergang
Stauffenberg
Cross of Iron
Europa Europa
Die Gustloff
Der Zweite Weltkrieg in Farbe VII
Lebensborn (!)
Hitler - The rise of evil
wurde bereits durchgeführt ? GEEZER... nil nisi bene 23:25, 23. Jun. 2013 (CEST)
@Pp.paul.4, der Fernsehsender ist Česká televize 1. --Rôtkæppchen68 23:42, 23. Jun. 2013 (CEST)

Betonverbundpflaster

Wann wurde das Betonverbundpflaster erfunden? --80.129.94.27 23:22, 23. Jun. 2013 (CEST)

Vor / um 1973 Suche => Deutsches Bundespatent Verbundpflaster Beläge aus Betonverbundpflaster <= GEEZER... nil nisi bene 23:29, 23. Jun. 2013 (CEST)
1967 GEEZER... nil nisi bene 23:32, 23. Jun. 2013 (CEST)
en:Interlocking concrete pavers zufolge soll das Betonverbundpflaster Mitte der 1940er-Jahre in den Niederlanden erfunden worden sein, leider ohne Quelle. --Rôtkæppchen68 23:35, 23. Jun. 2013 (CEST)
@Geezer: Sucht man Patente mit Verbundpflasterstein, dann gibt es bereits ab 1958 Treffer. Interessant wäre, ob in diesem Buch auch schon was zu Beton steht. Oder hier zur Geschichte. --тнояsтеn 09:56, 24. Jun. 2013 (CEST)

Steuervermeidung durch Hoeneß

Wieso hat dieser Mann eigentlich nicht die vielfältigen legalen Möglichkeiten zur Steuervermeidung genutzt, die er sicherlich mit wenig Aufwand hätte realisieren können? Obwohl das dann auch moralisch verwerflich wäre, müsste er jetzt immerhin nicht mir einer Gefängnisstrafe und den daraus resultierenden Folgen rechnen. --Verwaltungsgliederung (Diskussion) 23:26, 23. Jun. 2013 (CEST)

Schreib ihm eine Mail. mMn keine wissensfrage, weil nicht wirklich beantwortbar, man kann nur darüber spekulieren (eine Möglichkeit wäre z.B.: ihm habend ie legalen Möglichkeiten zur Steuervermeidung nicht gereicht, deshalb hat er zusätzlich noch illegale Möglichkeiten genutzt). --MrBurns (Diskussion) 23:31, 23. Jun. 2013 (CEST)
Selbstaussage Hoeneß: „Das war ein Konto zum Zocken, für nichts anderes“ (Bild), also auch nicht zum Zweck der Steuerhinterziehung, -verkürzung oder -vermeidung. Kriminologie ist übrigens auch eine Wissenschaft. --BlackEyedLion (Diskussion) 23:57, 23. Jun. 2013 (CEST)
Darüber kann man nur spekulieren. Vermutlich verstrickt man sich schon mit der Idee irgendwo einen "Notgroschen" zu deponieren von dem niemand wissen soll. Das ist offensichtlich eine ziemlich verbreitete Praxis bei den Wohlhabenden von der diverse Einzelstaaten, die sogenannten Steuerparadise, regelrecht hauptgeschäftlich leben. Dann hat man das Vermögen dort und kommt irgendwann unweigerlich auf die Idee es für sich "arbeiten" zu lassen. Die Gewinne kann man natürlich aber dann nicht mehr angeben weil es ja schon geheim ist. Dann braucht der Fiskus nur noch einen Faden zu erwischen mit dem die Verstrickung zur Schlingfalle wird die man sich selbst gelegt hat. --Kharon 00:58, 24. Jun. 2013 (CEST)

Edeka in der DDR

Gab es Edeka in der DDR? In dieser Zuschrift möchte ich zur weiteren Klärung beitragen. Ich besitze einen postalisch gelaufenen Briefumschlag mit im Absenderfeld bedruckten EDEKA-Logo von 1922 der EDEKAZENTRALE eGmbH in 901 Karl-Marx-Stadt, Postfach 225. Abgestempelt am 17.06.70-21 Uhr. Der Briefumschlag ist gut erhalten. --88.75.77.219 20:57, 20. Jun. 2013 (CEST)

Hinweis: Für Fragen, die nicht direkt etwas mit Wikipedia zu tun haben, gibt es die Wikipedia:Auskunft. XenonX3 – (RIP Lady Whistler)) 21:11, 20. Jun. 2013 (CEST)
Hier wirst du am ehesten eine Antwort auf diese Detailfrage bekommen.--Losdedos (Diskussion) 21:21, 20. Jun. 2013 (CEST)
(a) Sensation? (b) Fake? (c) Nachlässigkeit? (d) Zweideutigkeit? 1922 <=> (1953–1990: Karl-Marx-Stadt). GEEZER... nil nisi bene 08:23, 24. Jun. 2013 (CEST)
Und: Sicher, dass es 17.06.70 lautet und nicht 17.06.90 gewesen sein koennte auf dem Stempel? -- Arcimboldo (Diskussion) 08:27, 24. Jun. 2013 (CEST)
Einen Irrtum aufklärend: In der Fragestellung wird das Logo (wohl eher: Signet, aber den Kampf kämpfe ich wohl vergebens) als in der Fassung von 1922 bezeichnet. Das heißt nicht, daß es 1923 ein neues gab. Oder in der DDR im Jahr 1960. --Freud DISK Konservativ 08:33, 24. Jun. 2013 (CEST)
Hier wurde das Thema auch schon diskutiert. GEEZER... nil nisi bene 08:34, 24. Jun. 2013 (CEST)
(BK) Ah, doch, per Google findet man diesen Thread: http://www.cosmiq.de/qa/show/3556054/Gab-es-in-der-DDR-Edeka/. Also gab es wohl tatsaechlich Edeka, die aber wohl keine Verbindungen zur West-Edeka mehr hatten. Eventuell wurde der Name noch aus Vor-DDR-Zeiten weitergefuehrt, aber unter DDR-kompatibler Rechtsform. - Arcimboldo (Diskussion) 08:36, 24. Jun. 2013 (CEST)

Türklinkeninstallation, welche Seite?

Gibts eine Vorschrift, wie eine Türklinke installiert werden muss? Ich meine damit die Anordnung postitive und negative Türklinke. In meiner neuen Mietwohnung war die positive auf der Gangseite angebracht. Ich habe erstmal alle umgebaut, damit die postitive sich auf der Zimmerseite befindet. Hintergrund: sollte die Madenschraube, die die beiden Türklinken verbindet abreißen, kann man wenigstens das Zimmer ohne fremde Hilfe verlassen. Andersrum sehe ich keine Logik drin. Aber gibt eine Vorschrift welche Seite wo hin muss/soll? Ich sehe fast immer (bei Besuch in fremden Wohnungen/Bürogebäuden) dass die positive Klinke sich außerhalb des Zimmers (auf der Gangseite) befindet. --194.138.39.54 12:10, 21. Jun. 2013 (CEST)

Überlegung: Wenn du in deinem Zimmer umkippst, den herbeieilenden Rettern die Made abreisst, dabei dass Gegenstück der Klinke in dein Zimmer fällt ... kann keiner mehr rein um dir zu helfen. 194.25.103.254 12:55, 21. Jun. 2013 (CEST)
Wenn ich schon umkippe, dann bleib ich vor der geschlossenen Tür liegen und bekomme noch ein paar Schläge beim Versuch der Türöffnung ab. Aber in deinem Fall ist es einfacher, von einer anderen Tür die Türklinge zu entfernen und die Tür zu öffnen. Wobei die ich behaupte, dass es viel unwahrscheinlicher ist, dass beide Umstände, Ohnmacht und Madeschraubenabriss gleichzeitig auftreten. Vielleicht Mal ausgenommen bei der alten Oma, die seit 50 Jahren die gleichen Zimmertüren hat. Dann bleibt ja immer noch Türentreten. -194.138.39.54 13:07, 21. Jun. 2013 (CEST)
Bei einem Herzinfarkt zählt jede Sekunde... Ich vermute, dass die DIN 18360 das regelt, komme (hier) aber nicht auf die entsprechenden Seiten, um das zu verifizieren. 194.25.103.254 13:24, 21. Jun. 2013 (CEST)
Oder aber DIN 18255. --тнояsтеn 14:00, 21. Jun. 2013 (CEST)
Hier steht:
  • Stiftteil: der Griff mit Drücker-Vierkantstift; außen montiert
  • Lochteil: der Griff mit Aussparung für den Drückervierkantstift und Fixierstift oder -schraube; innen montiert
Zudem kann man doch von beiden Seiten auch ohne Madenschraube die Tür öffnen. Einmal steckt man den o.g. Stiftteil rein (bzw. er steckt auch ohne die Madenschraube), das andere mal steckt man den Lochteil auf den Stiftteil (wieso sollte letzterer denn rausfallen ohne Schraube?) --тнояsтеn 14:10, 21. Jun. 2013 (CEST)
Ich weiß nicht, ob Herzinfakt hier der entscheidende Punkt sein sollte: Dass die Klinke abreißt, passiert schon so gut wie nie. Dass jemand in einem Raum gerade einen Herzinfakt hat, passiert - gemessen an den "Betriebsstunden eines Raums - auch so gut wie nie. Dass das jetzt auch noch beides genau gleichzeitig zusammen passieren soll, ist noch viel unwahrscheinlicher.
Ich setze die "männliche" Seite der Türklinke immer auf die Innenseite des Raums. Der Gedanke dahinter ist: Wenn sich dieser Splint, der die beiden Teile zusammenhalten soll, mal lockert, hab ich innen das Stück Klinke in der Hand, das ich problemlos wieder in die Tür stecken und sie damit trotzdem noch öffnen kann. Mit dem anderen Teil geht das nicht. --88.130.115.255 14:11, 21. Jun. 2013 (CEST)
@Thorsten naja den Lochteil draufstecken ist garnicht so einfach weil der Stiftteil ja lose in der Tür steckt und dann reinrutscht.
@115.255 naja das ist halt die Sache mit dem Erwartungswert wenn beide Sachen zusammen auftreten dann ist das sehr schlecht. Das die Madenschraube raus fällt ist garnicht mal so selten. Und die Herzinfakt(oä) Wahrscheinlichkeit auch schon relativ hoch. Da benutze ich lieber alle paar Jahre nen Schlüssel(im Vierkantloch verkantet) zum öffnen der Tür als wenn die Rettung erschwert wird. --Saehrimnir (Diskussion) 18:18, 21. Jun. 2013 (CEST)
Zur Wahrscheinlichkeit: Es reicht nicht, dass jemand im Raum einen Infarkt hat und gleichzeitig die Innenklinke abreißt. Es muss auch außerdem noch jemand außerhalb des Raumes sein und dieser Jemand muss mitbekommen, dass die Person im Raum einen Infarkt hat. Nun rechne mal jemand die Wahrscheinlichkeit aus, dass diese vier Ereignisse / Zustände gleichzeitig stattfinden. Vermutlich kommt man dabei darauf, dass falsche Türklinken statistisch doch erst so auf dem dritten oder vierten Platz kommen, nach High Heels, Pkw und Haushaltsleitern, die wohl alle mehr Opfer produzieren. Csedfa (Diskussion) 19:39, 21. Jun. 2013 (CEST)
Ehrlich gesagt, als Retter wäre mir das ggf. defekte Türschloss reichlich egal. Notfalls wird die Türe mit Gewalt geöffnet. Das geht schneller, als erst den Türdrücker wieder versuchen zu montieren. Wenn die Tür abgeschlossen ist, hast Du ja ein ähnliches Problem, das durch gewaltsame Öffnung der Tür leicht gelöst werden kann. Ein zerstörtes Türblatt ist schnell repariert; ein Menschenleben nicht und das ist ungleich viel mehr wert. --Rôtkæppchen68 20:33, 21. Jun. 2013 (CEST)

Mal als Zwischenfazit: Bisher ist noch kein plausibler Grund genannt worden, warum der Installateur die männliche Türklinke außen montiert, wo es doch im Fehlerfall einen Vorteil ha, wenns umgekehrt wäre (männliche Klinke auf Zimmerseite). Und es ist deutlich wahrscheinlicher, dass ich allein im Zimmer bin, wenn ich beim Türöffnen die Madenschraube abreiße und das Telefon außerhalb des Zimmers liegt, um Hilfe zu rufen (passiert lt. Murphy im eigenen Schlafzimmer, Morgens nach dem aufstehen, wenn um 8 Uhr ein wichtiger Termin ansteht ;-) ) -Der Ursprungsfragesteller -84.148.29.18 21:14, 21. Jun. 2013 (CEST)

[BK] Als Retter wirst du doch aber nicht gleich die Tür eintreten, sondern erst die Klinke benutzen... Es wird schon seinen Grund haben, dass es so montiert wird wie nach тнояsтеns Link beschrieben. Rettungsgründe sind dabei wohl am wahrscheinlichsten. Da haben sicher wieder furchtbar schlaue Und wieso denkt hier wieder keiner an die Kinder? Oder die Katzen, Hunde, Meerschweinchen, die nun im Zimmer eingeschlossen... Oder Tür fällt zu und der Griff ab (weil Made weg)... Wer drinnen ist (also gefangen), wird in den meisten Fällen doch Hilfe rufen können. Imho überwiegen die Vorteile, wenn das Vierkantteil von außen eingeführt wird.
"Statistisch gesehen" ist es allerdings wahrscheinlicher, dass man wegen eines abgefallenen Türgriffs nicht ans Telefon kommt. Deswegen sterben wird man sicher nur einmal... -- Ian Dury Hit me  21:40, 21. Jun. 2013 (CEST)

Was würdet ihr denn machenb, wenn es sich um eine Klinke handelt, die ein Oben und ein Unten hat? Der Hersteller stellt keine für DIN-R und DIN-L her. Also isses wurscht. Ihr könnt Probleme haben. --Löschbold (Del) 22:25, 21. Jun. 2013 (CEST)

Wolt' ich gerade anfügen. Die meisten mir bekannten Klinken lassen sich nur in einer Richtung einbauen... --Lexx105 (Diskussion) 22:50, 21. Jun. 2013 (CEST)
Wenn die Madenschraube oder die breite Seite des Splintes nach oben zeigt, ist der Türdrücker falschherum eingebaut. --Rôtkæppchen68 23:39, 21. Jun. 2013 (CEST)
Aus Kostengründen sind wohl moderne Standardklinken links oder rechts einzusetzen (spart vermutlich auch Lagerhaltung beim Hersteller, nur ein Produkt statt zwei). Ich schau grad auf eine Tür aus den 80er und dort ist die Madenschraube nicht ober oder unten, sondern seitlich. Austausch von männlicher und weiblicher Klinke problemlos möglich. Wann entwickelt jemand die geschlechtneutrale Klinke, also eine Klinke bei der der Vierkant in der Mitte halbiert ist und erst beim Zusammenbau sich ein Vierkant bildet. Ich sehe schon, auf diesem Gebiet gabs seit Jahrzehnten keine große Entwicklungen mehr ;-)
Aber um nochmal aufs Thema zurückzukommen, Zimmertüren gehen typischerweise ins Zimmer auf, d. h. man drückt gegen die Türklinke, damit kann die niemals abfallen, selbst wenn keine Madenschraube eingesetzt ist. Umgekehrt, beim Türöffnen von innen muss man an der Klinke ziehen und hält sich bei abgerissener Madenschraube in der Hand. Für mich bleibts unlogisch, das männliche Klinkenteil auf der "Druck"-Seite und nicht auf der "Zieh"-Seite der Tür zu installieren (beim Öffnen der Tür). -84.148.13.11 16:56, 22. Jun. 2013 (CEST)
In Sonderfällen werden auch Türdrücker mit Madenschraube auf beiden Seiten eingesetzt. Dann muss man nur noch den zur Dicke der Tür passenden Vierkant dazwischensetzen. Bei extradicken schalldichten Türen passen nurmale Türdrücker nicht. --Rôtkæppchen68 17:16, 22. Jun. 2013 (CEST)

Das sind ja Probleme. Ich montiere so dass die Schraube innen ist, damit mir keiner heimlich die Klinke klauen kann. Zudem montiere ich die Klinke so gut dass sie nie abfällt, auch nicht wenn jemandem im Raum was zustossen sollte und man Hilfe braucht. Madenschraube so festziehen damit sie nicht raus fällt sollte wohl möglich sein. --Netpilots -Φ- 22:13, 24. Jun. 2013 (CEST)

Auch wenn dein Beitrag recht sarkastisch ist, irgendwas ist da m. M n. dran. Bei abgeschlossenen Türen im (halb-)öffentlichen Bereich könnte man von außen die Klinken demontieren und hat ein Loch (das Vierkant) in den Raum. Freut bestimmt die Voyeure unter uns ;-)
Mir fällt noch ein, dass bei alten Türklinken, die eine Splint (statt moderner Madenschraube) haben, Kinder schonmal daran rumspielen und den Splint entfernen. Fazit: Kind will aus dem Zimmer raus und wegen fehlendem Splint hält es dann nur die weibliche Klinke in der Hand, will diese wieder draufstecken, stößt die männliche aus dem Vierkantloch und der "Spaß" beginnt (das Kind kriegt Panik). Daher mein Unverständnis, dass es gerade andersrum ist, also nicht kindersicher (ok, ich komm damit wahrscheinlich 50 Jahre zu spät). -194.138.39.60 08:08, 25. Jun. 2013 (CEST)

Icons

bei Verlinkungen von Webseiten auf dem Desktop werden einige Icons (auch von WP) durch einen grünen Erdball ersetzt. Woher kommt das und wie kann ich das unterbinden? --62.143.112.145 09:55, 22. Jun. 2013 (CEST)

In Windows wäre das: Maus drüber=> rechtsklick=> Eigenschaften => anderes Symbol --Kharon 11:55, 22. Jun. 2013 (CEST)
Es gibt auch eine Kontextmenü-Taste, siehe auch Kontextmenü. --77.185.211.94 13:43, 22. Jun. 2013 (CEST)
Kontextmenü gibt unter Eigenschaften nur einen Reiter mit "Allgemein". Was ist jetzt zu tun? --62.143.112.145 19:22, 22. Jun. 2013 (CEST)
Prüfen, ob das zu bearbeitende Icon zuvor markiert war oder nicht darauf geachtet und versehentlich das Kontextmenü für etwas anderes aufgerufen wurde. --84.191.135.123 22:54, 22. Jun. 2013 (CEST)
wenn ich die grünen Bälle auf dem Desktop rechts mausklicke, erscheinen unter Eigenschaften 5 Reiter: Allgemein, Webseite, Sicherheit, Details und andere Versionen. Wie soll ich was prüfen? Bei WP will ich das W, bei Amazon das A und bei booking.com das B haben. --62.143.112.145 09:00, 23. Jun. 2013 (CEST)
Was dir nach meinem Eindruck fehlt ist offenbar ein grundlegender Einführungskurs für den Umgang mit Computern und der Vorbereitung auf ein eigenständiges Weiterlernen (mit Betonung auf eigenständig). Das läßt sich hier (zumindest von mir) nicht ausgleichen. Was du willst, ist möglich: Icons kann man selbst gestalten und dann einer konkreten Verknüpfung zuordnen. In deinem Fall rate ich aber: RTFM --84.191.187.101 13:20, 23. Jun. 2013 (CEST)
deine beide Sätze stimmen nicht: der erste ist hinreichend wiederlegt durch mein Leben, der zweite gibt wieder was ich nicht gefragt habe. Lass es doch einfach wenn du keine Lust hast ... --62.143.112.145 16:45, 24. Jun. 2013 (CEST)
Auf dem Reiter Web Document bzw Webseite ist ein Button Change Icon…. Da klicken und eine auf Deinem System existierende .ico-Datei (oder .icl,.exe,.dll) auswählen, die das gewünschte Icon enthält. Am allerschnellsten kriege ich das Wunschicon aber, indem ich die gewünschte Website mit dem IE10 oder FF21 öffne und dann das Symbol vor der Adresszeile auf den Desktop ziehe. --Rôtkæppchen68 13:28, 23. Jun. 2013 (CEST)
danke --62.143.112.145 16:45, 24. Jun. 2013 (CEST)

Ektotherme Tiere

Was machen ektotherme (wechselwarme) Tiere im Winter? Halten sie Winterschlaf oder fressen sie?--Joël57 (Diskussion) 11:47, 22. Jun. 2013 (CEST)

Kann man so nicht Entweder-odern.
1. Hinweis: Suche mit Bildersuche ein Bild von einer Ringelnatter im Schnee.
2. Hinweis: Was machen (Tiefsee)Fische im Winter? :-) GEEZER... nil nisi bene 11:54, 22. Jun. 2013 (CEST)
3. Hinweis: " Im Allgemeinen, aber nicht ganz korrekt wird „wechselwarm“ auch mit „ektotherm“ gleichgesetzt, da die meisten wechselwarmen Tiere auch ektotherm sind und umgekehrt („wechselwarm“ bezieht sich auf die schwankende Körpertemperatur der Tiere, wohingegen „ektotherm“ beschreibt, wodurch diese Schwankungen zustande kommen)."
Und was ist deren Anpassung an den Winter? (Ein Link wäre schon gut.)--Joël57 (Diskussion) 11:58, 22. Jun. 2013 (CEST)
Erst dein Feed-back dazu - dann mehr. Auch mit meinen Kindern war ich grausam... GEEZER... nil nisi bene 12:00, 22. Jun. 2013 (CEST)
Ich brauche nicht zwei Beispiele, sondern was allgemeineres.--Joël57 (Diskussion) 12:02, 22. Jun. 2013 (CEST)
Kommt normalerweise drauf an wo. In den Tropen werden die im "Winter" kaum etwas anderes machen als im Sommer. In kälteren Regionen gehen viele soweit unter die Erde das sie nicht hartfrieren (z.B. Kröten). Andere Arten halten sich durch Bewegung und gute isolation warm (z.B. Pinguine). --Kharon 12:04, 22. Jun. 2013 (CEST)

Hier in der Schweiz muss es sein.--Joël57 (Diskussion) 12:05, 22. Jun. 2013 (CEST)

In der Schweiz gibt es sicher Kröten. --Kharon 12:08, 22. Jun. 2013 (CEST)
Ich will nicht Beispiele, sondern verschiedene Arten von Anpassungen. (nicht signierter Beitrag von Joël57 (Diskussion | Beiträge) 12:09, 22. Jun. 2013 (CEST))
Hab ich doch geschrieben. Die buddeln sich 1-2 meter unter die Erde, wo es nicht mehr friert. Im frühling kommen sie wieder raus. --Kharon 12:12, 22. Jun. 2013 (CEST)
Einige Arten haben auch einen Gefrierschutz: en:Freeze_tolerance#Natural_cryopreservation. -- 78.34.9.197 13:16, 22. Jun. 2013 (CEST)

Joël57, kennst du eigentlich das kleine Wort mit den zwei t? Ich meine nicht Toilette. --Xocolatl (Diskussion) 22:55, 23. Jun. 2013 (CEST)

Toilette hat 3 T. --Rôtkæppchen68 01:40, 24. Jun. 2013 (CEST)
Flott? --Optimum (Diskussion) 12:17, 24. Jun. 2013 (CEST)
Welches Wort ist es? Ich kenne es nicht.--Joël57 (Diskussion) 10:20, 25. Jun. 2013 (CEST)

Coca-Cola koffeinfrei

Hi. Im Artikel Getränkemarken der Coca-Cola Company ist zu lesen, dass 1984 in Deutschland "Coca-Cola koffeinfrei" eingeführt wurde. Diese Angabe findet man auch auf verschiedenen Seiten im Internet. Ich habe aber nirgends die Info gefunden, wann diese Sorte eingestellt wurde. Weiß das zufällig jemand? Wär sicher auch interessant für den Artikel. --APPER\☺☹ 22:31, 22. Jun. 2013 (CEST)

Das gibt es noch. Z. B. hier. --Mauerquadrant (Diskussion) 23:30, 22. Jun. 2013 (CEST)
Das ist Coca-Cola light koffeinfrei (sieht man auf dem Bild). Von light und neuerdings auch zero gibt es koffeinfreie Varianten, wohingegen die koffeinfreie Variante der normalen Coca-Cola anscheinend seit Jahren in Deutschland vom Markt ist. --APPER\☺☹ 23:56, 22. Jun. 2013 (CEST)
Ups, die Feinheit hatte ich jetzt nicht bemerkt. --Mauerquadrant (Diskussion) 00:10, 23. Jun. 2013 (CEST)
das war die mit dem goldenen logo soweit ich mich erinnere, die gabe es bei uns in der gegend so ca bis 1990 rum, aber quellen habe ich nicht dafür, sorry. es gab kurz nach der einführung auch noch eine coffeinfrei/light variante, die aber auch wieder eingestellt wurde, bis sie einige jahre später wieder kam. 212.90.151.90 15:12, 23. Jun. 2013 (CEST)

Nach lesen diverser Foren: in vielen Ländern weltweit gibt es auch noch reguläre gezucktere koffeinfreie Cola. Ersatzbeschaffung aus dem europäischen Nachbarländern scheint auch möglich. Nur das deutsche Einstellungsdatum fand ich nicht.. -- southpark 16:04, 23. Jun. 2013 (CEST) Einstellung in Deutschland vermutlich 2002. Dafür habe ich aber noch keinen harten Beleg.

Ich meine, die Coca-Cola koffeinfrei über Jahre noch in einem großen Getränkehandel gesehen zu haben. Es ist vielleicht eher ein Problem der Einzelhändler, sich die diversen Sorten im Lager zu stellen. --Atamari (Diskussion) 02:19, 24. Jun. 2013 (CEST)
Nein, ich denke nicht. Es findet sich nirgends auf der Webseite von Coca-Cola irgendwelche Hinweise auf die koffeinfreie Zucker-Version (siehe auch [23]). --APPER\☺☹ 00:49, 25. Jun. 2013 (CEST)

Ergänzung auch nach der Archivierung (falls mal jemand das Archiv durchsucht): Laut Nachfrage bei Coca-Cola war Coca-Cola koffeinfrei von 1984 bis 2000 im Handel. --APPER\☺☹ 15:55, 2. Jul. 2013 (CEST)

Offene Fragen über die QPNC-PAGE

Leider gibt es hier einige offene Fragen über die QPNC-PAGE, die bis jetzt noch niemand beantwortet hat. -- Karl Bednarik (Diskussion) 05:15, 23. Jun. 2013 (CEST).

Karl sollte keine Wikipedia:Theoriefindung betreiben und Bernd sollte Wikipedia:Interessenkonflikte vermeiden und jemand anders den Artikel schreiben lassen. --Rôtkæppchen68 12:45, 23. Jun. 2013 (CEST)
Hallo Rotkaeppchen68, an welcher Stelle habe ich Theoriefindung betrieben? Mit freundlichen Grüssen, -- Karl Bednarik (Diskussion) 10:36, 25. Jun. 2013 (CEST).
Hier. --Rôtkæppchen68 10:54, 25. Jun. 2013 (CEST)
Hallo Rotkaeppchen68, nein, das wird hier erklärt. Mit freundlichen Grüssen, -- Karl Bednarik (Diskussion) 11:14, 25. Jun. 2013 (CEST).

Massenhysterie?

Im Zusammenhang mit UFO-Sichtungen in Großstädten, Geisterphänomene oder der mittelalterlichen Hexenjagt, seltener auch mit der Kommunistenangst in den USA, lese ich immer wieder von Massenhysterie. Die Hysterie ist heute allerdings aus dem psychiatrisch-psychologischen Sprachgebrauch verschwunden. Deshalb frage ich mich, wer denn eigentlich als erster auf den Gedanken kam, es gebe sowas wie eine Massenhysterie und ob man dieses Phänomen schon mal in untersucht hat? --188.100.83.225 13:26, 23. Jun. 2013 (CEST)

Der engl. Begriff mass hysteria und auch der dt. Begriff Massenhysterie treten erst relativ spät auf (Sigmund, warst du das?). Heute sagt man eher kollektive Konversionsstörung. Beim Überfliegen auch etwas vom Einfluss der Spiegelneurone gefunden... GEEZER... nil nisi bene 14:52, 23. Jun. 2013 (CEST)
In Anlehnung ans Englische auch kollektives Zwangsverhalten. GEEZER... nil nisi bene 16:55, 23. Jun. 2013 (CEST)
ich sag immer gern: folie a döh! :-) --Heimschützenzentrum (?) 18:59, 23. Jun. 2013 (CEST)
Wenn - sagen wir mal - 7 Mill. Deutsche Mario Barth / Dieter Nuhr / Rosa Kugelburg / Johann König im TV sehen ... und alle lachen über etwa 1 h an etwa denselben Stellen - gruselige Vorstellung, gell? - ist das dann spatial getrenntes kollektives Zwangsverhalten? GEEZER... nil nisi bene 20:58, 23. Jun. 2013 (CEST)
Das Volk wird ja durch Vorlachen synchronisiert. Ich wüsst ja sonst auch nicht, an welchen Stellen es grad lustig ist. --RobTorgel (Diskussion) 14:02, 25. Jun. 2013 (CEST)

Unterschied zwischen Ruhrdeutsch und Rheinischer Regiolekt?

Mir ist nicht recht klar, in welcher Beziehung die unter Ruhrdeutsch und Rheinischer Regiolekt beschriebenen Sprachformen zueinander stehen. Die Verbreitungsgebiete scheinen sich jedenfalls zu überschneiden. Ist das Ruhrdeutsche ein Teil des Rheinischen Regiolekts? Die Beziehungen zwischen beiden Konzepten werden nirgendwo erläutert; jedenfalls habe ich nichts dazu gefunden. --Florian Blaschke (Diskussion) 23:10, 23. Jun. 2013 (CEST)

Im Grunde sind das willkürliche Zusammenfassungen einzelner Dialekte, hier anhand der Flüsse Ruhr und Rhein, die allerdings durchaus ähnlich sind. Die lokalen Vertreter gewichten naturgemäß die teils feinen, kleinen Unterschiede erheblich höher und würden nie die in der Regel stadtspezifischen und sogar ortsspezifischen Dialekte in eine übergeordnete Schublade werfen. --Kharon 01:11, 24. Jun. 2013 (CEST)
Das sind beides Bezeichnungen für Irgendwas. Und Rheinischer Regiolekt ist schon wieder solch Machwerk, wo der Umfang von Literatur und Einzelnachweisen kaschiert, daß vieles reine Spekulation und Theoriefindung ist. Das Ganze wird dadurch lächerlich, daß gleichzeitig einige Leute versuchen die Substanz/Eigenständigkeit des Ripuarischen und Westfälischen zu verteidigen. Aber Atze Schröder vs. Guido Cantz ist keine Substanz für solchen Vergleich. Angesichts der Mobilität der Leute, und der großen Bevölkerungsdichte werden die Gebiete der Überschneidungen immer größer. Dazu kommt dort die Überschneidung sowohl des Nieder- zum Mittedeutschen als auch des Niederfränkischen zum Niedersächsischen, was früher grob an der Einheitsplurallinie zu trennen war, die aber auch nicht entlang des Rheins verlief, sondern das Ruhrgebiet in der Mitte von Nord nach Süd durchtrennte. Siehe Westfälische Dialekte, die Kerngebiete von Ripuarisch und Westfälisch haben gefühlt bis heute Bestand. Was bei den beiden Dialekten draufgegangen ist, sind das Bergische und Niederfränkische. In den Regionen gibt es wohl die Überschneidung, die Du feststellst, oder?Oliver S.Y. (Diskussion) 01:33, 24. Jun. 2013 (CEST)
Lass das aber nicht die Bergischen hören, dass ihr Dialekt "draufgegangen" sei. :-) Natürlich stehen die kleineren Dialekte unter einem erheblichen Druck seitens der Großzentren Köln/Düsseldorf und Ruhrgebiet und sind im allmählichen Niedergang begriffen, aber hier in Solingen z.B. gibt es durchaus noch eine aktive Mundartszene. --Jossi (Diskussion) 15:31, 24. Jun. 2013 (CEST)
Dennoch sollte sich das mal einer anschauen. Im Artikel Rheinischer Regiolekt sind schon einige interessante Behauptungen zu finden, z. B. unter "Übernahmen aus dem Rheinischen ins Hochdeutsche": "Sie sind sich nicht eins − für: Sie sind uneinig." Der Nachweis, der dann angeführt wird, ist nichtssagend. Ob das wirklich aus dem Rheinischen kommt, müßte hingegen überprüft werden. Im Meckl. Wb. (Bd. 2, Sp. 708) ist für das Mecklenburgische unter dem Lemma "eins" 'einmal'(inklusive mittelniederdeutscher Belege!) angeführt: "4. als präd. Bestimmung bei warden und sin im Sinne von einig: 'alse wy des enes gheworden syn mid en unde se mid uns' (GÜSchwaan 1397) UB. 23, 315; kunn ... sick nich eins warden, weckehr Lied un weckehr Melodei em bäter tauseggen ded, de ein oder de anner BRI 5, 29". Der Schiller/Lübben (Mnd. Wörterbuch, Bd. 1, S.639) kennt das als "ein sîn" 'eines Sinnes sein, sich vertragen' bzw. "ein werden" 'sich einigen'. Für die Negation siehe z. B. aus einem Gesetz wegen Totschlags (1392) aus Holstein: "Konde se des nicht enes werden, so scholen de veer eenen overman to sik kesen".
Für das Hochdeutsche gibt es wiederum genügend Belege im Grimm, auch ohne Bezug auf das Rheinland, z. B. Hans Sachs: "zwen giengen hin und wurden éins." Man vergleiche auch die Bibelübersetzung Luthers: "Wo Zween vnter euch eines werden auff erden / warumb es ist / das sie bitten wöllen / Das sol jnen widerfaren / von meinem Vater im Himel." (Matth. 18, 19) Bei Luther findet sich auch die Negation: "Wo er nicht eins mit dem andern ist / da ist liebe vnd friede aus" (Auslegung der Epistel, 1530). Goethe schreibt im Bundeslied (zit. nach dem Faksimile von H2, wohl 1778 entstanden): "So glühet fröhlich heute /Seyd recht von Herzen eins. / Auf! Trinckt der Dauer Freude / Ein Glas des ächten Weins!" Noch deutlicher wird das in einer früheren Überlieferung: "Ihr seyd nun Eins ihr Beyde, / Und wir mit euch sind eins." --IP-Los (Diskussion) 17:57, 24. Jun. 2013 (CEST)

Ich spiele seit knapp 6 Monaten in einem Verein Handball. Ich bin sicher nicht der beste aber auch nicht der schlechteste Spieler im Team. Torerfolge gibt es regelmässig. :-) Was mir auf Fotos von mir bei entsprechenden Spielen auffällt, dass ich im Gegensatz zu anderen Spielern immer irgendwie "lässig" oder "gelangweilt" (weiss nicht ob das die richtige Beschreibung ist) aussehe, obwohl ich alles gebe. Bin vor Spielen total aufgeregt, während des Spiels fokussiert und hinter ausgelaugt. Ich habe mit einem Freund schon drüber gesprochen und er meint, dass ich Null Körperspannung habe. Kann man sich so was irgendwie aneignen? --141.15.33.1 08:39, 21. Jun. 2013 (CEST)

Das könnte klappen, wenn Du immer schön gerade sitzt, während Du mit Deinem Sohn für seine Klassenarbeiten lernst (s.o.) -- Geaster (Diskussion) 08:46, 21. Jun. 2013 (CEST)
Wie sieht es bei euch (du <=> die anderen) mit Verletzungen aus? GEEZER... nil nisi bene 09:29, 21. Jun. 2013 (CEST)
Woher weisst du? Hab seit letzter Woche Probleme mit einem Knie. Arzt meinte, dass es nur eine Zerrung ist. Ansonsten aber in den 6 Monaten nichts auffälliges. --141.15.33.1 09:55, 21. Jun. 2013 (CEST)
Na ja, Sport geht meist nie ohne Verletzungen ab.
Der Hintergrund ist folgender: Ob du gespannt / cool / dynamisch (wie die anderen) aussiehst/daherkommst - oder eher lässig / gelangweilt spielt keine Rolle, solange dir das Spiel Spass macht, du deine Tore machst und du dich nicht verletzt. Personen mit "hoher Körperspannung" (whatever that means...) verletzen sich häufiger (deshalb hatte ich nach den anderen) gefragt. Man sollte - speziell wenn man schon länger gespielt hat - dem "Körpergefühl" vertrauen. Wenn du als lässiger Schlaffi deine Tore machst ist das doch völlig OK.
Oder willst du es "professionell" machen - und hast einen Coach und einen Onkel Doktor an der Seite? Dann her mit mehr Dynamik und Körperspannung ! :-) GEEZER... nil nisi bene 10:43, 21. Jun. 2013 (CEST)
Bei hoher Körperspannung steigt das Verletzungsrisiko? Das kann man wo nachlesen? (Ich kenne das nur andersherum, Beleg: Fehlanzeige ;-) .) Anka Wau! 16:03, 21. Jun. 2013 (CEST)
Ich bin auch verwundert, bei meinem Sport (Akrobatik) ist die Körperspannung sowas wie der heilige Grahl, nach dem alle streben und den man kaum erreichen kann (und ohne den die Figuren nicht gut klappen). Was wir allerdings nicht brauchen können sind Verspannungen, die machen uns schon Probleme. Wenn jemand in den kurzen Pausen zwischen einzelnen Belastungen eine "schlappe" Haltung hat, ist das (besonders bei Auftritten) nicht sonderlich hübsch, aber eher unproblematisch. Wenn die "lässige" Haltung aber in der Belastung auftritt korrigieren wir gleich, um eine Fehlbelastung der Gelenke zu vermeiden. Grüße --RalfDA (Diskussion) 16:15, 21. Jun. 2013 (CEST)
Schreiben wir doch erstmal Körperspannung (wiederholt: whatever that means...)... Dann wird es vielleicht klarer... ;-) GEEZER... nil nisi bene 13:01, 23. Jun. 2013 (CEST)
Wenn ich es könnte wäre ich sofort dabei, aber das was ich beschreiben kann sind die Symptome, das was man beobachtet, aber trotz aller langjährigen Versuche kann ich niemandem eine Anleitung in die Hand geben wie man die (bei uns positive) Körperspannung lernt (nur Umwege über Tanz & Ballettunterricht, oder stetiges Üben und Abwarten bis man es auf dem Weg zum sehr fortgeschrittenen Sportler "nebenbei" lernt). Wäre das was wir wissen für einen Artikel ausreichend ? oder sollten wir lieber erst mal noch in der Abteilung Theoriefindung bleiben. Grüße --RalfDA (Diskussion) 14:25, 24. Jun. 2013 (CEST)
Erster Punkt wäre: Wie ist "Körperspannung" medizinisch/physiologisch definiert. Das erscheint mir schwierig, da "verspannte Muskeln", "steife Sehnen und Gelenke" etc. im Wege stehen. Vielleicht hat es mit unbewusster (aber trainierbarer) Körperhaltung zu tun? Bei "Spannung" bin ich kein Fachmann, eher beim Gegenteil... :-) GEEZER... nil nisi bene 15:18, 24. Jun. 2013 (CEST)
Sportler entwickeln im Laufe der Jahre ein Gefühl dafür wann, und vor allem in welcher Situation, sie Spannung aufbauen müssen. Körperspannung im "richtigen" Moment ist verletzungsmindernd, da eine angespannte Muskulatur die Gelenke und Sehnen vor Überbelastung (Überdehnen etc.) schützt. In vielen Sportarten ist dies sogar Grundvorausetzung bestimmte Situationen überhaupt heil zu überstehen. Ein Boxer der seinen Kopf nicht stabilisiert, wird seinen Sport nicht lange ausüben. In bestimmten (Kampf)sportarten, zu denen im weitesten Sinne auch Handball oder Fußball zählen, ist die Körperspannung Voraussetzung unverletzt aus einen Zweikampf heraus zu kommen. Und der Zweikampf ist dort nicht die Ausnahme sondern die Regel. Jeder Fußballer weiß: Gehe ich mit lockerer Beinmuskulatur in den Preßschlag, ist das Spiel und manchmal auch die Laufbahn beendet; oft kann man als Unbeteiligter - im Augenblick, nur wegen der fehlenden Körperspannung - bereits erkennen: Jetz' knallst, da kommt der nicht unbeschadet raus. Selbstverständlich bringt es auch nichts, sich im ständigen körperlichen Alarmzustand zu befinden. Dann gibts die erwähnten Muskelverspannungen. Ein trainierter Sportler entwickelt ein Gefühl dafür wann er seine Konzentration in Anspannung umwandeln muß. Kann er dies nicht entwickeln, ist dies nicht sein Sport und er sollte sich einen anderen aussuchen... ThomasStahlfresser 11:56, 25. Jun. 2013 (CEST)
Das macht schon mal vieles klarer. Damit wäre es dann so etwas wie eine "unbewusste aber sinnvolle Muskelanspannung in bestimmten Situationen"? "Temporär optimal automatisierte An- und Entspannung" ? "Instinktives Körpergefühl" ?
Man sollte mal einen Sportmediziner fragen, ob es dafür einen Begriff gibt. "Körperspannung" scheint es aber nicht zu sein, da findet man zuviel Verschiedenes.
Dann wäre die Antwort auf obige Frage: Üben, üben, üben, üben, üben, da bei der Schnelligkeit der Reaktionen kein rationales Denken mehr möglich - sondern sogar hinderlich ist. Eagleman nennt das zombie systems oder automated alien systems :-) GEEZER... nil nisi bene 16:36, 25. Jun. 2013 (CEST)
Ich hab mir aus meinen Sporterfahrungen (verschiedene Tänze, Klettern) folgenden Reim darauf gemacht: Wenn manche Leute von Körperspannung reden, meinen sie zentral am Körper gelagerte Muskulatur gegenüber der an den Extremitäten. Die ist einerseits trainierbar, aber hat auch viel mit innerer Haltung im Alltag zu tun: man scheint sich diese "lässige Haltung" von der du sprichst anzugewöhnen und ist z.B. zu faul gerade zu stehen (weils anstrengend ist - probiers mal ein paar Minuten aus, nach Korrektur von außen). Dadurch werden die skelettnahen Muskeln unterfordert. Um meine Vorstellung zu veranschaulichen: stell dir vor jmd trainiert den Bizeps allein mit abgestützten Ellenbogen (z.B. http://www.muskelbody.de/images/biz_04.jpg ). Wenn er dann aber eine schwere Sache heben soll, scheitert eher sein Rücken, auch wenn er sonst 50kg oder mehr stemmt. Konsequenz daraus ist also, dass man in seinen Bewegungsmöglichkeiten, also der Wendigkeit und damit Schnelligkeit im Kleinen eingeschränkt ist. (Folgendes sind eher Hypothesen von mir: Zentralgelagerte Muskulatur scheint auch für effizientere Bewegungsabläufe zu sorgen: um die fehlende Körperspannung zu kompensieren sind die Muskeln der Extremitäten überausgeprägt.)
Vielleicht hilft in deinem Fall aber auch eine Konzentration, die schnelle Aktionen ermöglicht, also eher nach Reaktivkraft oder Schnellkraft suchen? -- Amtiss, SNAFU ? 20:18, 25. Jun. 2013 (CEST)

Auslieferungsantrag und weitere Straftat im Gastland

Aus aktuellem Anlass kam ich auf folgendes fiktives Szenario:
Ein Mensch befindet sich in einem fremden Land (z.B. Deutschland) und wird in seinem Heimatland (z.B. USA) wegen eines Verbrechens gesucht. Nun stellt das Heimatland einen Auslieferungsantrag. Vor Auslieferung begeht er jedoch im Gastland ebenfalls eine Straftat. Was passiert jetzt in der Regel? Wird die Auslieferung verzögert und erst ein Verfahren im Inland durchgeführt, bis zu einem Schuld- oder Freispruch? Wird er sogar zunächst die Strafe absitzen und dann ausgeliefert (oder sofort nach Beendigung des Verfahrens)? Oder besteht Spielraum und er kann ggf. nach Abwägung trotzdem sofort abgeschoben werden und bleibt dann ggf. für seine Straftat im Gastland straflos oder wird im Heimatland dafür angeklagt (oder später wieder zurück überstellt)? Solche Fälle gab es doch sicher schon. Gefunden habe ich aber nur, dass bei Auslieferungsersuchen zweier Länder das Gastland entscheiden darf, wem es die Person ausliefert. Hätte man so die Möglichkeit, Auslieferungen zu verzögern? --StYxXx 20:49, 23. Jun. 2013 (CEST)

also bei Law & Order war es mal so, dass 1. Kanada die Auslieferung verweigert, wenn die Todesstrafe, die man in Kanada für illegal hält, mit hoher Sicherheit droht, und dass 2. bei zwei verschiedenen Verbrechen in zwei verschiedenen Staaten der USA das wichtigere Verbrechen zuerst zu verhandeln sein soll (aber nicht unbedingt... es lag im Ermessen der New Yorker...)... :-) --Heimschützenzentrum (?) 21:16, 23. Jun. 2013 (CEST)
Ich habe keinen Fernseher und kann deshalb nicht mitreden, wie das bei Law&Order in Kanada gemacht würde. Auch ansonsten habe ich keine Ahnung, würde mich in Deutschland aber gemäß Nr. 145 der Richtlinie für den Verkehr mit dem Ausland in strafrechtlichen Angelegenheiten (RiVASt) an den §§ 154b und 456a StPO orientieren und ggf. auch an § 9 IRG. Bedeutet: Einiger Spielraum für Staatsanwaltschaften und Gerichte, aber in der Regel zunächst deutsches Strafverfahren, dann Aussetzung der Strafvollstreckung und Auslieferung. 2 BvR 66/00 zeigt aber auch, dass das durchaus zu praktischen Problemen führen kann. Und ich weiß nicht, wie weit die Details durch bilaterale Abkommen abgeändert sind. --Rudolph Buch (Diskussion) 22:58, 23. Jun. 2013 (CEST)
Wie so oft bei leider zu unpräzise gehaltenen Fragen kann die Antwort nur lauten: Es komm darauf an.
In diesem Fall kommt es vor allem darauf an, welche bilateralen Vertragswerke greifen. Für Deutschland gilt grundlegend das Gesetz über die internationale Rechtshilfe in Strafsachen (IRG). Bei Auslieferungen innerhalb der Europäischen Union wäre aber auch die sogenannte Polizeiliche und justizielle Zusammenarbeit in Strafsachen (PJZS) von Belang. Bei einer Auslieferung in die USA würde auch das Auslieferungsabkommen greifen, das zuletzt im Oktober 2009 zwischen der Europäischen Union und den Vereinigten Staaten neu formuliert wurde. Grundsätzlich können (nach §§ 5 ff. StGB) auch im Ausland begangene Verbrechen in Deutschland verfolgt werden. Und es gab auch Fälle, in denen ein Straftäter die Strafe in einem Land erst verbüßen mußte und dann zur einem Prozeß mit möglicher Veruteilung und Strafverbüßung an ein anderes Land ausgeliefert wurde, das wegen eines Verbrechens in diesem Land ein Auslieferungsersuchen gestellt hatte. Eine weitere Möglichkeit ist, daß Länder zusätzliche Absprachen treffen (beispielsweise durch Gerichtsauflagen) oder sich darüber einigen, in welchem Land jemand zur Strafverbüßung verbleiben soll.
Ein anschauliches Fallbeispiel: Rolf Clemens Wagner wurde im November 1979 in Zürich nach einem Banküberfall festgenommen, im September 1980 von einem Geschworenengericht in Winterthur zu einer lebenslangen Freiheitsstrafe verurteilt, im September 1983 jedoch nach einem Entscheid des schweizerischen Bundesgerichtes an die BRD ausgeliefert und an die JVA Düsseldorf überstellt.
Im März 1985 wurde er dann vom Düsseldorfer Oberlandesgericht zu zweimal lebenslanger Haftstrafe verurteilt. Der Spiegel schrieb 1985 dazu: "Die Schweizer Behörden bewilligten zwar die Auslieferung, doch nur mit rechtlichen Einschränkungen, an die auch die Düsseldorfer Richter gebunden waren. So durfte Wagner nicht wegen Mitgliedschaft in einer terroristischen Vereinigung verfolgt und bestraft werden, weil es ein solches Delikt im Schweizer Recht nicht gibt. Daran hielt sich der Düsseldorfer Strafsenat nur formal. (...) Gleichwohl ist der Düsseldorfer Richterspruch - abgesehen von dem Kunstgriff, mit dem jene Auslieferungsbeschränkung unterlaufen wurde - das von der Beweislage her am besten belegte, am sorgfältigsten begründete und deshalb wohl auch überzeugendste Urteil aller einschlägigen RAF-Prozesse."
Das Urteil hatte dann 1986 vor dem BGH allerdings keinen Bestand. Die sogenannte Stimmenfallen-Entscheidung des 3. Strafsenates (BGHSt. 34, 39) hielt eine heimlich und unter einem Vorwand erstellte Stimmenprobe des Rolf Clemens Wagner für unverwertbar und verwies an einen anderen Senat des OLG Düsseldorf zurück. Es folgen weitere Verurteilungen 1987, wiederum zu zweimal lebenslang und 1993 als Folge von mit Straferlaß verbundenen Kronzeugenaussagen von in die DDR geflüchteten RAF-Aussteigern nochmals zu 12 Jahren Gefängnis. Wagner wurde von Bundespräsident Johannes Rau begnadigt und nach 24 Jahren im Dezember 2003 entlassen.
Bei seiner ersten Verhaftung im Mai 1978 in Zagreb (zusammen mit anderen Gesuchten) hatte Wagner übrigens noch davon profitiert, daß das Auslieferungsabkommen zwischen der BRD und Jugoslawien nicht wirksam wurde. Jugoslawien verlangte im Gegenzug die Auslieferung des Exilkroaten Stjepan Bilandzic und sieben seiner Gesinnungsgenossen. Bilandzic führte den am 1. Juni 1976 vom Innenminister verbotenen deutschen Zweig der rechtsextremen terroristischen Organisation Hrvatski narodni otpor. Obwohl ein deutsches Gericht zumindest die Auslieferung Bilandzics für zulässig erklärt hatte verweigerte die deutsche Bundesregierung dessen Auslieferung und so konnte Rolf Clemens Wagner zusammen mit den anderen in Zagreb Verhafteten nach sechs Monaten das Gefängnis verlassen und von Jugoslawien aus über Bagdad in den Jemen ausreisen. --84.191.187.101 02:39, 24. Jun. 2013 (CEST)
Aus einer anderen Frage noch ein auch für diese Frage erhellendes Interview mit Kai Ambos, Professor für Strafrecht, Strafprozessrecht, Rechtsvergleichung und internationales Strafrecht an der Georg August Universität Göttingen und Richter am Landgericht Göttingen. --84.191.139.105 11:49, 26. Jun. 2013 (CEST)

"Festgezurrte" Berghütten

Bei diesem Foto ist mir mal wieder aufgefallen, dass Berghütten des Öfteren mit Stahlseilen abgespannt sind. Die Seile hängen ja etwas durch, wozu soll das gut sein? Gegen Windlasten, Schneeverwehungen oder zum Gebetsfahnen aufhängen? ;) In dieser Größenordnung macht Ersteres wohl durchaus Sinn, aber halten ein paar Stahlseile einen großen Dachstuhl vorm Wegfliegen auf (zumal dieser sich schon ein paar Zentimeter bewegen müsste, um die Seile zu spannen)? --тнояsтеn 09:41, 21. Jun. 2013 (CEST)

Guttenberghaus
Vermutlich ist wirklich nur der Unterbau des Guttenberghaus aus Stein und das Obergeschoss aus Holz. Ich tippt auf die enormen Windkräfte, die die exponierte Lage mit sich bringen. -- sk (Diskussion) 10:46, 21. Jun. 2013 (CEST)
Hatte ich ja auch schon vermutet, aber wieso sind die Stahlseile relativ locker gespannt? (Oder sind die im Winter strammer?) --тнояsтеn 16:11, 23. Jun. 2013 (CEST)
Stichwort Wärmeausdehnung. Wenn sie im Sommer nicht locker wären würden sie im Winter die Bude auseinander reißen. -- sk (Diskussion) 16:40, 25. Jun. 2013 (CEST)
Ein 10 Meter langes Stahlseil ändert die Länge bei einem ΔT von 50 K um rund 6 Millimeter. Hmm... --тнояsтеn 20:54, 25. Jun. 2013 (CEST)
Sie hängen auch wegen des Eigengewichts durch. --Giftzwerg 88 (Diskussion) 21:38, 25. Jun. 2013 (CEST)
Diese 6 Millimeter Längenänderung machen einen Durchhang d von bis zu 173 Millimetern aus (als Dreieck gerechnet): . --Rôtkæppchen68 22:06, 27. Jun. 2013 (CEST)
Als Kettenlinie mit Excel und Zielwertsuche gerechnet kommen 149 Millimeter Durchhang raus. --Rôtkæppchen68 22:35, 27. Jun. 2013 (CEST)
Im Winter sind die Seile wohl wirklich recht straff. Danke für alle Anworten, ich nehm das aml so hin ;) --тнояsтеn 13:04, 28. Jun. 2013 (CEST)